2003-2014考研真题

**2014真题
Directions:
  Read the following text. Choose the best word(s) for each numbered blank and mark A,B,C or D on the ANSWER SHEET.(10 points)
  As many people hit middle age, they often start to notice that their memory and mental clarity are not what they used to be. We suddenly can’t remember 1 we put the keys just a moment ago, or an old acquaintance’s name, or the name of an old band we used to love. As the brain 2, we refer to these occurrences as “senior moments.” 3 seemingly innocent, this loss of mental focus can potentially have a (n) 4 impact on our professional, social, and personal 5.
  Neuroscientists, experts who study the nervous system, are increasingly showing that there’s actually a lot that can be done. It 6 out that the brain needs exercise in much the same way our muscles do, and the right mental 7 can significantly improve our basic cognitive 8. Thinking is essentially a 9 of making connections in the brain. To a certain extent, our ability to 10 in making the connections that drive intelligence is inherited. 11, because these connections are made through effort and practice, scientists believe that intelligence can expand and fluctuate 12 mental effort.
  Now, a new Web-based company has taken it a step 13 and developed the first “brain training program” designed to actually help people improve and regain their mental 14.
  The Web-based program 15 you to systematically improve your memory and attention skills. The program keeps 16 of your progress and provides detailed feedback 17 your performance and improvement. Most importantly, it ___18___modifies and enhances the games you play to 19 on the strengths you are developing—much like a(n) ___20___exercise routine requires you to increase resistance and vary your muscle use.
  1. [A]where [B]when [C]that [D]why
  2. [A]improves [B]fades [C]recovers [D]collapses
  3. [A]If [B]Unless [C]Once [D]While
  4. [A]uneven [B]limited [C]damaging [D]obscure
  5. [A]wellbeing [B]environment [C]relationship [D]outlook
  6. [A]turns [B]finds [C]points [D]figures
  7. [A]roundabouts [B]responses [C]workouts [D]associations
  8. [A]genre [B]functions [C]circumstances [D]criterion
  9. [A]channel [B]condition [C]sequence [D]process
  10. [A]persist [B]believe [C]excel [D]feature
  11. [A] Therefore [B] Moreover [C] Otherwise [D] However
  12. [A]according to [B]regardless of [C]apart from [D]instead of
  13. [A]back [B]further [C]aside [D]around
  14. [A]sharpness [B]stability [C]framework [D]flexibility
  15. [A]forces [B]reminds [C]hurries [D]allows
  16. [A]hold [B]track [C]order [D]pace
  17. [A]to [B]with [C]for [D]on
  18. [A]irregularly [B]habitually [C]constantly [D]unusually
  19. [A]carry [B]put [C]build [D]take

  1. [A]risky [B]effective [C]idle [D]familiar
    Section Ⅱ Reading Comprehension
      Part A
      Directions:
      Read the following four texts. Answer the questions below each text by choosing A, B, C or D. Mark your answers on the ANSWER SHEET. (40 points)
      Text 1
      In order to “change lives for the better” and reduce “dependency” George Osborne, Chancellor of the Exchequer, introduced the “upfront work search” scheme. Only if the jobless arrive at the jobcentre with a CV, register for online job search, and start looking for work will they be eligible for benefit and then they should report weekly rather than fortnightly. What could be more reasonable?
      More apparent reasonableness followed. There will now be a seven-day wait for the jobseeker’s allowance. “Those first few days should be spent looking for work, not looking to sign on.” he claimed. “We’re doing these things because we know they help people stay off benefits and help those on benefits get into work faster.” Help? Really? On first hearing, this was the socially concerned chancellor, trying to change lives for the better, complete with “reforms” to an obviously indulgent system that demands too little effort from the newly unemployed to find work, and subsidises laziness. What motivated him, we were to understand, was his zeal for “fundamental fairness”— protecting the taxpayer, controlling spending and ensuring that only the most deserving claimants received their benefits.
      Losing a job is hurting: you don’t skip down to the jobcentre with a song in your heart, delighted at the prospect of doubling your income from the generous state. It is financially terrifying, psychologically embarrassing and you know that support is minimal and extraordinarily hard to get. You are now not wanted; you support is minimal and extraordinarily hard to get. You are now not wanted; you are now excluded from the work environment that offers purpose and structure in your life. Worse, the crucial income to feed yourself and your family and pay the bills has disappeared. Ask anyone newly unemployed what they want and the answer is always: a job.
      But in Osborneland, your first instinct is to fall into dependency — permanent dependency if you can get it — supported by a state only too ready to indulge your falsehood. It is as though 20 years of ever-tougher reforms of the job search and benefit administration system never happened. The principle of British welfare is no longer that you can insure yourself against the risk of unemployment and receive unconditional payments if the disaster happens. Even the very phrase “jobseeker’s allowance” — invented in 1996 — is about redefining the unemployed as a “jobseeker” who had no mandatory right to a benefit he or she has earned through making national insurance contributions. Instead, the claimant receives a time-limited “allowance,” conditional on actively seeking a job; no entitlement and no insurance, at £71.70 a week, one of the least generous in the EU.
      21. George Osborne’s scheme was intended to
      [A]provide the unemployed with easier access to benefits.
      [B]encourage jobseekers’ active engagement in job seeking.
      [C]motivate the unemployed to report voluntarily.
      [D]guarantee jobseekers’ legitimate right to benefits.
      22. The phrase, “to sign on” (Line 3, Para. 2) most probably means
      [A]to check on the availability of jobs at the jobcentre.
      [B]to accept the government’s restrictions on the allowance.
      [C]to register for an allowance from the government.
      [D]to attend a governmental job-training program.
      23. What prompted the chancellor to develop his scheme?
      [A]A desire to secure a better life for all.
      [B]An eagerness to protect the unemployed.
      [C]An urge to be generous to the claimants.
      [D]A passion to ensure fairness for taxpayers.
      24. According to Paragraph 3, being unemployed makes one feel
      [A]uneasy
      [B]enraged.
      [C]insulted.
      [D]guilty.
      25. To which of the following would the author most probably agree?
      [A]The British welfare system indulges jobseekers’ laziness.
      [B]Osborne’s reforms will reduce the risk of unemployment.
      [C]The jobseekers’ allowance has met their actual needs.
      [D]Unemployment benefits should not be made conditional.
      Text 2
      All around the world, lawyers generate more hostility than the members of any other profession—with the possible exception of journalism. But there are few places where clients have more grounds for complaint than America.
      During the decade before the economic crisis, spending on legal services in America grew twice as fast as inflation. The best lawyers made skyscrapers-full of money, tempting ever more students to pile into law schools. But most law graduates never get a big-firm job. Many of them instead become the kind of nuisance-lawsuit filer that makes the tort system a costly nightmare.
      There are many reasons for this. One is the excessive costs of a legal education. There is just one path for a lawyer in most American states: a four-year undergraduate degree in some unrelated subject, then a three-year law degree at one of 200 law schools authorized by the American Bar Association and an expensive preparation for the bar exam. This leaves today’s average law-school graduate with $100,000 of debt on top of undergraduate debts. Law-school debt means that many cannot afford to go into government or non-profit work, and that they have to work fearsomely hard.
      Reforming the system would help both lawyers and their customers. Sensible ideas have been around for a long time, but the state-level bodies that govern the profession have been too conservative to implement them. One idea is to allow people to study law as an undergraduate degree. Another is to let students sit for the bar after only two years of law school. If the bar exam is truly a stern enough test for a would-be lawyer, those who can sit it earlier should be allowed to
      do so. Students who do not need the extra training could cut their debt mountain by a third.
      The other reason why costs are so high is the restrictive guild-like ownership structure of the business. Except in the District of Columbia, non-lawyers may not own any share of a law firm. This keeps fees high and innovation slow. There is pressure for change from within the profession, but opponents of change among the regulators insist that keeping outsiders out of a law firm isolates lawyers from the pressure to make money rather than serve clients ethically.
      In fact, allowing non-lawyers to own shares in law firms would reduce costs and improve services to customers, by encouraging law firms to use technology and to employ professional managers to focus on improving firms’ efficiency. After all, other countries, such as Australia and Britain, have started liberalizing their legal professions. America should follow.
      26.a lot of students take up law as their profession due to
      [A]the growing demand from clients.
      [B]the increasing pressure of inflation.
      [C]the prospect of working in big firms.
      [D]the attraction of financial rewards.
      27.Which of the following adds to the costs of legal education in most American states?
      [A]Higher tuition fees for undergraduate studies.
      [B]Admissions approval from the bar association.
      [C]Pursuing a bachelor’s degree in another major.
      [D]Receiving training by professional associations.
      28.Hindrance to the reform of the legal system originates from
      [A]lawyers’ and clients’ strong resistance.
      [B]the rigid bodies governing the profession.
      [C]the stem exam for would-be lawyers.
      [D]non-professionals’ sharp criticism.
      29.The guild-like ownership structure is considered “restrictive”partly because it
      [A]bans outsiders’ involvement in the profession.
      [B]keeps lawyers from holding law-firm shares.
      [C]aggravates the ethical situation in the trade.
      [D]prevents lawyers from gaining due profits.
      30.In this text, the author mainly discusses
      [A]flawed ownership of America’s law firms and its causes.
      [B]the factors that help make a successful lawyer in America.
      [C]a problem in America’s legal profession and solutions to it.
      [D]the role of undergraduate studies in America’s legal education.
     Text 3
      The US$3-million Fundamental physics prize is indeed an interesting experiment, as Alexander Polyakov said when he accepted this year’s award in March. And it is far from the only one of its type. As a News Feature article in Nature discusses, a string of lucrative awards for researchers have joined the Nobel Prizes in recent years. Many, like the Fundamental Physics Prize, are funded from the telephone-number-sized bank accounts of Internet entrepreneurs. These benefactors have succeeded in their chosen fields, they say, and they want to use their wealth to draw attention to those who have succeeded in science.
      What’s not to like? Quite a lot, according to a handful of scientists quoted in the News Feature. You cannot buy class, as the old saying goes, and these upstart entrepreneurs cannot buy their prizes the prestige of the Nobels, The new awards are an exercise in self-promotion for those behind them, say scientists. They could distort the achievement-based system of peer-review-led research. They could cement the status quo of peer-reviewed research. They do not fund peer-reviewed research. They perpetuate the myth of the lone genius.
      The goals of the prize-givers seem as scattered as the criticism. Some want to shock, others to draw people into science, or to better reward those who have made their careers in research.
      As Nature has pointed out before, there are some legitimate concerns about how science prizes—both new and old—are distributed. The Breakthrough Prize in Life Sciences, launched this year, takes an unrepresentative view of what the life sciences include. But the Nobel Foundation’s limit of three recipients per prize, each of whom must still be living, has long been outgrown by the collaborative nature of modern research—as will be demonstrated by the inevitable row over who is ignored when it comes to acknowledging the discovery of the Higgs boson. The Nobels were, of course, themselves set up by a very rich individual who had decided what he wanted to do with his own money. Time, rather than intention, has given them legitimacy.
      As much as some scientists may complain about the new awards, two things seem clear. First, most researchers would accept such a prize if they were offered one. Second, it is surely a good thing that the money and attention come to science rather than go elsewhere, It is fair to criticize and question the mechanism—that is the culture of research, after all—but it is the prize-givers’ money to do with as they please. It is wise to take such gifts with gratitude and grace.
      31. The Fundamental Physics Prize is seen as
      [A]a symbol of the entrepreneurs’ wealth.
      [B]a possible replacement of the Nobel Prizes.
      [C]an example of bankers’ investments.
      [D]a handsome reward for researchers.
      32. The critics think that the new awards will most benefit
      [A]the profit-oriented scientists.
      [B]the founders of the new awards.
      [C]the achievement-based system.
      [D]peer-review-led research.
      33. The discovery of the Higgs boson is a typical case which involves
      [A]controversies over the recipients’ status.
      [B]the joint effort of modern researchers.
      [C]legitimate concerns over the new prizes.
      [D]the demonstration of research findings.
      34. According to Paragraph 4,which of the following is true of the Nobels?
      [A]Their endurance has done justice to them.
      [B]Their legitimacy has long been in dispute.
      [C]They are the most representative honor.
      [D]History has never cast doubt on them.
      35.The author believes that the now awards are
      [A]acceptable despite the criticism.
      [B]harmful to the culture of research.
      [C]subject to undesirable changes.
      [D]unworthy of public attention.
      Text 4
      “The Heart of the Matter,” the just-released report by the American Academy of Arts and Sciences (AAAS), deserves praise for affirming the importance of the humanities and social sciences to the prosperity and security of liberal democracy in America. Regrettably, however, the report’s failure to address the true nature of the crisis facing liberal education may cause more harm than good.
      In 2010, leading congressional Democrats and Republicans sent letters to the AAAS asking that it identify actions that could be taken by “federal, state and local governments, universities, foundations, educators, individual benefactors and others” to “maintain national excellence in humanities and social scientific scholarship and education.” In response, the American Academy formed the Commission on the Humanities and Social Sciences. Among the commission’s 51 members are top-tier-university presidents, scholars, lawyers, judges, and business executives, as well as prominent figures from diplomacy, filmmaking, music and journalism.
      The goals identified in the report are generally admirable. Because representative government presupposes an informed citizenry, the report supports full literacy; stresses the study of history and government, particularly American history and American government; and encourages the use of new digital technologies. To encourage innovation and competition, the report calls for increased investment in research, the crafting of coherent curricula that improve students’ ability to solve problems and communicate effectively in the 21st century, increased funding for teachers and the encouragement of scholars to bring their learning to bear on the great challenges of the day. The report also advocates greater study of foreign languages, international affairs and the expansion of study abroad programs.
      Unfortunately, despite 2½ years in the making, "The Heart of the Matter" never gets to the heart of the matter: the illiberal nature of liberal education at our leading colleges and universities. The commission ignores that for several decades America's colleges and universities have produced graduates who don’t know the content and character of liberal education and are thus deprived of its benefits. Sadly, the spirit of inquiry once at home on campus has been replaced by the use of the humanities and social sciences as vehicles for publicizing “progressive,” or left-liberal propaganda.
      Today, professors routinely treat the progressive interpretation of history and progressive public policy as the proper subject of study while portraying conservative or classical liberal ideas—such as free markets and self-reliance—as falling outside the boundaries of routine, and sometimes legitimate, intellectual investigation.
      The AAAS displays great enthusiasm for liberal education. Yet its report may well set back reform by obscuring the depth and breadth of the challenge that Congress asked it to illuminate.
      36. According to Paragraph 1, what is the author’s attitude toward the AAAS’s report?
      [A] Critical
      [B] Appreciative
      [C] Contemptuous
      [D] Tolerant
      37. Influential figures in the Congress required that the AAAS report on how to
      [A] retain people’s interest in liberal education
      [B] define the government’s role in education
      [C] keep a leading position in liberal education
      [D] safeguard individuals’ rights to education
      38. According to Paragraph 3, the report suggests
      [A] an exclusive study of American history
      [B] a greater emphasis on theoretical subjects
      [C] the application of emerging technologies
      [D] funding for the study of foreign languages
      39. The author implies in Paragraph 5 that professors are
      [A] supportive of free markets
      [B] cautious about intellectual investigation
      [C] conservative about public policy
      [D] biased against classical liberal ideas
      40. Which of the following would be the best title for the text?
      [A] Ways to Grasp “The Heart of the Matter”
      [B] Illiberal Education and “The Heart of the Matter”
      [C] The AAAS’s Contribution to Liberal Education
      [D] Progressive Policy vs. Liberal Education
    Part B
      Directions:
      The following paragraphs are given in a wrong order. For Questions 41-45, you are required to reorganize these paragraphs into a coherent text by choosing from the list A-G and filling them into the numbered boxes. Paragraphs A and E have been correctly placed Mark your answers on the ANSWER SHEET (10 points)
      [A] Some archaeological sites have always been easily observable—for example, the Parthenon in Athens, Greece, the pyramids of Giza in Egypt; and the megaliths of Stonehenge in southern England. But these sites are exceptions to the norm. Most archaeological sites have been located by means of careful searching, while many others have been discovered by accident. Olduvai Gorge, an early hominid site in Tanzania, was found by a butterfly hunter who literally fell into its deep valley in 1911. Thousands of Aztec artifacts came to light during the digging of the Mexico City subway in the 1970s.
      [B]In another case, American archaeologists Rene Million and George Cowgill spent years systematically mapping the entire city of Teotihuacan in the Valley of Mexico near what is now Mexico City. At its peak around AD 600, this city was one of the largest human settlements in the world. The researchers mapped not only the city’s vast and ornate ceremonial areas, but also hundreds of simpler apartment complexes where common people lived.
      [C] How do archaeologists know where to find what they are looking for when there is nothing visible on the surface of the ground? Typically, they survey and sample (make test excavations on) large areas of terrain to determine where excavation will yield useful information. Surveys and test samples have also become important for understanding the larger landscapes that contain archaeological sites.
      [D] Surveys can cover a single large settlement or entire landscapes. In one case, many researchers working around the ancient Maya city of Copan, Honduras, have located hundreds of small rural villages and individual dwellings by using aerial photographs and by making surveys on foot. The resulting settlement maps show how the distribution and density of the rural population around the city changed dramatically between AD 500 and 850, when Copan collapsed.
      [E] To find their sites, archaeologists today rely heavily on systematic survey methods and a variety of high-technology tools and techniques. Airborne technologies, such as different types of radar and photographic equipment carried by airplanes or spacecraft, allow archaeologists to learn about what lies beneath the ground without digging. Aerial surveys locate general areas of interest or larger buried features, such as ancient buildings or fields.
      [F] Most archaeological sites, however, are discovered by archaeologists who have set out to look for them. Such searches can take years. British archaeologist Howard Carter knew that the tomb of the Egyptian pharaoh Tutankhamun existed from information found in other sites. Carter sifted through rubble in the Valley of the Kings for seven years before he located the tomb in 1922. In the late 1800s British archaeologist Sir Arthur Evan combed antique dealers’ stores in Athens, Greece. He was searching for tiny engraved seals attributed to the ancient Mycenaean culture that dominated Greece from the 1400s to 1200s BC. Evans’s interpretations of these engravings eventually led him to find the Minoan palace at Knossos (Knossós) on the island of Crete, in 1900.
      [G] Ground surveys allow archaeologists to pinpoint the places where digs will be successful. Most ground surveys involve a lot of walking, looking for surface clues such as small fragments of pottery. They often include a certain amount of digging to test for buried materials at selected points across a landscape. Archaeologists also may locate buried remains by using such technologies as ground radar, magnetic-field recording, and metal detectors. Archaeologists commonly use computers to map sites and the landscapes around sites. Two and three-dimensional maps are helpful tools in planning excavations, illustrating how sites look, and presenting the results of archaeological research.
      41 --- A --- 42. --- F ---43---G --- 44---D --- 45---B
      Part C
      Directions:
      Read the following text carefully and then translate the underlined segments into Chinese. Your translation should be written neatly on the ANSWER SHEET. (10 points)
    Music means different things to different people and sometimes even different things to the same person at different moments of his life. It might be poetic, philosophical, sensual, or mathematical, but in any case it must, in my view, have something to do with the soul of the human being. Hence it is metaphysical; but the means of expression is purely and exclusively physical: sound. I believe it is precisely this permanent coexistence of metaphysical message through physical means that is the strength of music. (46)It is also the reason why when we try to describe music with words, all we can do is articulate our reactions to it, and not grasp music itself.
    Beethoven’s importance in music has been principally defined by the revolutionary nature of his compositions. He freed music from hitherto prevailing conventions of harmony and structure. Sometimes I feel in his late works a will to break all signs of continuity. The music is abrupt and seemingly disconnected, as in the last piano sonata. In musical expression, he did not feel restrained by the weight of convention. (47)By all accounts he was a freethinking person, and a courageous one, and I find courage an essential quality for the understanding, let alone the performance, of his works.
    This courageous attitude in fact becomes a requirement for the performers of Beethoven’s music. His compositions demand the performer to show courage, for example in the use of dynamics. (48)Beethoven’s habit of increasing the volume with an intense crescendo and then abruptly following it with a sudden soft passage was only rarely used by composers before him.
    Beethoven was a deeply political man in the broadest sense of the word. He was not interested in daily politics, but concerned with questions of moral behavior and the larger questions of right and wrong affecting the entire society. (49)Especially significant was his view of freedom, which, for him, was associated with the rights and responsibilities of the individual: he advocated freedom of thought and of personal expression.
    Beethoven’s music tends to move from chaos to order as if order were an imperative of human existence. For him, order does not result from forgetting or ignoring the disorders that plague our existence; order is a necessary development, an improvement that may lead to the Greek ideal of spiritual elevation. It is not by chance that the Funeral March is not the last movement of the Eroica Symphony, but the second, so that suffering does not have the last word. (50)One could interpret much of the work of Beethoven by saying that suffering is inevitable, but the courage to fight it renders life worth living.
     Section Ⅲ Writing
      Part A
      51. Directions:
      Write a letter of about 100 words to the president of your university, suggesting how to improve students’ physical condition.
      You should include the details you think necessary.
      You should write neatly on the ANSWER SHEET.
      Do not sign your own name at the end of the letter. Use “Li Ming” instead.
      Do not write the address. (10 points)
     Part B
      52. Directions:
      Write an essay of 160-200 words based on the following drawing. In your essay, you should
      1) describe the drawing briefly,
      2) interpret its intended meaning, and
      3) give your comments.
      You should write neatly on the ANSWER SHEET(20 points)
    2013年 真题
    Section I Use of English
      Directions: Read the following text. Choose the best word(s) for each numbered blank and mark A, B, C or D on ANSWER SHEET 1. (10 points)
      People are, on the whole, poor at considering background information when making individual decisions. At first glance this might seem like a strength that _1 the ability to make judgments which are unbiased by 2 factors. But Dr Simonton speculated that an inability to consider the big 3 was leading decision-makers to be biased by the daily samples of information they were working with. 4, he theorized that a judge 5 of appearing too soft 6_crime might be more likely to send someone to prison 7_he had already sentenced five or six other defendants only to forced community service on that day.
      To 8__this idea, they turned their attention to the university-admissions process. In theory, the ____9
    of an applicant should not depend on the few others___10____ randomly for interview during the same day, but Dr Simonton suspected the truth was____11____.
      He studied the results of 9,323 MBA interviews 12 by 31 admissions officers. The interviewers had 13 applicants on a scale of one to five. This scale 14 numerous factors into consideration. The scores were 15 used in conjunction with an applicant’s score on the GMAT, a standardized exam which is 16_out of 800 points, to make a decision on whether to accept him or her.
      Dr Simonton found if the score of the previous candidate in a daily series of interviewees was 0.75 points or more higher than that of the one 17
    that, then the score for the next applicant would_18
    by an average of 0.075 points. This might sound small, but to_19_the effects of such a decrease a candidate would need 30 more GMAT points than would otherwise have been 20
    .
      1. A grants B submits C transmits D delivers
      2. A minor B external C crucial D objective
      3. A issue B vision C picture D moment
      4. A Above all B On average C In principle D For example
      5. A fond B fearful C capable D thoughtless
      6. A in B for C to D on
      7. A if B until C though D unless
      8. A. test B. emphasize C. share D. promote
      9. A. decision B. quality C. status D. success
      10. A. found B. studied C. chosen D. identified
      11. A. otherwise B. defensible C. replaceable D. exceptional
      12. A. inspired B. expressed C. conducted D. secured
      13. A. assigned B. rated C. matched D. arranged
      14. A. put B. got C. took D. gave
      15. A. instead B. then C. ever D. rather
      16. A. selected B. passed C. marked D. introduced
      17. A below B after C above D before
      18. A jump B float C fluctuate D drop
      19. A achieve B undo C maintain D disregard
      20. A necessary B possible C promising D helpful
    Section II Reading Comprehension
      Part A
      Directions: Read the following four texts. Answer the questions below each text by choosing A, B, C or D. Mark your answers on ANSWER SHEET 1. (40 points)
      Text 1
      In the 2006 film version of The Devil Wears Prada ,Miranda Priestly, played by Meryl Streep, scolds her unattractive assistant for imagining that high fashion doesn’t affect her, Priestly explains how the deep blue color of the assistant’s sweater descended over the years from fashion shows to departments stores and to the bargain bin in which the poor girl doubtless found her garment.
      This top-down conception of the fashion business couldn’t be more out of date or at odds with the feverish would described in Overdressed, Elizabeth Cline’s three-year indictment of “fast fashion”. In the last decade or so ,advances in technology have allowed mass-market labels such as Zara ,H&M, and Uniqlo to react to trends more quickly and anticipate demand more precisely. Quicker turnarounds mean less wasted inventory, more frequent release, and more profit. These labels encourage style-conscious consumers to see clothes as disposable-meant to last only a wash or two, although they don’t advertise that –and to renew their wardrobe every few weeks. By offering on-trend items at dirt-cheap prices, Cline argues, these brands have hijacked fashion cycles, shaking an industry long accustomed to a seasonal pace.
      The victims of this revolution , of course ,are not limited to designers. For H&M to offer a $5.95 knit miniskirt in all its 2,300-pius stores around the world, it must rely on low-wage overseas labor, order in volumes that strain natural resources, and use massive amounts of harmful chemicals.
      Overdressed is the fashion world’s answer to consumer-activist bestsellers like Michael Pollan’s The Omnivore’s Dilemma. “Mass-produced clothing ,like fast food, fills a hunger and need, yet is non-durable and wasteful,” Cline argues. Americans, she finds, buy roughly 20 billion garments a year – about 64 items per person – and no matter how much they give away, this excess leads to waste.
      Towards the end of Overdressed, Cline introduced her ideal, a Brooklyn woman named Sarah Kate Beaumont, who since 2008 has made all of her own clothes – and beautifully. But as Cline is the first to note, it took Beaumont decades to perfect her craft; her example can’t be knocked off.
      Though several fast-fashion companies have made efforts to curb their impact on labor and the environment – including H&M, with its green Conscious Collection line –Cline believes lasting change can only be effected by the customer. She exhibits the idealism common to many advocates of sustainability, be it in food or in energy. Vanity is a constant; people will only start shopping more sustainably when they can’t afford not to.
      21. Priestly criticizes her assistant for her
      [A] poor bargaining skill.
      [B] insensitivity to fashion.
      [C] obsession with high fashion.
      [D] lack of imagination.
      22. According to Cline, mass-market labels urge consumers to
      [A] combat unnecessary waste.
      [B] shut out the feverish fashion world.
      [C] resist the influence of advertisements.
      [D] shop for their garments more frequently.
      23. The word “indictment” (Line 3, Para.2) is closest in meaning to
      [A] accusation.
      [B] enthusiasm.
      [C] indifference.
      [D] tolerance.
      24. Which of the following can be inferred from the last paragraph?
      [A] Vanity has more often been found in idealists.
      [B] The fast-fashion industry ignores sustainability.
      [C] People are more interested in unaffordable garments.
      [D] Pricing is vital to environment-friendly purchasing.
      25. What is the subject of the text?
      [A] Satire on an extravagant lifestyle.
      [B] Challenge to a high-fashion myth.
      [C] Criticism of the fast-fashion industry.
      [D] Exposure of a mass-market secret.
      Text 2
      An old saying has it that half of all advertising budgets are wasted-the trouble is, no one knows which half. In the internet age, at least in theory, this fraction can be much reduced . By watching what people search for, click on and say online, companies can aim “behavioral” ads at those most likely to buy.
      In the past couple of weeks a quarrel has illustrated the value to advertisers of such fine-grained information: Should advertisers assume that people are happy to be tracked and sent behavioral ads? Or should they have explicit permission?
      In December 2010 America's Federal Trade Commission (FTC) proposed adding a "do not track "(DNT) option to internet browsers ,so that users could tell advertisers that they did not want to be followed .Microsoft's Internet Explorer and Apple's Safari both offer DNT ;Google's Chrome is due to do so this year. In February the FTC and Digital Advertising Alliance (DAA) agreed that the industry would get cracking on responding to DNT requests.
      On May 31st Microsoft Set off the row: It said that Internet Explorer 10,the version due to appear windows 8, would have DNT as a default.
      It is not yet clear how advertisers will respond. Getting a DNT signal does not oblige anyone to stop tracking, although some companies have promised to do so. Unable to tell whether someone really objects to behavioral ads or whether they are sticking with Microsoft’s default, some may ignore a DNT signal and press on anyway.
      Also unclear is why Microsoft has gone it alone. After all, it has an ad business too, which it says will comply with DNT requests, though it is still working out how. If it is trying to upset Google, which relies almost wholly on default will become the norm. DNT does not seem an obviously huge selling point for windows 8-though the firm has compared some of its other products favorably with Google's on that count before. Brendon Lynch, M
      Microsoft's chief privacy officer, blogged:"we believe consumers should have more control." Could it really be that simple?
      26. It is suggested in paragraph 1 that “behavioral” ads help advertisers to:
      [A] ease competition among themselves
      [B] lower their operational costs
      [C] avoid complaints from consumers
      [D] provide better online services
      27. “The industry” (Line 6,Para.3) refers to:
      [A] online advertisers
      [B] e-commerce conductors
      [C] digital information analysis
      [D] internet browser developers
      28. Bob Liodice holds that setting DNT as a default
      [A] many cut the number of junk ads
      [B] fails to affect the ad industry
      [C] will not benefit consumers
      [D] goes against human nature
      29. which of the following is true according to Paragraph.6?
      [A] DNT may not serve its intended purpose
      [B] Advertisers are willing to implement DNT
      [C] DNT is losing its popularity among consumers
      [D] Advertisers are obliged to offer behavioral ads
      30. The author's attitude towards what Brendon Lynch said in his blog is one of:
      [A] indulgence
      [B] understanding
      [C] appreciation
      [D] skepticism
      Text 3
      Up until a few decades ago, our visions of the future were largely - though by no means uniformly - glowingly positive. Science and technology would cure all the ills of humanity, leading to lives of fulfillment and opportunity for all.
      Now utopia has grown unfashionable, as we have gained a deeper appreciation of the range of threats facing us, from asteroid strike to epidemic flu and to climate change. You might even be tempted to assume that humanity has little future to look forward to.
      But such gloominess is misplaced. The fossil record shows that many species have endured for millions of years - so why shouldn't we? Take a broader look at our species' place in the universe, and it becomes clear that we have an excellent chance of surviving for tens, if not hundreds, of thousands of years . Look up Homo sapiens in the "Red List" of threatened species of the International Union for the Conversation of Nature (IUCN) ,and you will read: "Listed as Least Concern as the species is very widely distributed, adaptable, currently increasing, and there are no major threats resulting in an overall population decline."
      So what does our deep future hold? A growing number of researchers and organizations are now thinking seriously about that question. For example, the Long Now Foundation has its flagship project a medical clock that is designed to still be marking time thousands of years hence .
      Perhaps willfully , it may be easier to think about such lengthy timescales than about the more immediate future. The potential evolution of today's technology, and its social consequences, is dazzlingly complicated, and it's perhaps best left to science fiction writers and futurologists to explore the many possibilities we can envisage. That's one reason why we have launched Arc, a new publication dedicated to the near future.
      But take a longer view and there is a surprising amount that we can say with considerable assurance. As so often, the past holds the key to the future: we have now identified enough of the long-term patterns shaping the history of the planet, and our species, to make evidence-based forecasts about the situations in which our descendants will find themselves.
      This long perspective makes the pessimistic view of our prospects seem more likely to be a passing fad. To be sure, the future is not all rosy. But we are now knowledgeable enough to reduce many of the risks that threatened the existence of earlier humans, and to improve the lot of those to come.
      31. Our vision of the future used to be inspired by
      [A] our desire for lives of fulfillment
      [B] our faith in science and technology
      [C] our awareness of potential risks
      [D] our belief in equal opportunity
      32. The IUCN’s “Red List” suggest that human being are
      [A] a sustained species
      [B] a threaten to the environment
      [C] the world’s dominant power
      [D] a misplaced race
      33. Which of the following is true according to Paragraph 5?
      [A] Arc helps limit the scope of futurological studies.
      [B] Technology offers solutions to social problem.
      [C] The interest in science fiction is on the rise.
      [D] Our Immediate future is hard to conceive.
      34. To ensure the future of mankind, it is crucial to
      [A] explore our planet’s abundant resources
      [B] adopt an optimistic view of the world
      [C] draw on our experience from the past
      [D] curb our ambition to reshape history
      35. Which of the following would be the best title for the text?
      [A] Uncertainty about Our Future
      [B] Evolution of the Human Species
      [C] The Ever-bright Prospects of Mankind
      [D] Science, Technology and Humanity
      Text 4
      On a five to three vote, the Supreme Court knocked out much of Arizona's immigration law Monday-a modest policy victory for the Obama Administration. But on the more important matter of the Constitution, the decision was an 8-0 defeat for the federal government and the states.
      Anarizona. United States, the majority overturned three of the four contested provisions of Arizona's controversial plan to have state and local police enforce federal immigrations law. The Constitutional principles that Washington alone has the power to "establish a uniform Rule of Anturalization" and that federal laws precede state laws are noncontroversial. Arizona had attempted to fashion state police that ran to the existing federal ones.
      Justice Anthony Kennedy, joined by Chief Justice John Roberts and the Court's liberals, ruled that the state flew too close to the federal sun .On the overturned provisions the majority held the congress had deliberately "occupied the field " and Arizona had thus intruded on the federal's privileged powers
      However, the Justices said that Arizona police would be allowed to verify the legal status of people who come in contact with law enforcement. That’s because Congress has always envisioned joint federal-state immigration enforcement and explicitly encourages state officers to share information and cooperate with federal colleagues.
      Two of the three objecting Justice-Samuel Alito and Clarence Thomas-agreed with this Constitutional logic but disagreed about which Arizona rules conflicted with the federal statute. The only major objection came from Justice Antonin Scalia, who offered an even more robust defense of state privileges going back to the alien and Sedition Acts.
      The 8-0 objection to President Obama turns on what Justice Samuel Alito describes in his objection as “a shocking assertion of federal executive power”. The White House argued the Arizona’s laws conflicted with its enforcement priorities, even if state laws complied with federal statutes to the letter. In effect, the White House claimed that it could invalidate any otherwise legitimate state law that it disagrees with.
      Some powers do belong exclusively to the federal government, and control of citizenship and the borders is among them. But if Congress wanted to prevent states from using their own resources to check immigration status. It could. never did so. The administration was in essence asserting that because it didn't want to carry out Congress's immigration wishes, no state should be allowed to do so either. Every Justice rightly rejected this remarkable claim.
      36. Three provisions of Arizona’s plan were overturned because they
      [A] deprived the federal police of Constitutional powers.
      [B] disturbed the power balance between different states.
      [C] overstepped the authority of federal immigration law.
      [D] contradicted both the federal and state policies.
      37. On which of the following did the Justices agree, according to Paragraph4?
      [A] Federal officers’ duty to withhold immigrants’information.
      [B] States’ independence from federal immigration law.
      [C] States’ legitimate role in immigration enforcement.
      [D] Congress’s intervention in immigration enforcement.
      38. It can be inferred from Paragraph 5 that the Alien and Sedition Acts
      [A] violated the Constitution.
      [B] undermined the states’ interests.
      [C] supported the federal statute.
      [D] stood in favor of the states.
      39. The White House claims that its power of enforcement
      [A] Outweighs that held by the states.
      [B] is dependent on the states’ support.
      [C] is established by federal statutes.
      [D] rarely goes against state laws.
      40. What can be learned from the last paragraph?
      [A] Immigration issues are usually decided by Congress.
      [B] Justices intended to check the power of the Administration.
      [C] Justices wanted to strengthen its coordination with Congress.
      [D] The Administration is dominant over immigration issues.
    Part B
      Directions:
      In the following text, some sentences have been removed. For Questions 41-45, choose the most suitable one from the list A-G to fit into each of the numbered blanks. There are two extra choices, which do not fit in any of the blanks. Mark your answers on ANSWER SHEET1.(10 points)
      The social sciences are flourishing. As of 2005, there were almost half a million professional social scientists from all fields in the world, working both inside and outside academia. According to the World Social Science Report 2010, the number of social-science students worldwide has swollen by about 11% every year since 2000.
      Yet this enormous resource in not contributing enough to today’s global challenges including climate change, security, sustainable development and health.(41)______Humanity has the necessary agro-technological tools to eradicate hunger , from genetically engineered crops to artificial fertilizers . Here, too, the problems are social: the organization and distribution of food, wealth and prosperity.
      (42)____This is a shame—the community should be grasping the opportunity to raise its influence in the real world. To paraphrase the great social scientist Joseph Schumpeter: there is no radical innovation without creative destruction.
      Today ,the social sciences are largely focused on disciplinary problems and internal scholarly debates, rather than on topics with external impact.
      Analyses reveal that the number of papers including the keywords “environmental changed” or “climate change” have increased rapidly since 2004,(43)____
      When social scientists do tackle practical issues ,their scope is often local: Belgium is interested mainly in the effects of poverty on Belgium for example .And whether the community’s work contributes much to an overall accumulation of knowledge is doubtful.
      The problem is not necessarily the amount of available funding (44)____this is an adequate amount so long as it is aimed in the right direction. Social scientists who complain about a lack of funding should not expect more in today’s economic climate.
      The trick is to direct these funds better. The European Union Framework funding programs have long had a category specifically targeted at social scientists. This year, it was proposed that system be changed: Horizon 2020,a new program to be enacted in 2014,would not have such a category ,This has resulted in protests from social scientists. But the intention is not to neglect social science ; rather ,the complete opposite.(45)____That should create more collaborative endeavors and help to develop projects aimed directly at solving global problems.
      [A] It could be that we are evolving two communities of social
      scientists: one that is discipline-oriented and publishing in highly
      specialized journals, and one that is problem-oriented and publishing
      elsewhere, such as policy briefs.
      [B] However, the numbers are still small: in 2010,about 1,600 of the
      100,000 social-sciences papers published globally included one of these
      Keywords.
      [C] the idea is to force social to integrate their work with other categories, including health and demographic change food security, marine research and the bio-economy, clear, efficient energy; and inclusive, innovative and secure societies.
      [D] the solution is to change the mindset of the academic community, and what it considers to be its main goal. Global challenges and social innovation ought to receive much more attention from scientists, especially the young ones.
      [E] These issues all have root causes in human behavior. All require behavioral change and social innovations, as well as technological development. Stemming climate change, for example, is as much about changing consumption patterns and promoting tax acceptance as it is about developing clean energy.
      [F] Despite these factors , many social scientists seem reluctant to tackle such problems . And in Europe , some are up in arms over a proposal to drop a specific funding category for social-science research and to integrate it within cross-cutting topics of sustainable development .
      [G] During the late 1990s , national spending on social sciences and the humanities as a percentage of all research and development funds-including government, higher education, non-profit and corporate -varied from around 4% to 25%; in most European nations , it is about 15%.
    Part C
      Directions: Read the following text carefully and then translate the underlined segments into Chinese. Your translation should be written clearly on ANSWER SHEET 2. (10 points)
      It is speculated that gardens arise from a basic need in the individuals who made them: the need for creative expression. There is no doubt that gardens evidence an impossible urge to create, express, fashion, and beautify and that self-expression is a basic human urge; (46) Yet when one looks at the photographs of the garden created by the homeless, it strikes one that , for all their diversity of styles, these gardens speak of various other fundamental urges, beyond that of decoration and creative expression.
      One of these urges had to do with creating a state of peace in the midst of turbulence, a “still point of the turning world,” to borrow a phrase from T. S. Eliot. (47)A sacred place of peace, however crude it may be, is a distinctly human need, as opposed to shelter, which is a distinctly animal need. This distinction is so much so that where the latter is lacking, as it is for these unlikely gardens, the former becomes all the more urgent. Composure is a state of mind made possible by the structuring of one’s relation to one’s environment. (48) The gardens of the homeless which are in effect homeless gardens introduce from into an urban environment where it either didn’t exist or was not discernible as such. In so doing they give composure to a segment of the inarticulate environment in which they take their stand.
      Another urge or need that these gardens appear to respond to, or to arise from is so intrinsic that we are barely ever conscious of its abiding claims on us. When we are deprived of green, of plants, of trees, (49) most of us give into a demoralization of spirit which we usually blame on some psychological conditions, until one day we find ourselves in garden and feel the expression vanish as if by magic. In most of the homeless gardens of New York City the actual cultivation of plants is unfeasible, yet even so the compositions often seem to represent attempts to call arrangement of materials, an institution of colors, small pool of water, and a frequent presence of petals or leaves as well as of stuffed animals. On display here are various fantasy elements whose reference, at some basic level, seems to be the natural world. (50)It is this implicit or explicit reference to nature that fully justifies the use of word garden though in a “liberated” sense, to describe these synthetic constructions. In them we can see basophilic- a yearning for contact with nonhuman life-assuming uncanny representational forms.
      46. yet when one looks at the photographs of the gardens created by the homeless, it strikes one that, for all their diversity of styles, these gardens speak of various other fundamental urges beyond that of decoration and creative expression.
      47. A sacred place of peace, however, crude it may be, is a distinctly human need, as opposed to shelter which is a distinctly animal need.
      48. The gardens of the homeless which are in effect homeless garden introduce from in to an urban environment where it either didn’t exist or was not discernible as such
      49 . Most of us give in to a demoralization of spirit which we usually blame on some psychological conditions until one day we find ourselves in a garden and feel the oppression vanish as if by magic
      50. It is this implicit or explicit reference to nature that fully justifies the use of the word garden, though in a “liberated” sense, to describe these synthetic constructions.
    Section III Writing
      Part A
      51.Directions:
      Write an e-mail of about 100 words to a foreign teacher in your college, inviting him/her to be a judge for the upcoming English speech contest.
      You should include the details you think necessary.
      You should write neatly on the ANSWER SHEET.
      Do not sign your own name at the end of the e-mail. Use “Li Ming “instead.
      Do not write the address.(10 points)
      Part B
      52.Directions:
      Write an essay of 160-200 words based on the following drawing .In your essay, you should
      1) describe the drawing briefly.
      2) interpret its intended meaning ,and
      3) give your comments.
      You should write neatly on the ANSWER SHEET.(20points)
    2012年全国硕士研究生入学统一考试英语(一)
    Section I Use of English

Directions:
Read the following text. Choose the best word(s) for each numbered blank and mark A, B, C or D on ANSWER SHEET 1. (10 points)
Read the following text. Choose the best word(s) for each numbered blank and mark A, B, C or D on ANSWER SHEET 1. (10 points)
The ethical judgments of the Supreme Court justices have become an important issue recently. The court cannot 1 its legitimacy as guardian of the rule of law 2 justices behave like politicians. Yet, in several instances, justices acted in ways that 3 the court’s reputation for being independent and impartial.
Justice Antonin Scalia, for example, appeared at political events. That kind of activity makes it less likely that the court’s decisions will be 4 as impartial judgments. Part of the problem is that the justices are not _5_by an ethics code. At the very least, the court should make itself _6_to the code of conduct that _7_to the rest of the federal judiciary.
This and other similar cases _8_the question of whether there is still a _9_between the court and politics.
The framers of the Constitution envisioned law _10_having authority apart from politics. They gave justices permanent positions _11_they would be free to 12 those in power and have no need to 13 political support. Our legal system was designed to set law apart from politics precisely because they are so closely 14.
Constitutional law is political because it results from choices rooted in fundamental social 15 like liberty and property. When the court deals with social policy decisions, the law it 16 is inescapably political-which is why decisions split along ideological lines are so easily 17 as unjust.
The justices must 18 doubts about the court’s legitimacy by making themselves 19 to the code of conduct. That would make rulings more likely to be seen as separate from politics and, 20, convincing as law.

  1. [A]emphasize [B]maintain [C]modify [D] recognize
  2. [A]when [B]lest [C]before [D] unless
  3. [A]restored [B]weakened [C]established [D] eliminated
  4. [A]challenged [B]compromised [C]suspected [D] accepted
  5. [A]advanced [B]caught [C]bound [D]founded
  6. [A]resistant [B]subject [C]immune [D]prone
  7. [A]resorts [B]sticks [C]loads [D]applies
  8. [A]evade [B]raise [C]deny [D]settle
  9. [A]line [B]barrier [C]similarity [D]conflict
  10. [A]by [B]as [C]though [D]towards
  11. [A]so [B]since [C]provided [D]though
  12. [A]serve [B]satisfy [C]upset [D]replace
  13. [A]confirm [B]express [C]cultivate [D]offer
  14. [A]guarded [B]followed [C]studied [D]tied
  15. [A]concepts [B]theories [C]divisions [D]conceptions
  16. [A]excludes [B]questions [C]shapes [D]controls
  17. [A]dismissed [B]released [C]ranked [D]distorted
  18. [A]suppress [B]exploit [C]address [D]ignore
  19. [A]accessible [B]amiable [C]agreeable [D]accountable
  20. [A]by all mesns [B]atall costs [C]in a word [D]as a result

Section II Reading Comprehension

Part A
Directions:
Read the following four texts. Answer the questions below each text by choosing A, B, C or D. Mark your answers on ANSWER SHEET 1. (40 points)
Text 1
Come on –Everybody’s doing it. That whispered message, half invitation and half forcing, is what most of us think of when we hear the words peer pressure. It usually leads to no good-drinking, drugs and casual sex. But in her new book Join the Club, Tina Rosenberg contends that peer pressure can also be a positive force through what she calls the social cure, in which organizations and officials use the power of group dynamics to help individuals improve their lives and possibly the word.
Rosenberg, the recipient of a Pulitzer Prize, offers a host of example of the social cure in action: In South Carolina, a state-sponsored antismoking program called Rage Against the Haze sets out to make cigarettes uncool. In South Africa, an HIV-prevention initiative known as LoveLife recruits young people to promote safe sex among their peers.
The idea seems promising,and Rosenberg is a perceptive observer. Her critique of the lameness of many pubic-health campaigns is spot-on: they fail to mobilize peer pressure for healthy habits, and they demonstrate a seriously flawed understanding of psychology.” Dare to be different, please don’t smoke!” pleads one billboard campaign aimed at reducing smoking among teenagers-teenagers, who desire nothing more than fitting in. Rosenberg argues convincingly that public-health advocates ought to take a page from advertisers, so skilled at applying peer pressure.
But on the general effectiveness of the social cure, Rosenberg is less persuasive. Join the Club is filled with too much irrelevant detail and not enough exploration of the social and biological factors that make peer pressure so powerful. The most glaring flaw of the social cure as it’s presented here is that it doesn’t work very well for very long. Rage Against the Haze failed once state funding was cut. Evidence that the LoveLife program produces lasting changes is limited and mixed.
There’s no doubt that our peer groups exert enormous influence on our behavior. An emerging body of research shows that positive health habits-as well as negative ones-spread through networks of friends via social communication. This is a subtle form of peer pressure: we unconsciously imitate the behavior we see every day.
Far less certain, however, is how successfully experts and bureaucrats can select our peer groups and steer their activities in virtuous directions. It’s like the teacher who breaks up the troublemakers in the back row by pairing them with better-behaved classmates. The tactic never really works. And that’s the problem with a social cure engineered from the outside: in the real world, as in school, we insist on choosing our own friends.

  1. According to the first paragraph, peer pressure often emerges as
    [A] a supplement to the social cure
    [B] a stimulus to group dynamics
    [C] an obstacle to school progress
    [D] a cause of undesirable behaviors

  2. Rosenberg holds that public advocates should
    [A] recruit professional advertisers
    [B] learn from advertisers’ experience
    [C] stay away from commercial advertisers
    [D] recognize the limitations of advertisements

  3. In the author’s view, Rosenberg’s book fails to
    [A] adequately probe social and biological factors
    [B] effectively evade the flaws of the social cure
    [C] illustrate the functions of state funding
    [D]produce a long-lasting social effect

  4. Paragraph 5shows that our imitation of behaviors
    [A] is harmful to our networks of friends
    [B] will mislead behavioral studies
    [C] occurs without our realizing it
    [D] can produce negative health habits

  5. The author suggests in the last paragraph that the effect of peer pressure is
    [A] harmful
    [B] desirable
    [C] profound
    [D] questionable

Text 2
A deal is a deal-except, apparently ,when Entergy is involved. The company, a major energy supplier in New England, provoked justified outrage in Vermont last week when it announced it was reneging on a longstanding commitment to abide by the strict nuclear regulations.
Instead, the company has done precisely what it had long promised it would not challenge the constitutionality of Vermont’s rules in the federal court, as part of a desperate effort to keep its Vermont Yankee nuclear power plant running. It’s a stunning move.
The conflict has been surfacing since 2002, when the corporation bought Vermont’s only nuclear power plant, an aging reactor in Vernon. As a condition of receiving state approval for the sale, the company agreed to seek permission from state regulators to operate past 2012. In 2006, the state went a step further, requiring that any extension of the plant’s license be subject to Vermont legislature’s approval. Then, too, the company went along.
Either Entergy never really intended to live by those commitments, or it simply didn’t foresee what would happen next. A string of accidents, including the partial collapse of a cooling tower in 207 and the discovery of an underground pipe system leakage, raised serious questions about both Vermont Yankee’s safety and Entergy’s management– especially after the company made misleading statements about the pipe. Enraged by Entergy’s behavior, the Vermont Senate voted 26 to 4 last year against allowing an extension.
Now the company is suddenly claiming that the 2002 agreement is invalid because of the 2006 legislation, and that only the federal government has regulatory power over nuclear issues. The legal issues in the case are obscure: whereas the Supreme Court has ruled that states do have some regulatory authority over nuclear power, legal scholars say that Vermont case will offer a precedent-setting test of how far those powers extend. Certainly, there are valid concerns about the patchwork regulations that could result if every state sets its own rules. But had Entergy kept its word, that debate would be beside the point.
The company seems to have concluded that its reputation in Vermont is already so damaged that it has noting left to lose by going to war with the state. But there should be consequences. Permission to run a nuclear plant is a poblic trust. Entergy runs 11 other reactors in the United States, including Pilgrim Nuclear station in Plymouth. Pledging to run Pilgrim safely, the company has applied for federal permission to keep it open for another 20 years. But as the Nuclear Regulatory Commission (NRC) reviews the company’s application, it should keep it mind what promises from Entergy are worth.

  1. The phrase “reneging on”(Line 3.para.1) is closest in meaning to
    [A] condemning.
    [B] reaffirming.
    [C] dishonoring.
    [D] securing.

  2. By entering into the 2002 agreement, Entergy intended to
    [A] obtain protection from Vermont regulators.
    [B] seek favor from the federal legislature.
    [C] acquire an extension of its business license .
    [D] get permission to purchase a power plant.

  3. According to Paragraph 4, Entergy seems to have problems with its
    [A] managerial practices.
    [B] technical innovativeness.
    [C] financial goals.
    [D] business vision

  4. In the author’s view, the Vermont case will test
    [A] Entergy’s capacity to fulfill all its promises.
    [B] the mature of states’ patchwork regulations.
    [C] the federal authority over nuclear issues .
    [D] the limits of states’ power over nuclear issues.

  5. It can be inferred from the last paragraph that
    [A] Entergy’s business elsewhere might be affected.
    [B] the authority of the NRC will be defied.
    [C] Entergy will withdraw its Plymouth application.
    [D] Vermont’s reputation might be damaged.

Text 3
In the idealized version of how science is done, facts about the world are waiting to be observed and collected by objective researchers who use the scientific method to carry out their work. But in the everyday practice of science, discovery frequently follows an ambiguous and complicated route. We aim to be objective, but we cannot escape the context of our unique life experience. Prior knowledge and interest influence what we experience, what we think our experiences mean, and the subsequent actions we take. Opportunities for misinterpretation, error, and self-deception abound.
Consequently, discovery claims should be thought of as protoscience. Similar to newly staked mining claims, they are full of potential. But it takes collective scrutiny and acceptance to transform a discovery claim into a mature discovery. This is the credibility process, through which the individual researcher’s me, here, now becomes the community’s anyone, anywhere, anytime. Objective knowledge is the goal, not the starting point.
Once a discovery claim becomes public, the discoverer receives intellectual credit. But, unlike with mining claims, the community takes control of what happens next. Within the complex social structure of the scientific community, researchers make discoveries; editors and reviewers act as gatekeepers by controlling the publication process; other scientists use the new finding to suit their own purposes; and finally, the public (including other scientists) receives the new discovery and possibly accompanying technology. As a discovery claim works it through the community, the interaction and confrontation between shared and competing beliefs about the science and the technology involved transforms an individual’s discovery claim into the community’s credible discovery.
Two paradoxes exist throughout this credibility process. First, scientific work tends to focus on some aspect of prevailing Knowledge that is viewed as incomplete or incorrect. Little reward accompanies duplication and confirmation of what is already known and believed. The goal is new-search, not re-search. Not surprisingly, newly published discovery claims and credible discoveries that appear to be important and convincing will always be open to challenge and potential modification or refutation by future researchers. Second, novelty itself frequently provokes disbelief. Nobel Laureate and physiologist Albert Azent-Gyorgyi once described discovery as “seeing what everybody has seen and thinking what nobody has thought.” But thinking what nobody else has thought and telling others what they have missed may not change their views. Sometimes years are required for truly novel discovery claims to be accepted and appreciated.
In the end, credibility “happens” to a discovery claim – a process that corresponds to what philosopher Annette Baier has described as the commons of the mind. “We reason together, challenge, revise, and complete each other’s reasoning and each other’s conceptions of reason.”

  1. According to the first paragraph, the process of discovery is characterized by its
    [A] uncertainty and complexity.
    [B] misconception and deceptiveness.
    [C] logicality and objectivity.
    [D] systematicness and regularity.

  2. It can be inferred from Paragraph 2 that credibility process requires
    [A] strict inspection.
    [B]shared efforts.
    [C] individual wisdom.
    [D]persistent innovation.

33.Paragraph 3 shows that a discovery claim becomes credible after it
[A] has attracted the attention of the general public.
[B]has been examined by the scientific community.
[C] has received recognition from editors and reviewers.
[D]has been frequently quoted by peer scientists.

  1. Albert Szent-Györgyi would most likely agree that
    [A] scientific claims will survive challenges.
    [B]discoveries today inspire future research.
    [C] efforts to make discoveries are justified.
    [D]scientific work calls for a critical mind.

35.Which of the following would be the best title of the test?
[A] Novelty as an Engine of Scientific Development.
[B]Collective Scrutiny in Scientific Discovery.
[C] Evolution of Credibility in Doing Science.
[D]Challenge to Credibility at the Gate to Science.

Text 4
If the trade unionist Jimmy Hoffa were alive today, he would probably represent civil servant. When Hoffa’s Teamsters were in their prime in 1960, only one in ten American government workers belonged to a union; now 36% do. In 2009 the number of unionists in America’s public sector passed that of their fellow members in the private sector. In Britain, more than half of public-sector workers but only about 15% of private-sector ones are unionized.
There are three reasons for the public-sector unions’ thriving. First, they can shut things down without suffering much in the way of consequences. Second, they are mostly bright and well-educated. A quarter of America’s public-sector workers have a university degree. Third, they now dominate left-of-centre politics. Some of their ties go back a long way. Britain’s Labor Party, as its name implies, has long been associated with trade unionism. Its current leader, Ed Miliband, owes his position to votes from public-sector unions.
At the state level their influence can be even more fearsome. Mark Baldassare of the Public Policy Institute of California points out that much of the state’s budget is patrolled by unions. The teachers’ unions keep an eye on schools, the CCPOA on prisons and a variety of labor groups on health care.
In many rich countries average wages in the state sector are higher than in the private one. But the real gains come in benefits and work practices. Politicians have repeatedly “backloaded” public-sector pay deals, keeping the pay increases modest but adding to holidays and especially pensions that are already generous.
Reform has been vigorously opposed, perhaps most egregiously in education, where charter schools, academies and merit pay all faced drawn-out battles. Even though there is plenty of evidence that the quality of the teachers is the most important variable, teachers’ unions have fought against getting rid of bad ones and promoting good ones.
As the cost to everyone else has become clearer, politicians have begun to clamp down. In Wisconsin the unions have rallied thousands of supporters against Scott Walker, the hardline Republican governor. But many within the public sector suffer under the current system, too.
John Donahue at Harvard’s Kennedy School points out that the norms of culture in Western civil services suit those who want to stay put but is bad for high achievers. The only American public-sector workers who earn well above $250,000 a year are university sports coaches and the president of the United States. Bankers’ fat pay packets have attracted much criticism, but a public-sector system that does not reward high achievers may be a much bigger problem for America.

  1. It can be learned from the first paragraph that
    [A] Teamsters still have a large body of members.
    [B] Jimmy Hoffa used to work as a civil servant.
    [C] unions have enlarged their public-sector membership.
    [D]the government has improved its relationship with unionists.

  2. Which of the following is true of Paragraph 2?
    [A] Public-sector unions are prudent in taking actions.
    [B] Education is required for public-sector union membership.
    [C] Labor Party has long been fighting against public-sector unions.
    [D]Public-sector unions seldom get in trouble for their actions.

  3. It can be learned from Paragraph 4 that the income in the state sector is
    [A] illegally secured.
    [B] indirectly augmented.
    [C] excessively increased.
    [D]fairly adjusted.

  4. The example of the unions in Wisconsin shows that unions
    [A]often run against the current political system.
    [B]can change people’s political attitudes.
    [C]may be a barrier to public-sector reforms.
    [D]are dominant in the government.

  5. John Donahue’s attitude towards the public-sector system is one of
    [A]disapproval.
    [B]appreciation.
    [C]tolerance.
    [D]indifference.

Part B
Directions:
In the following text, some sentences have been removed. For Questions 41-45, choose the most suitable one from the list A-G to fit into each of the numbered blanks. There are two extra choices, which do not fit in any of the blanks. Mark your answers on ANSWER SHEET1.(10 points)
Think of those fleeting moments when you look out of an aeroplane window and realise that you are flying, higher than a bird. Now think of your laptop, thinner than a brown-paper envelope, or your cellphone in the palm of your hand. Take a moment or two to wonder at those marvels. You are the lucky inheritor of a dream come true.
The second half of the 20th century saw a collection of geniuses, warriors, entrepreneurs and visionaries labour to create a fabulous machine that could function as a typewriter and printing press, studio and theatre, paintbrush and gallery, piano and radio, the mail as well as the mail carrier. (41)
The networked computer is an amazing device, the first media machine that serves as the mode of production, means of distribution, site of reception, and place of praise and critique. The computer is the 21st century's culture machine.
But for all the reasons there are to celebrate the computer, we must also tread with caution. (42)I call it a secret war for two reasons. First, most people do not realise that there are strong commercial agendas at work to keep them in passive consumption mode. Second, the majority of people who use networked computers to upload are not even aware of the significance of what they are doing.
All animals download, but only a few upload. Beavers build dams and birds make nests. Yet for the most part, the animal kingdom moves through the world downloading. Humans are unique in their capacity to not only make tools but then turn around and use them to create superfluous material goods - paintings, sculpture and architecture - and superfluous experiences - music, literature, religion and philosophy. (43)
For all the possibilities of our new culture machines, most people are still stuck in download mode. Even after the advent of widespread social media, a pyramid of production remains, with a small number of people uploading material, a slightly larger group commenting on or modifying that content, and a huge percentage remaining content to just consume. (44)
Television is a one-way tap flowing into our homes. The hardest task that television asks of anyone is to turn the power off after he has turned it on.
(45)
What counts as meaningful uploading? My definition revolves around the concept of "stickiness" - creations and experiences to which others adhere.

[A] Of course, it is precisely these superfluous things that define human culture and ultimately what it is to be human. Downloading and consuming culture requires great skills, but failing to move beyond downloading is to strip oneself of a defining constituent of humanity.
[B] Applications like tumblr.com, which allow users to combine pictures, words and other media in creative ways and then share them, have the potential to add stickiness by amusing, entertaining and enlightening others.
[C] Not only did they develop such a device but by the turn of the millennium they had also managed to embed it in a worldwide system accessed by billions of people every day.
[D] This is because the networked computer has sparked a secret war between downloading and uploading - between passive consumption and active creation - whose outcome will shape our collective future in ways we can only begin to imagine.
[E] The challenge the computer mounts to television thus bears little similarity to one format being replaced by another in the manner of record players being replaced by CD players.
[F] One reason for the persistence of this pyramid of production is that for the past half-century, much of the world's media culture has been defined by a single medium - television - and television is defined by downloading.
[G]The networked computer offers the first chance in 50 years to reverse the flow, to encourage thoughtful downloading and, even more importantly, meaningful uploading.

Part C
Directions:
Read the following text carefully and then translate the underlined segments into Chinese. Your translation should be written clearly on ANSWER SHEET 2. (10 points)
Since the days of Aristotle, a search for universal principles has characterized the scientific enterprise. In some ways, this quest for commonalities defines science. Newton’s laws of motion and Darwinian evolution each bind a host of different phenomena into a single explicatory frame work.
(46)In physics, one approach takes this impulse for unification to its extreme, and seeks a theory of everything—a single generative equation for all we see.It is becoming less clear, however, that such a theory would be a simplification, given the dimensions and universes that it might entail, nonetheless, unification of sorts remains a major goal.
This tendency in the natural sciences has long been evident in the social sciences too. (47)Here, Darwinism seems to offer justification for it all humans share common origins it seems reasonable to suppose that cultural diversity could also be traced to more constrained beginnings. Just as the bewildering variety of human courtship rituals might all be considered forms of sexual selection, perhaps the world’s languages, music, social and religious customs and even history are governed by universal features. (48)To filter out what is unique from what is shared might enable us to understand how complex cultural behavior arose and what guides it in evolutionary or cognitive terms.
That, at least, is the hope. But a comparative study of linguistic traits published online today supplies a reality check. Russell Gray at the University of Auckland and his colleagues consider the evolution of grammars in the light of two previous attempts to find universality in language.
The most famous of these efforts was initiated by Noam Chomsky, who suggested that humans are born with an innate language—acquisition capacity that dictates a universal grammar. A few generative rules are then sufficient to unfold the entire fundamental structure of a language, which is why children can learn it so quickly.
(49)The second, by Joshua Greenberg, takes a more empirical approach to universality identifying traits (particularly in word order) shared by many language which are considered to represent biases that result from cognitive constraints
Gray and his colleagues have put them to the test by examining four family trees that between them represent more than 2,000 languages.(50)Chomsky’s grammar should show patterns of language change that are independent of the family tree or the pathway tracked through it. Whereas Greenbergian universality predicts strong co-dependencies between particular types of word-order relations. Neither of these patterns is borne out by the analysis, suggesting that the structures of the languages are lire age-specific and not governed by universals

Section III Writing

Part A

  1. Directions:
    Some internationals students are coming to your university. Write them an email in the name of the Students’ Union to
  1.  extend your welcome and
    
  2.  provide some suggestions for their campus life here.
    

You should write about 100 words on ANSWER SHEET2.Do not sign your name at the end of the letter. Use “Li Ming” instead.
Do not write the address(10 points)

Part B

  1. Directions: write an essay of 160-200 words based on the following drawing. In your essay you should
  1. describe the drawing briefly
  2. explain its intended meaning, and
  3. give your comments
    You should write neatly on ANSWER SHEET2.(20 points)
    2011年考研英语一真题及答案
    Section I Use of English
    Directions:
    Ancient Greek philosopher Aristotle viewed laughter as “a bodily exercise precious to health.” But _____some claims to the contrary, laughing probably has little influence on physical filness Laughter does _____short-term changes in the function of the heart and its blood vessels, ____ heart rate and oxygen consumption But because hard laughter is difficult to ____, a good laugh is unlikely to have _____ benefits the way, say, walking or jogging does.
    ____, instead of straining muscles to build them, as exercise does, laughter apparently accomplishes the ____, studies dating back to the 1930’s indicate that laughter. muscles,
    Such bodily reaction might conceivably help____the effects of psychological stress.Anyway,the act of laughing probably does produce other types of ______feedback,that improve an individual’s emotional state. ______one classical theory of emotion,our feelings are partially rooted _______ physical reactions. It was argued at the end of the 19th century that humans do not cry ______they are sad but they become sad when te tears begin to flow.
    Although sadness also _______ tears,evidence suggests that emotions can flow _____ muscular responses.In an experiment published in 1988,social psychologist Fritz.
    1.[A]among [B]except [C]despite [D]like
    2.[A]reflect [B]demand [C]indicate [D]produce
    3.[A]stabilizing [B]boosting [C]impairing [D]determining
    4.[A]transmit [B]sustain [C]evaluate [D]observe
    5.[A]measurable [B]manageable [C]affordable [D]renewable
    6.[A]In turn [B]In fact [C]In addition [D]In brief
    7.[A]opposite [B]impossible [C]average [D]expected
    8.[A]hardens [B]weakens [C]tightens [D]relaxes
    9.[A]aggravate [B]generate [C]moderate [D]enhance
    10.[A]physical [B]mentl [C]subconscious [D]internal
    11.[A]Except for [B]According to [C]Due to [D]As for
    12.[A]with [B]on [C]in [D]at
    13.[A]unless [B]until C]if [D]because
    14.[A]exhausts [B]follows [C]precedes [D]suppresses
    15.[A]into [B]from [C]towards [D]beyond
    16.[A]fetch [B]bite [C]pick [D]hold
    17.[A]disappointed [B]excited [C]joyful [D]indifferent
    18.[A]adapted [B]catered [C]turned [D]reacted
    19.[A]suggesting [B]requiring [C]mentioning [D]supposing
    20.[A]Eventually [B]Consequently [C]Similarly [D]Conversely
    Section II Reading Comprehension
    Part A
    Directions:
    Read the following four texts. Answer the questions below each text by choosing [A], [B], [C] or [D]. Mark your answers on ANSWER SHEET 1. (40 points)
    Text 1
    The decision of the New York Philharmonic to hire Alan Gilbert as its next music director has been the talk of the classical-music world ever since the sudden announcement of his appointment in 2009. For the most part, the response has been favorable, to say the least. “Hooray! At last!” wrote Anthony Tommasini, a sober-sided classical-music critic.
    One of the reasons why the appointment came as such a surprise, however, is that Gilbert is comparatively little known. Even Tommasini, who had advocated Gilbert’s appointment in the Times, calls him “an unpretentious musician with no air of the formidable conductor about him.” As a description of the next music director of an orchestra that has hitherto been led by musicians like Gustav Mahler and Pierre Boulez, that seems likely to have struck at least some Times readers as faint praise.
    For my part, I have no idea whether Gilbert is a great conductor or even a good one. To be sure, he performs an impressive variety of interesting compositions, but it is not necessary for me to visit Avery Fisher Hall, or anywhere else, to hear interesting orchestral music. All I have to do is to go to my CD shelf, or boot up my computer and download still more recorded music from iTunes.
    Devoted concertgoers who reply that recordings are no substitute for live performance are missing the point. For the time, attention, and money of the art-loving public, classical instrumentalists must compete not only with opera houses, dance troupes, theater companies, and museums, but also with the recorded performances of the great classical musicians of the 20th century. There recordings are cheap, available everywhere, and very often much higher in artistic quality than today’s live performances; moreover, they can be “consumed” at a time and place of the listener’s choosing. The widespread availability of such recordings has thus brought about a crisis in the institution of the traditional classical concert.
    One possible response is for classical performers to program attractive new music that is not yet available on record. Gilbert’s own interest in new music has been widely noted: Alex Ross, a classical-music critic, has described him as a man who is capable of turning the Philharmonic into “a markedly different, more vibrant organization.” But what will be the nature of that difference? Merely expanding the orchestra’s repertoire will not be enough. If Gilbert and the Philharmonic are to succeed, they must first change the relationship between America’s oldest orchestra and the new audience it hops to attract.
  1. We learn from Para.1 that Gilbert’s appointment has
    [A]incurred criticism. [B]raised suspicion. [C]received acclaim. [D]aroused curiosity.
  2. Tommasini regards Gilbert as an artist who is
    [A]influential. [B]modest. [C]respectable. [D]talented.
  3. The author believes that the devoted concertgoers
    [A]ignore the expenses of live performances. [B]reject most kinds of recorded performances.
    [C]exaggerate the variety of live performances. [D]overestimate the value of live performances.
  4. According to the text, which of the following is true of recordings?
    [A]They are often inferior to live concerts in quality.
    [B]They are easily accessible to the general public.
    [C]They help improve the quality of music. [D]They have only covered masterpieces.
  5. Regarding Gilbert’s role in revitalizing the Philharmonic, the author feels
    [A]doubtful. [B]enthusiastic. [C]confident. [D]puzzled.
    Text 2
    When Liam McGee departed as president of Bank of America in August, his explanation was surprisingly straight up. Rather than cloaking his exit in the usual vague excuses, he came right out and said he was leaving “to pursue my goal of running a company.” Broadcasting his ambition was “very much my decision,” McGee says. Within two weeks, he was talking for the first time with the board of Hartford Financial Services Group, which named him CEO and chairman on September 29.
    McGee says leaving without a position lined up gave him time to reflect on what kind of company he wanted to run. It also sent a clear message to the outside world about his aspirations. And McGee isn’t alone. In recent weeks the No.2 executives at Avon and American Express quit with the explanation that they were looking for a CEO post. As boards scrutinize succession plans in response to shareholder pressure, executives who don’t get the nod also may wish to move on. A turbulent business environment also has senior managers cautious of letting vague pronouncements cloud their reputations.
    As the first signs of recovery begin to take hold, deputy chiefs may be more willing to make the jump without a net. In the third quarter, CEO turnover was down 23% from a year ago as nervous boards stuck with the leaders they had, according to Liberum Research. As the economy picks up, opportunities will abound for aspiring leaders.
    The decision to quit a senior position to look for a better one is unconventional. For years executives and headhunters have adhered to the rule that the most attractive CEO candidates are the ones who must be poached. Says Korn/Ferry senior partner Dennis Carey:”I can’t think of a single search I’ve done where a board has not instructed me to look at sitting CEOs first.”
    Those who jumped without a job haven’t always landed in top positions quickly. Ellen Marram quit as chief of Tropicana a decade age, saying she wanted to be a CEO. It was a year before she became head of a tiny Internet-based commodities exchange. Robert Willumstad left Citigroup in 2005 with ambitions to be a CEO. He finally took that post at a major financial institution three years later.
    Many recruiters say the old disgrace is fading for top performers. The financial crisis has made it more acceptable to be between jobs or to leave a bad one. “The traditional rule was it’s safer to stay where you are, but that’s been fundamentally inverted,” says one headhunter. “The people who’ve been hurt the worst are those who’ve stayed too long.”
  6. When McGee announced his departure, his manner can best be described as being
    [A]arrogant. [B]frank. [C]self-centered. [D]impulsive.
  7. According to Paragraph 2, senior executives’ quitting may be spurred by
    [A]their expectation of better financial status. [B]their need to reflect on their private life.
    [C]their strained relations with the boards. [D]their pursuit of new career goals.
  8. The word “poached” (Line 3, Paragraph 4) most probably means
    [A]approved of. [B]attended to. [C]hunted for. [D]guarded against.
  9. It can be inferred from the last paragraph that
    [A]top performers used to cling to their posts. [B]loyalty of top performers is getting out-dated.
    [C]top performers care more about reputations. [D]it’s safer to stick to the traditional rules.
  10. Which of the following is the best title for the text?
    [A]CEOs: Where to Go? [B]CEOs: All the Way Up?
    [C]Top Managers Jump without a Net [D]The Only Way Out for Top Performers
    Text 3
    The rough guide to marketing success used to be that you got what you paid for. No longer. While traditional “paid” media – such as television commercials and print advertisements – still play a major role, companies today can exploit many alternative forms of media. Consumers passionate about a product may create “owned” media by sending e-mail alerts about products and sales to customers registered with its Web site. The way consumers now approach the broad range of factors beyond conventional paid media.
    Paid and owned media are controlled by marketers promoting their own products. For earned media , such marketers act as the initiator for users’ responses. But in some cases, one marketer’s owned media become another marketer’s paid media – for instance, when an e-commerce retailer sells ad space on its Web site. We define such sold media as owned media whose traffic is so strong that other organizations place their content or e-commerce engines within that environment. This trend ,which we believe is still in its infancy, effectively began with retailers and travel providers such as airlines and hotels and will no doubt go further. Johnson & Johnson, for example, has created BabyCenter, a stand-alone media property that promotes complementary and even competitive products. Besides generating income, the presence of other marketers makes the site seem objective, gives companies opportunities to learn valuable information about the appeal of other companies’ marketing, and may help expand user traffic for all companies concerned.
    The same dramatic technological changes that have provided marketers with more (and more diverse) communications choices have also increased the risk that passionate consumers will voice their opinions in quicker, more visible, and much more damaging ways. Such hijacked media are the opposite of earned media: an asset or campaign becomes hostage to consumers, other stakeholders, or activists who make negative allegations about a brand or product. Members of social networks, for instance, are learning that they can hijack media to apply pressure on the businesses that originally created them.
    If that happens, passionate consumers would try to persuade others to boycott products, putting the reputation of the target company at risk. In such a case, the company’s response may not be sufficiently quick or thoughtful, and the learning curve has been steep. Toyota Motor, for example, alleviated some of the damage from its recall crisis earlier this year with a relatively quick and well-orchestrated social-media response campaign, which included efforts to engage with consumers directly on sites such as Twitter and the social-news site Digg.
    31.Consumers may create “earned” media when they are
    [A] obscssed with online shopping at certain Web sites.
    [B] inspired by product-promoting e-mails sent to them.
    [C] eager to help their friends promote quality products.
    [D] enthusiastic about recommending their favorite products.
  11. According to Paragraph 2,sold media feature
    [A] a safe business environment. [B] random competition.
    [C] strong user traffic. [D] flexibility in organization.
  12. The author indicates in Paragraph 3 that earned media
    [A] invite constant conflicts with passionate consumers.
    [B] can be used to produce negative effects in marketing.
    [C] may be responsible for fiercer competition.
    [D] deserve all the negative comments about them.
  13. Toyota Motor’s experience is cited as an example of
    [A] responding effectively to hijacked media.[B] persuading customers into boycotting products.
    [C] cooperating with supportive consumers. [D] taking advantage of hijacked media.
  14. Which of the following is the text mainly about ?
    [A] Alternatives to conventional paid media.[B] Conflict between hijacked and earned media.
    [C] Dominance of hijacked media. [D] Popularity of owned media.
    Text 4
    It’s no surprise that Jennifer Senior’s insightful, provocative magazine cover story, “I love My Children, I Hate My Life,” is arousing much chatter – nothing gets people talking like the suggestion that child rearing is anything less than a completely fulfilling, life-enriching experience. Rather than concluding that children make parents either happy or miserable, Senior suggests we need to redefine happiness: instead of thinking of it as something that can be measured by moment-to-moment joy, we should consider being happy as a past-tense condition. Even though the day-to-day experience of raising kids can be soul-crushingly hard, Senior writes that “the very things that in the moment dampen our moods can later be sources of intense gratification and delight.”
    The magazine cover showing an attractive mother holding a cute baby is hardly the only Madonna-and-child image on newsstands this week. There are also stories about newly adoptive – and newly single – mom Sandra Bullock, as well as the usual “Jennifer Aniston is pregnant” news. Practically every week features at least one celebrity mom, or mom-to-be, smiling on the newsstands.
    In a society that so persistently celebrates procreation, is it any wonder that admitting you regret having children is equivalent to admitting you support kitten-killing ? It doesn’t seem quite fair, then, to compare the regrets of parents to the regrets of the children. Unhappy parents rarely are provoked to wonder if they shouldn’t have had kids, but unhappy childless folks are bothered with the message that children are the single most important thing in the world: obviously their misery must be a direct result of the gaping baby-size holes in their lives.
    Of course, the image of parenthood that celebrity magazines like Us Weekly and People present is hugely unrealistic, especially when the parents are single mothers like Bullock. According to several studies concluding that parents are less happy than childless couples, single parents are the least happy of all. No shock there, considering how much work it is to raise a kid without a partner to lean on; yet to hear Sandra and Britney tell it, raising a kid on their “own” (read: with round-the-clock help) is a piece of cake.
    It’s hard to imagine that many people are dumb enough to want children just because Reese and Angelina make it look so glamorous: most adults understand that a baby is not a haircut. But it’s interesting to wonder if the images we see every week of stress-free, happiness-enhancing parenthood aren’t in some small, subconscious way contributing to our own dissatisfactions with the actual experience, in the same way that a small part of us hoped getting “ the Rachel” might make us look just a little bit like Jennifer Aniston.
    36.Jennifer Senior suggests in her article that raising a child can bring
    [A]temporary delight [B]enjoyment in progress
    [C]happiness in retrospect [D]lasting reward
    37.We learn from Paragraph 2 that
    [A]celebrity moms are a permanent source for gossip.
    [B]single mothers with babies deserve greater attention.
    [C]news about pregnant celebrities is entertaining.
    [D]having children is highly valued by the public.
    38.It is suggested in Paragraph 3 that childless folks
    [A]are constantly exposed to criticism. [B]are largely ignored by the media.
    [C]fail to fulfill their social responsibilities. [D]are less likely to be satisfied with their life.
    39.According to Paragraph 4, the message conveyed by celebrity magazines is
    [A]soothing .[B]ambiguous. [C]compensatory. [D]misleading.
    40.Which of the following can be inferred from the last paragraph?
    [A]Having children contributes little to the glamour of celebrity moms.
    [B]Celebrity moms have influenced our attitude towards child rearing.
    [C]Having children intensifies our dissatisfaction with life.
    [D]We sometimes neglect the happiness from child rearing.
    Part B
    Directions:
    The following paragraph are given in a wrong order. For Questions 41-45, you are required to reorganize these paragraphs into a coherent text by choosing from the list A-G to filling them into the numbered boxes. Paragraphs E and G have been correctly placed. Mark your answers on ANSWER SHEET 1. (10 points)
    [A] No disciplines have seized on professionalism with as much enthusiasm as the humanities. You can, Mr Menand points out, became a lawyer in three years and a medical doctor in four. But the regular time it takes to get a doctoral degree in the humanities is nine years. Not surprisingly, up to half of all doctoral students in English drop out before getting their degrees.
    [B] His concern is mainly with the humanities: Literature, languages, philosophy and so on. These are disciplines that are going out of style: 22% of American college graduates now major in business compared with only 2% in history and 4% in English. However, many leading American universities want their undergraduates to have a grounding in the basic canon of ideas that every educated person should posses. But most find it difficult to agree on what a “general education” should look like. At Harvard, Mr Menand notes, “the great books are read because they have been read”-they form a sort of social glue.
    [C] Equally unsurprisingly, only about half end up with professorships for which they entered graduate school. There are simply too few posts. This is partly because universities continue to produce ever more PhDs. But fewer students want to study humanities subjects: English departments awarded more bachelor’s degrees in 1970-71 than they did 20 years later. Fewer students requires fewer teachers. So, at the end of a decade of theses-writing, many humanities students leave the profession to do something for which they have not been trained.
    [D] One reason why it is hard to design and teach such courses is that they can cut across the insistence by top American universities that liberal-arts educations and professional education should be kept separate, taught in different schools. Many students experience both varieties. Although more than half of Harvard undergraduates end up in law, medicine or business, future doctors and lawyers must study a non-specialist liberal-arts degree before embarking on a professional qualification.
    [E] Besides professionalizing the professions by this separation, top American universities have professionalised the professor. The growth in public money for academic research has speeded the process: federal research grants rose fourfold between 1960and 1990, but faculty teaching hours fell by half as research took its toll. Professionalism has turned the acquisition of a doctoral degree into a prerequisite for a successful academic career: as late as 1969a third of American professors did not possess one. But the key idea behind professionalisation, argues Mr Menand, is that “the knowledge and skills needed for a particular specialization are transmissible but not transferable.”So disciplines acquire a monopoly not just over the production of knowledge, but also over the production of the producers of knowledge.
    [F] The key to reforming higher education, concludes Mr Menand, is to alter the way in which “the producers of knowledge are produced.”Otherwise, academics will continue to think dangerously alike, increasingly detached from the societies which they study, investigate and criticize.”Academic inquiry, at least in some fields, may need to become less exclusionary and more holistic.”Yet quite how that happens, Mr Menand dose not say.
    [G] The subtle and intelligent little book The Marketplace of Ideas: Reform and Resistance in the American University should be read by every student thinking of applying to take a doctoral degree. They may then decide to go elsewhere. For something curious has been happening in American Universities, and Louis Menand, a professor of English at Harvard University, captured it skillfully.
    G → 41. → 42. →E → 43. → 44. →45.
    Part C
    Directions:
    Read the following text carefully and then translate the underlined segments into Chinese. Your translation should be written carefully on ANSWER SHEET 2. (10 points)
    With its theme that “Mind is the master weaver,” creating our inner character and outer circumstances, the book As a Man Thinking by James Allen is an in-depth exploration of the central idea of self-help writing.
    (46) Allen’s contribution was to take an assumption we all share-that because we are not robots we therefore control our thoughts-and reveal its erroneous nature. Because most of us believe that mind is separate from matter, we think that thoughts can be hidden and made powerless; this allows us to think one way and act another. However, Allen believed that the unconscious mind generates as much action as the conscious mind, and (47) while we may be able to sustain the illusion of control through the conscious mind alone, in reality we are continually faced with a question: “Why cannot I make myself do this or achieve that? ”
    Since desire and will are damaged by the presence of thoughts that do not accord with desire, Allen concluded : “ We do not attract what we want, but what we are.” Achievement happens because you as a person embody the external achievement; you don’t “ get” success but become it. There is no gap between mind and matter.
    Part of the fame of Allen’s book is its contention that “Circumstances do not make a person, they reveal him.” (48) This seems a justification for neglect of those in need, and a rationalization of exploitation, of the superiority of those at the top and the inferiority of those at the bottom.
    This ,however, would be a knee-jerk reaction to a subtle argument. Each set of circumstances, however bad, offers a unique opportunity for growth. If circumstances always determined the life and prospects of people, then humanity would never have progressed. In fat, (49)circumstances seem to be designed to bring out the best in us and if we feel that we have been “wronged” then we are unlikely to begin a conscious effort to escape from our situation .Nevertheless, as any biographer knows, a person’s early life and its conditions are often the greatest gift to an individual.
    The sobering aspect of Allen’s book is that we have no one else to blame for our present condition except ourselves. (50) The upside is the possibilities contained in knowing that everything is up to us; where before we were experts in the array of limitations, now we become authorities of what is possible.
    Section Ⅲ Writing
    Part A
  15. Directions:
    Write a letter to a friend of yours to
  1. recommend one of your favorite movies and
  2. give reasons for your recommendation
    Your should write about 100 words on ANSWER SHEET 2
    Do not sign your own name at the end of the leter. User “LI MING” instead.
    Do not writer the address.(10 points)
    Part B
  1. Directions:
    Write an essay of 160---200 words based on the following drawing. In your essay, you should
  1. describe the drawing briefly,
  2. explain it’s intended meaning, and
  3. give your comments.
    Your should write neatly on ANSWER SHEET 2. (20 points)



    2010年全国硕士研究生入学统一考试英语试题
    Section I Use of English
    Directions:
    In 1924 America's National Research Council sent two engineers to supervise a series of industrial experiments at a large telephone-parts factory called the Hawthorne Plant near Chicago. It hoped they would learn how stop-floor lighting workers' productivity. Instead, the studies ended giving their name to the "Hawthorne effect", the extremely influential idea that the very to being experimented upon changed subjects' behavior.
    The idea arose because of the behavior of the women in the Hawthorne plant. According to of the experiments, their hourly output rose when lighting was increased, but also when it was dimmed. It did not what was done in the experiment; something was changed, productivity rose. A(n) that they were being experimented upon seemed to be to alter workers' behavior itself.
    After several decades, the same data were to econometric the analysis. Hawthorne experiments has another surprise store the descriptions on record, no systematic was found that levels of productivity were related to changes in lighting.
    It turns out that peculiar way of conducting the experiments may be have let to interpretation of what happed. , lighting was always changed on a Sunday. When work started again on Monday, output rose compared with the previous Saturday and 17 to rise for the next couple of days. , a comparison with data for weeks when there was no experimentation showed that output always went up on Monday, workers to be diligent for the first few days of the week in any case, before a plateau and then slackening off. This suggests that the alleged "Hawthorne effect" is hard to pin down.

  1. [A] affected [B] achieved [C] extracted [D] restored
  2. [A] at [B] up [C] with [D] off
  3. [A] truth [B] sight [C] act [D] proof
  4. [A] controversial [B] perplexing [C] mischievous [D] ambiguous
  5. [A] requirements [B] explanations [C] accounts [D] assessments
  6. [A] conclude [B] matter [C] indicate [D] work
  7. [A] as far as [B] for fear that [C] in case that [D] so long as
  8. [A] awareness [B] expectation [C] sentiment [D] illusion
  9. [A] suitable [B] excessive [C] enough [D] abundant
  10. [A] about [B] for [C] on [D] by
  11. [A] compared [B] shown [C] subjected [D] conveyed
  12. [A] contrary to [B] consistent with [C] parallel with [D] peculiar to
  13. [A] evidence [B] guidance [C] implication [D] source
  14. [A] disputable [B] enlightening [C] reliable [D] misleading
  15. [A] In contrast [B] For example [C] In consequence [D] As usual
  16. [A] duly [B] accidentally [C] unpredictably [D] suddenly
  17. [A] failed [B] ceased [C] started [D] continued
  18. [A] breaking [B] climbing [C] surpassing [D] hitting
     Section II Reading Comprehension
    Part A
    Directions:
    Read the following four texts. Answer the questions below each text by choosing [A], [B], [C] or [D]. Mark your answers on ANSWER SHEET 1. (40 points)
    Text 1
    Of all the changes that have taken place in English-language newspapers during the past quarter-century, perhaps the most far-reaching has been the inexorable decline in the scope and seriousness of their arts coverage.
    It is difficult to the point of impossibility for the average reader under the age of forty to imagine a time when high-quality arts criticism could be found in most big-city newspapers. Yet a considerable number of the most significant collections of criticism published in the 20th century consisted in large part of newspaper reviews. To read such books today is to marvel at the fact that their learned contents were once deemed suitable for publication in general-circulation dailies.
    We are even farther removed from the unfocused newspaper reviews published in England between the turn of the 20th century and the eve of World War II, at a time when newsprint was dirt-cheap and stylish arts criticism was considered an ornament to the publications in which it appeared. In those far-off days, it was taken for granted that the critics of major papers would write in detail and at length about the events they covered. Theirs was a serious business, and even those reviewers who wore their learning lightly, like George Bernard Shaw and Ernest Newman, could be trusted to know what they were about. These men believed in journalism as a calling, and were proud to be published in the daily press. “So few authors have brains enough or literary gift enough to keep their own end up in journalism,” Newman wrote, “that I am tempted to define ‘journalism’ as ‘a term of contempt applied by writers who are not read to writers who are.’”
    Unfortunately, these critics are virtually forgotten. Neville Cardus, who wrote for the Manchester Guardian from 1917 until shortly before his death in 1975, is now known solely as a writer of essays on the game of cricket. During his lifetime, though, he was also one of England’s foremost classical-music critics, a stylist so widely admired that his Autobiography (1947) became a best-seller. He was knighted in 1967, the first music critic to be so honored. Yet only one of his books is now in print, and his vast body of writings on music is unknown save to specialists.
    Is there any chance that Cardus’s criticism will enjoy a revival? The prospect seems remote. Journalistic tastes had changed long before his death, and postmodern readers have little use for the richly upholstered Vicwardian prose in which he specialized. Moreover, the amateur tradition in music criticism has been in headlong retreat.
  19. It is indicated in Paragraphs 1 and 2 that
    [A] arts criticism has disappeared from big-city newspapers.
    [B] English-language newspapers used to carry more arts reviews.
    [C] high-quality newspapers retain a large body of readers.
    [D] young readers doubt the suitability of criticism on dailies.
  20. Newspaper reviews in England before World War II were characterized by
    [A] free themes. [B] casual style. [C] elaborate layout. [D] radical viewpoints.
  21. Which of the following would Shaw and Newman most probably agree on?
    [A] It is writers' duty to fulfill journalistic goals.[B] It is contemptible for writers to be journalists.
    [C] Writers are likely to be tempted into journalism.
    [D] Not all writers are capable of journalistic writing.
  22. What can be learned about Cardus according to the last two paragraphs?
    [A] His music criticism may not appeal to readers today.
    [B] His reputation as a music critic has long been in dispute.
    [C] His style caters largely to modern specialists.[D] His writings fail to follow the amateur tradition.
  23. What would be the best title for the text?
    [A] Newspapers of the Good Old Days [B] The Lost Horizon in Newspapers
    [C] Mournful Decline of Journalism [D] Prominent Critics in Memory
    Text 2
    Over the past decade, thousands of patents have been granted for what are called business methods. Amazon.com received one for its "one-click" online payment system. Merrill Lynch got legal protection for an asset allocation strategy. One inventor patented a technique for lifting a box.
    Now the nation's top patent court appears completely ready to scale back on business-method patents, which have been controversial ever since they were first authorized 10 years ago. In a move that has intellectual-property lawyers abuzz the U.S. court of Appeals for the federal circuit said it would use a particular case to conduct a broad review of business-method patents. In re Bilski, as the case is known , is "a very big deal", says Dennis D. Crouch of the University of Missouri School of law. It "has the potential to eliminate an entire class of patents."
    Curbs on business-method claims would be a dramatic about-face, because it was the federal circuit itself that introduced such patents with is 1998 decision in the so-called state Street Bank case, approving a patent on a way of pooling mutual-fund assets. That ruling produced an explosion in business-method patent filings, initially by emerging internet companies trying to stake out exclusive rights to specific types of online transactions. Later, move established companies raced to add such patents to their files, if only as a defensive move against rivals that might beat them to the punch. In 2005, IBM noted in a court filing that it had been issued more than 300 business-method patents despite the fact that it questioned the legal basis for granting them. Similarly, some Wall Street investment films armed themselves with patents for financial products, even as they took positions in court cases opposing the practice.
    The Bilski case involves a claimed patent on a method for hedging risk in the energy market. The Federal circuit issued an unusual order stating that the case would be heard by all 12 of the court's judges, rather than a typical panel of three, and that one issue it wants to evaluate is whether it should "reconsider" its state street Bank ruling.
    The Federal Circuit's action comes in the wake of a series of recent decisions by the supreme Court that has narrowed the scope of protections for patent holders. Last April, for example the justices signaled that too many patents were being upheld for "inventions" that are obvious. The judges on the Federal circuit are "reacting to the anti-patent trend at the Supreme Court", says Harold C. Wegner, a patent attorney and professor at George Washington University Law School.
  24. Business-method patents have recently aroused concern because of
    [A] their limited value to business [B] their connection with asset allocation
    [C] the possible restriction on their granting [D] the controversy over authorization
  25. Which of the following is true of the Bilski case?
    [A] Its ruling complies with the court decisions [B] It involves a very big business transaction
    [C] It has been dismissed by the Federal Circuit [D] It may change the legal practices in the U.S.
  26. The word "about-face" (Line 1, Para 3) most probably means
    [A] loss of good will [B] increase of hostility
    [C] change of attitude [D] enhancement of dignity
  27. We learn from the last two paragraphs that business-method patents
    [A] are immune to legal challenges [B] are often unnecessarily issued
    [C] lower the esteem for patent holders [D] increase the incidence of risks
  28. Which of the following would be the subject of the text?
    [A] A looming threat to business-method patents[B] Protection for business-method patent holders
    [C] A legal case regarding business-method patents
    [D] A prevailing trend against business-method patents
    Text 3
    In his book The Tipping Point, Malcolm Gladwell argues that social epidemics are driven in large part by the acting of a tiny minority of special individuals, often called influentials, who are unusually informed, persuasive, or well-connected. The idea is intuitively compelling, but it doesn't explain how ideas actually spread.
    The supposed importance of influentials derives from a plausible sounding but largely untested theory called the "two step flow of communication": Information flows from the media to the influentials and from them to everyone else. Marketers have embraced the two-step flow because it suggests that if they can just find and influence the influentials, those selected people will do most of the work for them. The theory also seems to explain the sudden and unexpected popularity of certain looks, brands, or neighborhoods. In many such cases, a cursory search for causes finds that some small group of people was wearing, promoting, or developing whatever it is before anyone else paid attention. Anecdotal evidence of this kind fits nicely with the idea that only certain special people can drive trends
    In their recent work, however, some researchers have come up with the finding that influentials have far less impact on social epidemics than is generally supposed. In fact, they don't seem to be required of all.
    The researchers' argument stems from a simple observing about social influence, with the exception of a few celebrities like Oprah Winfrey—whose outsize presence is primarily a function of media, not interpersonal, influence—even the most influential members of a population simply don't interact with that many others. Yet it is precisely these non-celebrity influentials who, according to the two-step-flow theory, are supposed to drive social epidemics by influencing their friends and colleagues directly. For a social epidemic to occur, however, each person so affected, must then influence his or her own acquaintances, who must in turn influence theirs, and so on; and just how many others pay attention to each of these people has little to do with the initial influential. If people in the network just two degrees removed from the initial influential prove resistant, for example from the initial influential prove resistant, for example the cascade of change won't propagate very far or affect many people.
    Building on the basic truth about interpersonal influence, the researchers studied the dynamics of populations manipulating a number of variables relating of populations, manipulating a number of variables relating to people's ability to influence others and their tendency to be influenced. Our work shows that the principal requirement for what we call "global cascades"– the widespread propagation of influence through networks – is the presence not of a few influentials but, rather, of a critical mass of easily influenced people, each of whom adopts, say, a look or a brand after being exposed to a single adopting neighbor. Regardless of how influential an individual is locally, he or she can exert global influence only if this critical mass is available to propagate a chain reaction.
  29. By citing the book The Tipping Point, the author intends to
    [A] analyze the consequences of social epidemics[B] discuss influentials' function in spreading ideas
    [C] exemplify people's intuitive response to social epidemics
    [D] describe the essential characteristics of influentials.
  30. The author suggests that the "two-step-flow theory"
    [A] serves as a solution to marketing problems[B] has helped explain certain prevalent trends
    [C] has won support from influentials[D] requires solid evidence for its validity
  31. What the researchers have observed recently shows that
    [A] the power of influence goes with social interactions
    [B] interpersonal links can be enhanced through the media
    [C] influentials have more channels to reach the public
    [D] most celebrities enjoy wide media attention
  32. The underlined phrase "these people" in paragraph 4 refers to the ones who
    [A] stay outside the network of social influence[B] have little contact with the source of influence
    [C] are influenced and then influence others [D] are influenced by the initial influential
  33. what is the essential element in the dynamics of social influence?
    [A] The eagerness to be accepted [B] The impulse to influence others
    [C] The readiness to be influenced [D] The inclination to rely on others
    Text 4
    Bankers have been blaming themselves for their troubles in public. Behind the scenes, they have been taking aim at someone else: the accounting standard-setters. Their rules, moan the banks, have forced them to report enormous losses, and it's just not fair. These rules say they must value some assets at the price a third party would pay, not the price managers and regulators would like them to fetch.
    Unfortunately, banks' lobbying now seems to be working. The details may be unknowable, but the independence of standard-setters, essential to the proper functioning of capital markets, is being compromised. And, unless banks carry toxic assets at prices that attract buyers, reviving the banking system will be difficult.
    After a bruising encounter with Congress, America's Financial Accounting Standards Board (FASB) rushed through rule changes. These gave banks more freedom to use models to value illiquid assets and more flexibility in recognizing losses on long-term assets in their income statement. Bob Herz, the FASB's chairman, cried out against those who "question our motives." Yet bank shares rose and the changes enhance what one lobby group politely calls "the use of judgment by management."
    European ministers instantly demanded that the International Accounting Standards Board (IASB) do likewise. The IASB says it does not want to act without overall planning, but the pressure to fold when it completes it reconstruction of rules later this year is strong. Charlie McCreevy, a European commissioner, warned the IASB that it did "not live in a political vacuum" but "in the real word" and that Europe could yet develop different rules.
    It was banks that were on the wrong planet, with accounts that vastly overvalued assets. Today they argue that market prices overstate losses, because they largely reflect the temporary illiquidity of markets, not the likely extent of bad debts. The truth will not be known for years. But bank's shares trade below their book value, suggesting that investors are skeptical. And dead markets partly reflect the paralysis of banks which will not sell assets for fear of booking losses, yet are reluctant to buy all those supposed bargains.
    To get the system working again, losses must be recognized and dealt with. America's new plan to buy up toxic assets will not work unless banks mark assets to levels which buyers find attractive. Successful markets require independent and even combative standard-setters. The FASB and IASB have been exactly that, cleaning up rules on stock options and pensions, for example, against hostility from special interests. But by giving in to critics now they are inviting pressure to make more concessions.
  34. Bankers complained that they were forced to
    [A] follow unfavorable asset evaluation rules [B] collect payments from third parties
    [C] cooperate with the price managers [D] reevaluate some of their assets.
  35. According to the author , the rule changes of the FASB may result in
    [A] the diminishing role of management [B] the revival of the banking system
    [C] the banks' long-term asset losses [D] the weakening of its independence
  36. According to Paragraph 4, McCreevy objects to the IASB's attempt to
    [A] keep away from political influences. [B] evade the pressure from their peers.
    [C] act on their own in rule-setting. [D] take gradual measures in reform.
  37. The author thinks the banks were "on the wrong planet" in that they
    [A] misinterpreted market price indicators [B] exaggerated the real value of their assets
    [C] neglected the likely existence of bad debts. [D] denied booking losses in their sale of assets.
  38. The author's attitude towards standard-setters is one of
    [A] satisfaction. [B] skepticism. [C] objectiveness [D] sympathy
    Part B
    Directions:
    For Questions 41-45, choose the most suitable paragraphs from the list A-G and fill them into the numbered boxes to form a coherent text. Paragraph E has been correctly placed. There is one paragraph which does not fit in with the text. Mark your answers on ANSWER SHEET1. (10 points)
    [A] The first and more important is the consumer's growing preference for eating out; the consumption of food and drink in places other than homes has risen from about 32 percent of total consumption in 1995 to 35 percent in 2000 and is expected to approach 38 percent by 2005. This development is boosting wholesale demand from the food service segment by 4 to 5 percent a year across Europe, compared with growth in retail demand of 1 to 2 percent. Meanwhile, as the recession is looming large, people are getting anxious. They tend to keep a tighter hold on their purse and consider eating at home a realistic alternative.
    [B] Retail sales of food and drink in Europe's largest markets are at a standstill, leaving European grocery retailers hungry for opportunities to grow. Most leading retailers have already tried e-commerce, with limited success, and expansion abroad. But almost all have ignored the big, profitable opportunity in their own backyard: the wholesale food and drink trade, which appears to be just the kind of market retailers need.
    [C] Will such variations bring about a change in the overall structure of the food and drink market? Definitely not. The functioning of the market is based on flexible trends dominated by potential buyers. In other words, it is up to the buyer, rather than the seller, to decide what to buy .At any rate, this change will ultimately be acclaimed by an ever-growing number of both domestic and international consumers, regardless of how long the current consumer pattern will take hold.
    [D] All in all, this clearly seems to be a market in which big retailers could profitably apply their scale, existing infrastructure and proven skills in the management of product ranges, logistics, and marketing intelligence. Retailers that master the intricacies of wholesaling in Europe may well expect to rake in substantial profits thereby. At least, that is how it looks as a whole. Closer inspection reveals important differences among the biggest national markets, especially in their customer segments and wholesale structures, as well as the competitive dynamics of individual food and drink categories. Big retailers must understand these differences before they can identify the segments of European wholesaling in which their particular abilities might unseat smaller but entrenched competitors. New skills and unfamiliar business models are needed too.
    [E] Despite variations in detail, wholesale markets in the countries that have been closely examined—France, Germany, Italy, and Spain—are made out of the same building blocks. Demand comes mainly from two sources: independent mom-and-pop grocery stores which, unlike large retail chains, are two small to buy straight from producers, and food service operators that cater to consumers when they don't eat at home. Such food service operators range from snack machines to large institutional catering ventures, but most of these businesses are known in the trade as "horeca": hotels, restaurants, and cafes. Overall, Europe's wholesale market for food and drink is growing at the same sluggish pace as the retail market, but the figures, when added together, mask two opposing trends.
    [F] For example, wholesale food and drink sales come to $268 billion in France, Germany, Italy, Spain, and the United Kingdom in 2000—more than 40 percent of retail sales. Moreover, average overall margins are higher in wholesale than in retail; wholesale demand from the food service sector is growing quickly as more Europeans eat out more often; and changes in the competitive dynamics of this fragmented industry are at last making it feasible for wholesalers to consolidate.
    [G] However, none of these requirements should deter large retailers (and even some large good producers and existing wholesalers) from trying their hand, for those that master the intricacies of wholesaling in Europe stand to reap considerable gains.
    41 →42 →43 →44 → E →45
    Part C
    Directions:
    Read the following text carefully and then translate the underlined segments into Chinese. Your translation should be written carefully on ANSWER SHEET 2. (10 points)
    One basic weakness in a conservation system based wholly on economic motives is that most members of the land community have no economic value. Yet these creatures are members of the biotic community and, if its stability depends on its integrity, they are entitled to continuance.
    When one of these noneconomic categories is threatened and, if we happen to love it .We invert excuses to give it economic importance. At the beginning of century songbirds were supposed to be disappearing. (46) Scientists jumped to the rescue with some distinctly shaky evidence to the effect that insects would eat us up if birds failed to control them. the evidence had to be economic in order to be valid.
    It is painful to read these round about accounts today. We have no land ethic yet, (47) but we have at least drawn near the point of admitting that birds should continue as a matter of intrinsic right, regardless of the presence or absence of economic advantage to us.
    A parallel situation exists in respect of predatory mammals and fish-eating birds. (48) Time was when biologists somewhat over worded the evidence that these creatures preserve the health of game by killing the physically weak, or that they prey only on "worthless" species.
    Some species of tree have been read out of the party by economics-minded foresters because they grow too slowly, or have too low a sale vale to pay as timber crops. (49) In Europe, where forestry is ecologically more advanced, the non-commercial tree species are recognized as members of native forest community, to be preserved as such, within reason.
    To sum up: a system of conservation based solely on economic self-interest is hopelessly lopsided. (50) It tends to ignore, and thus eventually to eliminate, many elements in the land community that lack commercial value, but that are essential to its healthy functioning. It assumes, falsely, I think, that the economic parts of the biotic clock will function without the uneconomic parts.
    Section Ⅲ Writing
    Part A
  39. Directions:
    You are supposed to write for the postgraduate association a notice to recruit volunteers for an international conference on globalization, you should conclude the basic qualification of applicant and the other information you think relative.
    You should write about 100 words. Do not sign your own name at the end of the letter. Use "postgraduate association" instead.
    Part B
  40. Directions:
    Write an essay of 160-200 words based on the following drawing. In your essay, you should
  1. describe the drawing briefly,
  2. explain its intended meaning, and then
  3. give your comments.
    2009年全国硕士研究生入学统一考试
    英语试题
    Section I Use of English
    Directions:
    Research on animal intelligence always makes me wonder just how smart humans are. 1 the fruit-fly experiments described in Carl Zimmer’s piece in the Science Times on Tuesday. Fruit flies who were taught to be smarter than the average fruit fly 2 to live shorter lives. This suggests that 3 bulbs burn longer, that there is an 4 in not being too terrifically bright. Intelligence, it 5 out, is a high-priced option. It takes more upkeep, burns more fuel and is slow 6 the starting line because it depends on learning — a gradual 7 — instead of instinct. Plenty of other species are able to learn, and one of the things they’ve apparently learned is when to 8 . Is there an adaptive value to 9 intelligence? That’s the question behind this new research. I like it. Instead of casting a wistful glance 10 at all the species we’ve left in the dust I.Q.-wise, it implicitly asks what the real 11 of our own intelligence might be. This is 12 the mind of every animal I’ve ever met. Research on animal intelligence also makes me wonder what experiments animals would 13 on humans if they had the chance. Every cat with an owner, 14 , is running a small-scale study in operant conditioning. we believe that 15 animals ran the labs, they would test us to 16 the limits of our patience, our faithfulness, our memory for terrain. They would try to decide what intelligence in humans is really 17 , not merely how much of it there is. 18 , they would hope to study a 19 question: Are humans actually aware of the world they live in? 20 the results are inconclusive.
  1. [A] Suppose [B] Consider [C] Observe [D] Imagine
  2. [A] tended [B] feared [C] happened [D] threatened
  3. [A] thinner [B] stabler [C] lighter [D] dimmer
  4. [A] tendency [B] advantage [C] inclination [D] priority
  5. [A] insists on [B] sums up [C] turns out [D] puts forward
  6. [A] off [B] behind [C] over [D] along
  7. [A] incredible [B] spontaneous [C]inevitable [D] gradual
  8. [A] fight [B] doubt [C] stop [D] think
  9. [A] invisible [B] limited [C] indefinite [D] different
    10.[A] upward [B] forward [C] afterward [D] backward
  10. [A] features [B] influences [C] results [D] costs
  11. [A] outside [B] on [C] by [D] across
  12. [A] deliver [B] carry [C] perform [D] apply
  13. [A] by chance [B] in contrast [C] as usual [D] for instance
  14. [A] if [B] unless [C] as [D] lest
  15. [A] moderate [B] overcome [C] determine [D] reach
  16. [A] at [B] for [C] after [D] with
  17. [A] Above all [B] After all [C] However [D] Otherwise
  18. [A] fundamental [B] comprehensive [C] equivalent [D] hostile
  19. [A] By accident [B] In time [C] So far [D] Better still
    Section II Reading Comprehension
    Part A
    Directions:
    Read the following four texts. Answer the questions below each text by choosing A, B, C or D. Mark your answers on ANSWER SHEET 1. (40 points)
    Text1
    Habits are a funny thing. We reach for them mindlessly, setting our brains on auto-pilot and relaxing into the unconscious comfort of familiar routine. “Not choice, but habit rules the unreflecting herd,” William Wordsworth said in the 19th century. In the ever-changing 21st century, even the word “habit” carries a negative connotation.
    So it seems antithetical to talk about habits in the same context as creativity and innovation. But brain researchers have discovered that when we consciously develop new habits, we create parallel synaptic paths, and even entirely new brain cells, that can jump our trains of thought onto new, innovative tracks.
    But don’t bother trying to kill off old habits; once those ruts of procedure are worn into the hippocampus, they’re there to stay. Instead, the new habits we deliberately ingrain into ourselves create parallel pathways that can bypass those old roads.
    “The first thing needed for innovation is a fascination with wonder,” says Dawna Markova, author of “The Open Mind” and an executive change consultant for Professional Thinking Partners. “But we are taught instead to ‘decide,’ just as our president calls himself ‘the Decider.’ ” She adds, however, that “to decide is to kill off all possibilities but one. A good innovational thinker is always exploring the many other possibilities.”
    All of us work through problems in ways of which we’re unaware, she says. Researchers in the late 1960 covered that humans are born with the capacity to approach challenges in four primary ways: analytically, procedurally, relationally (or collaboratively) and innovatively. At puberty, however, the brain shuts down half of that capacity, preserving only those modes of thought that have seemed most valuable during the first decade or so of life.
    The current emphasis on standardized testing highlights analysis and procedure, meaning that few of us inherently use our innovative and collaborative modes of thought. “This breaks the major rule in the American belief system — that anyone can do anything,” explains M. J. Ryan, author of the 2006 book “This Year I Will...” and Ms. Markova’s business partner. “That’s a lie that we have perpetuated, and it fosters commonness. Knowing what you’re good at and doing even more of it creates excellence.” This is where developing new habits comes in.
  20. The view of Wordsworth habit is claimed by being
    A. casual B. familiar C. mechanical D. changeable.
  21. The researchers have discovered that the formation of habit can be
    A. predicted B. regulated C. traced D. guided
    23.“ruts”(in line one, paragraph 3) has closest meaning to
    A. tracks B. series C. characteristics D. connections
  22. Ms. Markova’s comments suggest that the practice of standard testing ?
    A, prevents new habits form being formed B, no longer emphasizes commonness
    C, maintains the inherent American thinking model D, complies with the American belief system
  23. Ryan most probably agree that
    A. ideas are born of a relaxing mind B. innovativeness could be taught
    C. decisiveness derives from fantastic ideas D. curiosity activates creative minds
    Text 2
    It is a wise father that knows his own child, but today a man can boost his paternal (fatherly) wisdom – or at least confirm that he’s the kid’s dad. All he needs to do is shell our $30 for paternity testing kit (PTK) at his local drugstore – and another $120 to get the results.
    More than 60,000 people have purchased the PTKs since they first become available without prescriptions last years, according to Doug Fog, chief operating officer of Identigene, which makes the over-the-counter kits. More than two dozen companies sell DNA tests Directly to the public , ranging in price from a few hundred dollars to more than $2500.
    Among the most popular : paternity and kinship testing , which adopted children can use to find their biological relatives and latest rage a many passionate genealogists-and supports businesses that offer to search for a family’s geographic roots .
    Most tests require collecting cells by webbing saliva in the mouth and sending it to the company for testing. All tests require a potential candidate with whom to compare DNA.
    But some observers are skeptical, “There is a kind of false precision being hawked by people claiming they are doing ancestry testing,” says Trey Duster, a New York University sociologist. He notes that each individual has many ancestors-numbering in the hundreds just a few centuries back. Yet most ancestry testing only considers a single lineage, either the Y chromosome inherited through men in a father’s line or mitochondrial DNA, which a passed down only from mothers. This DNA can reveal genetic information about only one or two ancestors, even though, for example, just three generations back people also have six other great-grandparents or, four generations back, 14 other great-great-grandparents.
    Critics also argue that commercial genetic testing is only as good as the reference collections to which a sample is compared. Databases used by some companies don’t rely on data collected systematically but rather lump together information from different research projects. This means that a DNA database may differ depending on the company that processes the results. In addition, the computer programs a company uses to estimate relationships may be patented and not subject to peer review or outside evaluation.
    26.In paragraphs 1 and 2 , the text shows PTK’s ___________.
    [A]easy availability [B]flexibility in pricing [C] successful promotion[D] popularity with households
  24. PTK is used to __________.
    [A] locate one’s birth place [B] promote genetic research
    [C] identify parent-child kinship [D] choose children for adoption
  25. Skeptical observers believe that ancestry testing fails to__________.
    [A]trace distant ancestors [B]rebuild reliable bloodlines
    [C]fully use genetic information [D]achieve the claimed accuracy
  26. In the last paragraph ,a problem commercial genetic testing faces is __________.
    [A]disorganized data collection [B]overlapping database building
  27. An appropriate title for the text is most likely to be__________.
    [A] Fors and Againsts of DNA testing [B] DNA testing and It’s problems
    [C] DNA testing outside the lab [D] lies behind DNA testing
    Text 3
    The relationship between formal education and economic growth in poor countries is widely misunderstood by economists and politicians alike progress in both area is undoubtedly necessary for the social, political and intellectual development of these and all other societies; however, the conventional view that education should be one of the very highest priorities for promoting rapid economic development in poor countries is wrong. We are fortunate that is it, because new educational systems there and putting enough people through them to improve economic performance would require two or three generations. The findings of a research institution have consistently shown that workers in all countries can be trained on the job to achieve radical higher productivity and, as a result, radically higher standards of living.
    Ironically, the first evidence for this idea appeared in the United States. Not long ago, with the country entering a recessing and Japan at its pre-bubble peak. The U.S. workforce was derided as poorly educated and one of primary cause of the poor U.S. economic performance. Japan was, and remains, the global leader in automotive-assembly productivity. Yet the research revealed that the U.S. factories of Honda Nissan, and Toyota achieved about 95 percent of the productivity of their Japanese countere pants a result of the training that U.S. workers received on the job.
    More recently, while examing housing construction, the researchers discovered that illiterate, non-English- speaking Mexican workers in Houston, Texas, consistently met best-practice labor productivity standards despite the complexity of the building industry’s work.
    What is the real relationship between education and economic development? We have to suspect that continuing economic growth promotes the development of education even when governments don’t force it. After all, that’s how education got started. When our ancestors were hunters and gatherers 10,000 years ago, they didn’t have time to wonder much about anything besides finding food. Only when humanity began to get its food in a more productive way was there time for other things.
    As education improved, humanity’s productivity potential, they could in turn afford more education. This increasingly high level of education is probably a necessary, but not a sufficient, condition for the complex political systems required by advanced economic performance. Thus poor countries might not be able to escape their poverty traps without political changes that may be possible only with broader formal education. A lack of formal education, however, doesn’t constrain the ability of the developing world’s workforce to substantially improve productivity for the forested future. On the contrary, constraints on improving productivity explain why education isn’t developing more quickly there than it is.
  28. The author holds in paragraph 1 that the important of education in poor countries ___________.
    [A] is subject groundless doubts [B] has fallen victim of bias
    [C] is conventional downgraded [D] has been overestimated
  29. It is stated in paragraph 1 that construction of a new education system __________.
    [A]challenges economists and politicians [B]takes efforts of generations
    [C] demands priority from the government [D] requires sufficient labor force
    33.A major difference between the Japanese and U.S workforces is that __________.
    [A] the Japanese workforce is better disciplined [B] the Japanese workforce is more productive
    [C]the U.S workforce has a better education [D] ]the U.S workforce is more organize
  30. The author quotes the example of our ancestors to show that education emerged __________.
    [A] when people had enough time [B] prior to better ways of finding food
    [C] when people on longer went hung [D] as a result of pressure on government
  31. According to the last paragraph , development of education __________.
    [A] results directly from competitive environments [B] does not depend on economic performance
    [C] follows improved productivity [D] cannot afford political changes
    Text 4
    The most thoroughly studied in the history of the new world are the ministers and political leaders of seventeenth-century New England. According to the standard history of American philosophy, nowhere else in colonial America was “So much important attached to intellectual pursuits ” According to many books and articles, New England’s leaders established the basic themes and preoccupations of an unfolding, dominant Puritan tradition in American intellectual life.
    To take this approach to the New Englanders normally mean to start with the Puritans’ theological innovations and their distinctive ideas about the church-important subjects that we may not neglect. But in keeping with our examination of southern intellectual life, we may consider the original Puritans as carriers of European culture adjusting to New world circumstances. The New England colonies were the scenes of important episodes in the pursuit of widely understood ideals of civility and virtuosity.
    The early settlers of Massachusetts Bay included men of impressive education and influence in England. `Besides the ninety or so learned ministers who came to Massachusetts church in the decade after 1629,There were political leaders like John Winthrop, an educated gentleman, lawyer, and official of the Crown before he journeyed to Boston. There men wrote and published extensively, reaching both New World and Old World audiences, and giving New England an atmosphere of intellectual earnestness.
    We should not forget , however, that most New Englanders were less well educated. While few crafts men or farmers, let alone dependents and servants, left literary compositions to be analyzed, The in thinking often had a traditional superstitions quality. A tailor named John Dane, who emigrated in the late 1630s, left an account of his reasons for leaving England that is filled with signs. sexual confusion, economic frustrations , and religious hope-all name together in a decisive moment when he opened the Bible, told his father the first line he saw would settle his fate, and read the magical words: “come out from among them, touch no unclean thing , and I will be your God and you shall be my people.” One wonders what Dane thought of the careful sermons explaining the Bible that he heard in puritan churched.
    Mean while , many settles had slighter religious commitments than Dane’s, as one clergyman learned in confronting folk along the coast who mocked that they had not come to the New world for religion . “Our main end was to catch fish. ”
  32. The author notes that in the seventeenth-century New England___________.
    [A] Puritan tradition dominated political life. [B] intellectual interests were encouraged.
    [C] Politics benefited much from intellectual endeavors.
    [D] intellectual pursuits enjoyed a liberal environment.
  33. It is suggested in paragraph 2 that New Englanders__________.
    [A] experienced a comparatively peaceful early history.
    [B] brought with them the culture of the Old World
    [C] paid little attention to southern intellectual life[D] were obsessed with religious innovations
  34. The early ministers and political leaders in Massachusetts Bay__________.
    [A] were famous in the New World for their writings
    [B] gained increasing importance in religious affairs
    [C] abandoned high positions before coming to the New World
    [D] created a new intellectual atmosphere in New England
  35. The story of John Dane shows that less well-educated New Englanders were often __________.
    [A] influenced by superstitions [B] troubled with religious beliefs
    [C] puzzled by church sermons [D] frustrated with family earnings
  36. The text suggests that early settlers in New England__________.
    [A] were mostly engaged in political activities[B] were motivated by an illusory prospect
    [C] came from different backgrounds. [D] left few formal records for later reference
    Part B
    Directions:
    Directions: In the following text, some sentences have been removed. For Questions (41-45), choose the most suitable one from the list A-G to fit into each of the numbered blank. There are two extra choices, which do not fit in any of the gaps. Mark your answers on ANSWER SHEET 1. (10 points)
    Coinciding with the groundbreaking theory of biological evolution proposed by British naturalist Charles Darwin in the 1860s, British social philosopher Herbert Spencer put forward his own theory of biological and cultural evolution. Spencer argued that all worldly phenomena, including human societies, changed over time, advancing toward perfection. 41.____________.
    American social scientist Lewis Henry Morgan introduced another theory of cultural evolution in the late 1800s. Morgan, along with Tylor, was one of the founders of modern anthropology. In his work, he attempted to show how all aspects of culture changed together in the evolution of societies.42._____________.
    In the early 1900s in North America, German-born American anthropologist Franz Boas developed a new theory of culture known as historical particularism. Historical particularism, which emphasized the uniqueness of all cultures, gave new direction to anthropology. 43._____________ .
    Boas felt that the culture of any society must be understood as the result of a unique history and not as one of many cultures belonging to a broader evolutionary stage or type of culture. 44._______________.
    Historical particularism became a dominant approach to the study of culture in American anthropology, largely through the influence of many students of Boas. But a number of anthropologists in the early 1900s also rejected the particularist theory of culture in favor of diffusionism. Some attributed virtually every important cultural achievement to the inventions of a few, especially gifted peoples that, according to diffusionists, then spread to other cultures. 45.________________.
    Also in the early 1900s, French sociologist Émile Durkheim developed a theory of culture that would greatly influence anthropology. Durkheim proposed that religious beliefs functioned to reinforce social solidarity. An interest in the relationship between the function of society and culture—known as functionalism—became a major theme in European, and especially British, anthropology.
    [A] Other anthropologists believed that cultural innovations, such as inventions, had a single origin and passed from society to society. This theory was known as diffusionism.
    [B] In order to study particular cultures as completely as possible, Boas became skilled in linguistics, the study of languages, and in physical anthropology, the study of human biology and anatomy.
    [C] He argued that human evolution was characterized by a struggle he called the “survival of the fittest,” in which weaker races and societies must eventually be replaced by stronger, more advanced races and societies.
    [D] They also focused on important rituals that appeared to preserve a people’s social structure, such as initiation ceremonies that formally signify children’s entrance into adulthood.
    [E] Thus, in his view, diverse aspects of culture, such as the structure of families, forms of marriage, categories of kinship, ownership of property, forms of government, technology, and systems of food production, all changed as societies evolved.
    [F]Supporters of the theory viewed as a collection of integrated parts that work together to keep a society functioning.
    [G] For example, British anthropologists Grafton Elliot Smith and W. J. Perry incorrectly suggested, on the basis of inadequate information, that farming, pottery making, and metallurgy all originated in ancient Egypt and diffused throughout the world. In fact, all of these cultural developments occurred separately at different times in many parts of the world.
    Part C
    Directions:
    Read the following text carefully and then translate the underlined segments into Chinese. Your translation should be written carefully on ANSWER SHEET 2. (10 points)
    There is a marked difference between the education which every one gets from living with others, and the deliberate educating of the young. In the former case the education is incidental; it is natural and important, but it is not the express reason of the association.46 It may be said that the measure of the worth of any social institution is its effect in enlarging and improving experience; but this effect is not a part of its original motive. Religious associations began, for example, in the desire to secure the favor of overruling powers and to ward off evil influences; family life in the desire to gratify appetites and secure family perpetuity; systematic labor, for the most part, because of enslavement to others, etc. 47Only gradually was the by-product of the institution noted, and only more gradually still was this effect considered as a directive factor in the conduct of the institution. Even today, in our industrial life, apart from certain values of industriousness and thrift, the intellectual and emotional reaction of the forms of human association under which the world's work is carried on receives little attention as compared with physical output.
    But in dealing with the young, the fact of association itself as an immediate human fact, gains in importance.48 While it is easy to ignore in our contact with them the effect of our acts upon their disposition, it is not so easy as in dealing with adults. The need of training is too evident; the pressure to accomplish a change in their attitude and habits is too urgent to leave these consequences wholly out of account. 49Since our chief business with them is to enable them to share in a common life we cannot help considering whether or no we are forming the powers which will secure this ability. If humanity has made some headway in realizing that the ultimate value of every institution is its distinctively human effect we may well believe that this lesson has been learned largely through dealings with the young.
    50 We are thus led to distinguish, within the broad educational process which we have been so far considering, a more formal kind of education -- that of direct tuition or schooling. In undeveloped social groups, we find very little formal teaching and training. These groups mainly rely for instilling needed dispositions into the young upon the same sort of association which keeps the adults loyal to their group.
    Section Ⅲ Writing
    Part A
  37. Directions: Restrictions on the use of plastic bags have not been so successful in some regions. “White pollution ”is still going on. Write a letter to the editor(s) of your local newspaper to
    give your opinions briefly and
    make two or three suggestions
    You should write about 100 words. Do not sign your own name at the end of the letter. Use "Li Ming" instead. You do not need to write the address.
    Part B
  38. Directions:
    In your essay, you should
  1. describe the drawing briefly,
  2. explain its intended meaning, and then
  3. give your comments.
    2008年全国硕士研究生入学统一考试英语试题
    Section I Use of English
    Directions:
    The idea that some groups of people may be more intelligent than others is one of those hypotheses that dare not speak its name. But Gregory Cochran is to say it anyway. He is that bird, a scientist who works independently any institution. He helped popularize the idea that some diseases not thought to have a bacterial cause were actually infections, which aroused much controversy when it was first suggested.
    he, however, might tremble at the of what he is about to do. Together with another two scientists, he is publishing a paper which not only that one group of humanity is more intelligent than the others, but explains the process that has brought this about. The group in are a particular people originated from central Europe. The process is natural selection.
    This group generally do well in IQ test, 12-15 points above the value of 100, and have contributed to the intellectual and cultural life of the West, as the of their elites, including several world-renowned scientists, . They also suffer more often than most people from a number of nasty genetic diseases, such as breast cancer. These facts, , have previously been thought unrelated. The former has been to social effects, such as a strong tradition of education. The latter was seen as a (an) of genetic isolation. Dr. Cochran suggests that the intelligence and diseases are intimately . His argument is that the unusual history of these people has them to unique evolutionary pressures that have resulted in this state of affairs.
  1. [A] selected [B] prepared [C] obliged [D] pleased
  2. [A] unique [B] particular [C] special [D] rare
  3. [A] of [B] with [C] in [D] against
  4. [A] subsequently[B] presently [C] previously [D] lately
  5. [A] Only [B] So [C] Even [D] Hence
  6. [A] thought [B] sight [C] cost [D] risk
  7. [A] advises [B] suggests [C] protests [D] objects
  8. [A] progress [B] fact [C] need [D] question
  9. [A] attaining [B] scoring [C] reaching [D] calculating
  10. [A] normal [B] common [C] mean [D] total
  11. [A] unconsciously [B] disproportionately
    [C] indefinitely [D] unaccountably
  12. [A] missions [B] fortunes [C] interests [D] careers
  13. [A] affirm [B] witness [C] observe [D] approve
  14. [A] moreover [B] therefore [C] however [D] meanwhile
  15. [A] given up [B] got over [C] carried on [D] put down
  16. [A] assessing [B] supervising [C] administering [D] valuing
  17. [A] development [B] origin [C] consequence [D] instrument
  18. [A] linked [B] integrated [C] woven [D] combined
  19. [A] limited [B] subjected [C] converted [D] directed
  20. [A] paradoxical [B] incompatible [C] inevitable [D] continuous
    Section II Reading Comprehension
    Part A
    Directions:
    Read the following four texts. Answer the questions below each text by choosing A, B, C or D. Mark your answers on ANSWER SHEET 1. (40 points)
    Text 1
    While still catching-up to men in some spheres of modern life, women appear to be way ahead in at least one undesirable category. “Women are particularly susceptible to developing depression and anxiety disorders in response to stress compared to men,” according to Dr. Yehuda, chief psychiatrist at New York’s Veteran’s Administration Hospital.
    Studies of both animals and humans have shown that sex hormones somehow affect the stress response, causing females under stress to produce more of the trigger chemicals than do males under the same conditions. In several of the studies, when stressed-out female rats had their ovaries (the female reproductive organs) removed, their chemical responses became equal to those of the males.
    Adding to a woman’s increased dose of stress chemicals, are her increased “opportunities” for stress. “It’s not necessarily that women don’t cope as well. It’s just that they have so much more to cope with,” says Dr. Yehuda. “Their capacity for tolerating stress may even be greater than men’s,” she observes, “it’s just that they’re dealing with so many more things that they become worn out from it more visibly and sooner.”
    Dr. Yehuda notes another difference between the sexes. “I think that the kinds of things that women are exposed to tend to be in more of a chronic or repeated nature. Men go to war and are exposed to combat stress. Men are exposed to more acts of random physical violence. The kinds of interpersonal violence that women are exposed to tend to be in domestic situations, by, unfortunately, parents or other family members, and they tend not to be one-shot deals. The wear-and-tear that comes from these longer relationships can be quite devastating.”
    Adeline Alvarez married at 18 and gave birth to a son, but was determined to finish college. “I struggled a lot to get the college degree. I was living in so much frustration that that was my escape, to go to school, and get ahead and do better.” Later, her marriage ended and she became a single mother. “It’s the hardest thing to take care of a teenager, have a job, pay the rent, pay the car payment, and pay the debt. I lived from paycheck to paycheck.”
    Not everyone experiences the kinds of severe chronic stresses Alvarez describes. But most women today are coping with a lot of obligations, with few breaks, and feeling the strain. Alvarez’s experience demonstrates the importance of finding ways to diffuse stress before it threatens your health and your ability to function.
  21. Which of the following is true according to the first two paragraphs?
    [A] Women are biologically more vulnerable to stress.
    [B] Women are still suffering much stress caused by men.
    [C] Women are more experienced than men in coping with stress.
    [D] Men and women show different inclinations when faced with stress.
  22. Dr. Yehuda’s research suggests that women
    [A] need extra doses of chemicals to handle stress.
    [B] have limited capacity for tolerating stress.
    [C] are more capable of avoiding stress. [D] are exposed to more stress.
  23. According to Paragraph 4, the stress women confront tends to be
    [A] domestic and temporary. [B] irregular and violent.
    [C] durable and frequent. [D] trivial and random.
  24. The sentence “I lived from paycheck to paycheck.” (Line 6, Para. 5) shows that
    [A] Alvarez cared about nothing but making money.
    [B] Alvarez’s salary barely covered her household expenses.
    [C] Alvarez got paychecks from different jobs.
    [D] Alvarez paid practically everything by check.
  25. Which of the following would be the best title for the text?
    [A] Strain of Stress: No Way Out? [B] Responses to Stress: Gender Difference
    [C] Stress Analysis: What Chemicals Say [D] Gender Inequality: Women Under Stress
    Text 2
    It used to be so straightforward. A team of researchers working together in the laboratory would submit the results of their research to a journal. A journal editor would then remove the authors’ names and affiliations from the paper and send it to their peers for review. Depending on the comments received, the editor would accept the paper for publication or decline it. Copyright rested with the journal publisher, and researchers seeking knowledge of the results would have to subscribe to the journal.
    No longer. The Internet – and pressure from funding agencies, who are questioning why commercial publishers are making money from government-funded research by restricting access to it – is making access to scientific results a reality. The Organization for Economic Co-operation and Development (OECD) has just issued a report describing the far-reaching consequences of this. The report, by John Houghton of Victoria University in Australia and Graham Vickery of the OECD, makes heavy reading for publishers who have, so far, made handsome profits. But it goes further than that. It signals a change in what has, until now, been a key element of scientific endeavor.
    The value of knowledge and the return on the public investment in research depends, in part, upon wide distribution and ready access. It is big business. In America, the core scientific publishing market is estimated at between $7 billion and $11 billion. The International Association of Scientific, Technical and Medical Publishers says that there are more than 2,000 publishers worldwide specializing in these subjects. They publish more than 1.2 million articles each year in some 16,000 journals.
    This is now changing. According to the OECD report, some 75% of scholarly journals are now online. Entirely new business models are emerging; three main ones were identified by the report’s authors. There is the so-called big deal, where institutional subscribers pay for access to a collection of online journal titles through site-licensing agreements. There is open-access publishing, typically supported by asking the author (or his employer) to pay for the paper to be published. Finally, there are open-access archives, where organizations such as universities or international laboratories support institutional repositories. Other models exist that are hybrids of these three, such as delayed open-access, where journals allow only subscribers to read a paper for the first six months, before making it freely available to everyone who wishes to see it. All this could change the traditional form of the peer-review process, at least for the publication of papers.
  26. In the first paragraph, the author discusses
    [A] the background information of journal editing.[B] the publication routine of laboratory reports.
    [C] the relations of authors with journal publishers.[D] the traditional process of journal publication.
  27. Which of the following is true of the OECD report?
    [A] It criticizes government-funded research.[B] It introduces an effective means of publication.
    [C] It upsets profit-making journal publishers.[D] It benefits scientific research considerably.
  28. According to the text, online publication is significant in that
    [A] it provides an easier access to scientific results.[B] it brings huge profits to scientific researchers.
    [C] it emphasizes the crucial role of scientific knowledge.
    [D] it facilitates public investment in scientific research.
  29. With the open-access publishing model, the author of a paper is required to
    [A] cover the cost of its publication.[B] subscribe to the journal publishing it.
    [C] allow other online journals to use it freely.[D] complete the peer-review before submission.
  30. Which of the following best summarizes the main idea of the text?
    [A] The Internet is posing a threat to publishers.[B] A new mode of publication is emerging.
    [C] Authors welcome the new channel for publication.
    [D] Publication is rendered easier by online service.
    Text 3
    In the early 1960s Wilt Chamberlain was one of only three players in the National Basketball Association (NBA) listed at over seven feet. If he had played last season, however, he would have been one of 42. The bodies playing major professional sports have changed dramatically over the years, and managers have been more than willing to adjust team uniforms to fit the growing numbers of bigger, longer frames.
    The trend in sports, though, may be obscuring an unrecognized reality: Americans have generally stopped growing. Though typically about two inches taller now than 140 years ago, today’s people – especially those born to families who have lived in the U.S. for many generations – apparently reached their limit in the early 1960s. And they aren’t likely to get any taller. “In the general population today, at this genetic, environmental level, we’ve pretty much gone as far as we can go,” says anthropologist William Cameron Chumlea of Wright State University. In the case of NBA players, their increase in height appears to result from the increasingly common practice of recruiting players from all over the world.
    Growth, which rarely continues beyond the age of 20, demands calories and nutrients – notably, protein – to feed expanding tissues. At the start of the 20th century, under-nutrition and childhood infections got in the way. But as diet and health improved, children and adolescents have, on average, increased in height by about an inch and a half every 20 years, a pattern known as the secular trend in height. Yet according to the Centers for Disease Control and Prevention, average height – 5′9″ for men, 5′4″ for women – hasn’t really changed since 1960.
    Genetically speaking, there are advantages to avoiding substantial height. During childbirth, larger babies have more difficulty passing through the birth canal. Moreover, even though humans have been upright for millions of years, our feet and back continue to struggle with bipedal posture and cannot easily withstand repeated strain imposed by oversize limbs. “There are some real constraints that are set by the genetic architecture of the individual organism,” says anthropologist William Leonard of Northwestern University.
    Genetic maximums can change, but don’t expect this to happen soon. Claire C. Gordon, senior anthropologist at the Army Research Center in Natick, Mass., ensures that 90 percent of the uniforms and workstations fit recruits without alteration. She says that, unlike those for basketball, the length of military uniforms has not changed for some time. And if you need to predict human height in the near future to design a piece of equipment, Gordon says that by and large, “you could use today’s data and feel fairly confident.”
  31. Wilt Chamberlain is cited as an example to
    [A] illustrate the change of height of NBA players.
    [B] show the popularity of NBA players in the U.S..
    [C] compare different generations of NBA players.
    [D] assess the achievements of famous NBA players.
  32. Which of the following plays a key role in body growth according to the text?
    [A] Genetic modification. [B] Natural environment.
    [C] Living standards. [D] Daily exercise.
  33. On which of the following statements would the author most probably agree?
    [A] Non-Americans add to the average height of the nation.
    [B] Human height is conditioned by the upright posture.
    [C] Americans are the tallest on average in the world.
    [D] Larger babies tend to become taller in adulthood.
  34. We learn from the last paragraph that in the near future
    [A] the garment industry will reconsider the uniform size.
    [B] the design of military uniforms will remain unchanged.
    [C] genetic testing will be employed in selecting sportsmen.
    [D] the existing data of human height will still be applicable.
  35. The text intends to tell us that
    [A] the change of human height follows a cyclic pattern.
    [B] human height is becoming even more predictable.
    [C] Americans have reached their genetic growth limit.
    [D] the genetic pattern of Americans has altered.
    Text 4
    In 1784, five years before he became president of the United States, George Washington, 52, was nearly toothless. So he hired a dentist to transplant nine teeth into his jaw – having extracted them from the mouths of his slaves.
    That’s a far different image from the cherry-tree-chopping George most people remember from their history books. But recently, many historians have begun to focus on the roles slavery played in the lives of the founding generation. They have been spurred in part by DNA evidence made available in 1998, which almost certainly proved Thomas Jefferson had fathered at least one child with his slave Sally Hemings. And only over the past 30 years have scholars examined history from the bottom up. Works of several historians reveal the moral compromises made by the nation’s early leaders and the fragile nature of the country’s infancy. More significantly, they argue that many of the Founding Fathers knew slavery was wrong – and yet most did little to fight it.
    More than anything, the historians say, the founders were hampered by the culture of their time. While Washington and Jefferson privately expressed distaste for slavery, they also understood that it was part of the political and economic bedrock of the country they helped to create.
    For one thing, the South could not afford to part with its slaves. Owning slaves was “like having a large bank account,” says Wiencek, author of An Imperfect God: George Washington, His Slaves, and the Creation of America. The southern states would not have signed the Constitution without protections for the “peculiar institution,” including a clause that counted a slave as three fifths of a man for purposes of congressional representation.
    And the statesmen’s political lives depended on slavery. The three-fifths formula handed Jefferson his narrow victory in the presidential election of 1800 by inflating the votes of the southern states in the Electoral College. Once in office, Jefferson extended slavery with the Louisiana Purchase in 1803; the new land was carved into 13 states, including three slave states.
    Still, Jefferson freed Hemings’s children – though not Hemings herself or his approximately 150 other slaves. Washington, who had begun to believe that all men were created equal after observing the bravery of the black soldiers during the Revolutionary War, overcame the strong opposition of his relatives to grant his slaves their freedom in his will. Only a decade earlier, such an act would have required legislative approval in Virginia.
  36. George Washington’s dental surgery is mentioned to
    [A] show the primitive medical practice in the past.
    [B] demonstrate the cruelty of slavery in his days.
    [C] stress the role of slaves in the U.S. history.[D] reveal some unknown aspect of his life.
  37. We may infer from the second paragraph that
    [A] DNA technology has been widely applied to history research.
    [B] in its early days the U.S. was confronted with delicate situations.
    [C] historians deliberately made up some stories of Jefferson’s life.
    [D] political compromises are easily found throughout the U.S. history.
  38. What do we learn about Thomas Jefferson?
    [A] His political view changed his attitude towards slavery.
    [B] His status as a father made him free the child slaves.
    [C] His attitude towards slavery was complex.[D] His affair with a slave stained his prestige.
  39. Which of the following is true according to the text?
    [A] Some Founding Fathers benefit politically from slavery.
    [B] Slaves in the old days did not have the right to vote.
    [C] Slave owners usually had large savings accounts.
    [D] Slavery was regarded as a peculiar institution.
  40. Washington’s decision to free slaves originated from his
    [A] moral considerations. [B] military experience.[C] financial conditions. [D] political stand.
    Part B
    Directions:
    In the following article, some sentences have been removed. For Questions 41—45, choose the most suitable one from the list A-G to fit into each of the numbered blanks. There are two extra choices, which do not fit in any of the blanks. Mark your answers on ANSWER SHEET 1. (10 points)
    The time for sharpening pencils, arranging your desk, and doing almost anything else instead of writing has ended. The first draft will appear on the page only if you stop avoiding the inevitable and sit, stand up, or lie down to write. (41)是大家网原创出品
    Be flexible. Your outline should smoothly conduct you from one point to the next, but do not permit it to railroad you. If a relevant and important idea occurs to you now, work it into the draft. (42) 是大家网原创出品 Grammar, punctuation, and spelling can wait until you revise. Concentrate on what you are saying. Good writing most often occurs when you are in hot pursuit of an idea rather than in a nervous search for errors.
    (43) 是大家网原创出品 Your pages will be easier to keep track of that way, and, if you have to clip a paragraph to place it elsewhere, you will not lose any writing on the other side.
    If you are working on a word processor, you can take advantage of its capacity to make additions and deletions as well as move entire paragraphs by making just a few simple keyboard commands. Some software programs can also check spelling and certain grammatical elements in your writing. (44) 是大家网原创出品 These printouts are also easier to read than the screen when you work on revisions.
    Once you have a first draft on paper, you can delete material that is unrelated to your thesis and add material necessary to illustrate your points and make your paper convincing. The student who wrote “The A & P as a State of Mind” wisely dropped a paragraph that questioned whether Sammy displays chauvinistic attitudes toward women. (45) 是大家网原创出品
    Remember that your initial draft is only that. You should go through the paper many times – and then again – working to substantiate and clarify your ideas. You may even end up with several entire versions of the paper. Rewrite. The sentences within each paragraph should be related to a single topic. Transitions should connect one paragraph to the next so that there are no abrupt or confusing shifts. Awkward or wordy phrasing or unclear sentences and paragraphs should be mercilessly poked and prodded into shape.
    [A] To make revising easier, leave wide margins and extra space between lines so that you can easily add words, sentences, and corrections. Write on only one side of the paper.
    [B] After you have clearly and adequately developed the body of your paper, pay particular attention to the introductory and concluding paragraphs. It’s probably best to write the introduction last, after you know precisely what you are introducing. Concluding paragraphs demand equal attention because they leave the reader with a final impression.
    [C] It’s worth remembering, however, that though a clean copy fresh off a printer may look terrific, it will read only as well as the thinking and writing that have gone into it. Many writers prudently store their data on disks and print their pages each time they finish a draft to avoid losing any material because of power failures or other problems.
    [D] It makes no difference how you write, just so you do. Now that you have developed a topic into a tentative thesis, you can assemble your notes and begin to flesh out whatever outline you have made.
    [E] Although this is an interesting issue, it has nothing to do with the thesis, which explains how the setting influences Sammy’s decision to quit his job. Instead of including that paragraph, she added one that described Lengel’s crabbed response to the girls so that she could lead up to the A & P “policy” he enforces.
    [F] In the final paragraph about the significance of the setting in “A & P,” the student brings together the reasons Sammy quit his job by referring to his refusal to accept Lengel’s store policies.
    [G] By using the first draft as a means of thinking about what you want to say, you will very likely discover more than your notes originally suggested. Plenty of good writers don’t use outlines at all but discover ordering principles as they write. Do not attempt to compose a perfectly correct draft the first time around.
    Part C
    Directions:
    Read the following text carefully and then translate the underlined segments into Chinese. Your translation should be written clearly on ANSWER SHEET 2. (10 points)
    In his autobiography, Darwin himself speaks of his intellectual powers with extraordinary modesty. He points out that he always experienced much difficulty in expressing himself clearly and concisely, but (46) he believes that this very difficulty may have had the compensating advantage of forcing him to think long and intently about every sentence, and thus enabling him to detect errors in reasoning and in his own observations. He disclaimed the possession of any great quickness of apprehension or wit, such as distinguished Huxley. (47) He asserted, also, that his power to follow a long and purely abstract train of thought was very limited, for which reason he felt certain that he never could have succeeded with mathematics. His memory, too, he described as extensive, but hazy. So poor in one sense was it that he never could remember for more than a few days a single date or a line of poetry. (48) On the other hand, he did not accept as well founded the charge made by some of his critics that, while he was a good observer, he had no power of reasoning. This, he thought, could not be true, because the “Origin of Species” is one long argument from the beginning to the end, and has convinced many able men. No one, he submits, could have written it without possessing some power of reasoning. He was willing to assert that “I have a fair share of invention, and of common sense or judgment, such as every fairly successful lawyer or doctor must have, but not, I believe, in any higher degree.” (49) He adds humbly that perhaps he was “superior to the common run of men in noticing things which easily escape attention, and in observing them carefully.”
    Writing in the last year of his life, he expressed the opinion that in two or three respects his mind had changed during the preceding twenty or thirty years. Up to the age of thirty or beyond it poetry of many kinds gave him great pleasure. Formerly, too, pictures had given him considerable, and music very great, delight. In 1881, however, he said: “Now for many years I cannot endure to read a line of poetry. I have also almost lost my taste for pictures or music.” (50) Darwin was convinced that the loss of these tastes was not only a loss of happiness, but might possibly be injurious to the intellect, and more probably to the moral character.
    Section III Writing
    Part A
  41. Directions:
    You have just come back from Canada and found a music CD in your luggage that you forgot to return to Bob, your landlord there. Write him a letter to
  1. make an apology, and
  2. suggest a solution.
    You should write about 100 words on ANSWER SHEET 2.
    Do not sign your own name at the end of the letter. Use “Li Ming” instead.
    Do not write the address. (10 points)
    Part B
  1. Directions:
    Write an essay of 160-200 words based on the following drawing. In your essay, you should
  1. describe the drawing briefly,
  2. explain its intended meaning, and then
  3. give your comments.
    2007年全国硕士研究生入学统一考试英语试题
    Section I Use of English
    Directions:
    Read the following text. Choose the best word(s) for each numbered blank and mark [A], [B], [C] or [D] on ANSWER SHEET 1. (10 points)
    By 1830 the former Spanish and Portuguese colonies had become independent nations. The roughly 20 million of these nations looked to the future. Born in the crisis of the old regime and Iberian Colonialism, many of the leaders of independence the ideals of representative government, careers to talent, freedom of commerce and trade, the to private property, and a belief in the individual as the basis of society. there was a belief that the new nations should be sovereign and independent states, large enough to be economically viable and integrated by a set of laws.
    On the issue of of religion and the position of the church, , there was less agreement the leadership. Roman Catholicism had been the state religion and the only one by the Spanish crown. most leaders sought to maintain Catholicism the official religion of the new states, some sought to end the of other faiths. The defense of the Church became a rallying for the conservative forces.
    The ideals of the early leaders of independence were often egalitarian, valuing equality of everything. Bolivar had received aid from Haiti and had in return to abolish slavery in the areas he liberated. By 1854 slavery had been abolished everywhere except Spain’s colonies. Early promises to end Indian tribute and taxes on people of mixed origin came much because the new nations still needed the revenue such policies . Egalitarian sentiments were often tempered by fears that the mass of the population was self-rule and democracy.
  1. [A] natives [B] inhabitants [C] peoples [D] individuals
  2. [A] confusedly [B] cheerfully [C] worriedly [D] hopefully
  3. [A] shared [B] forgot [C] attained [D] rejected
  4. [A] related [B] close [C] open [D] devoted
  5. [A] access [B] succession [C] right [D] return
  6. [A] Presumably [B] Incidentally [C] Obviously [D] Generally
  7. [A] unique [B] common [C] particular [D] typical
  8. [A] freedom [B] origin [C] impact [D] reform
  9. [A] therefore [B] however [C] indeed [D] moreover
  10. [A] with [B] about [C] among [D] by
  11. [A] allowed [B] preached [C] granted [D] funded
  12. [A] Since [B] If [C] Unless [D] While
  13. [A] as [B] for [C] under [D] against
  14. [A] spread [B] interference [C] exclusion [D] influence
  15. [A] support [B] cry [C] plea [D] wish
  16. [A] urged [B] intended [C] expected [D] promised
  17. [A] controlling [B] former [C] remaining [D] original
  18. [A] slower [B] faster [C] easier [D] tougher
  19. [A] created [B] produced [C] contributed [D] preferred
  20. [A] puzzled by[B] hostile to[C] pessimistic about[D] unprepared for
    Section II Reading Comprehension
    Part A
    Directions:
    Read the following four texts. Answer the questions below each text by choosing [A], [B], [C], or [D]. Mark your answers on ANSWER SHEET 1. (40 points)
    Text 1
    If you were to examine the birth certificates of every soccer player in 2006’s World Cup tournament, you would most likely find a noteworthy quirk: elite soccer players are more likely to have been born in the earlier months of the year than in the later months. If you then examined the European national youth teams that feed the World Cup and professional ranks, you would find this strange phenomenon to be even more pronounced.
    What might account for this strange phenomenon? Here are a few guesses: a) certain astrological signs confer superior soccer skills; b) winter-born babies tend to have higher oxygen capacity, which increases soccer stamina; c) soccer-mad parents are more likely to conceive children in springtime, at the annual peak of soccer mania; d) none of the above.
    Anders Ericsson, a 58-year-old psychology professor at Florida State University, says he believes strongly in “none of the above.” Ericsson grew up in Sweden, and studied nuclear engineering until he realized he would have more opportunity to conduct his own research if he switched to psychology. His first experiment, nearly 30 years ago, involved memory: training a person to hear and then repeat a random series of numbers. “With the first subject, after about 20 hours of training, his digit span had risen from 7 to 20,” Ericsson recalls. “He kept improving, and after about 200 hours of training he had risen to over 80 numbers.”
    This success, coupled with later research showing that memory itself is not genetically determined, led Ericsson to conclude that the act of memorizing is more of a cognitive exercise than an intuitive one. In other words, whatever inborn differences two people may exhibit in their abilities to memorize, those differences are swamped by how well each person “encodes” the information. And the best way to learn how to encode information meaningfully, Ericsson determined, was a process known as deliberate practice. Deliberate practice entails more than simply repeating a task. Rather, it involves setting specific goals, obtaining immediate feedback and concentrating as much on technique as on outcome.
    Ericsson and his colleagues have thus taken to studying expert performers in a wide range of pursuits, including soccer. They gather all the data they can, not just performance statistics and biographical details but also the results of their own laboratory experiments with high achievers. Their work makes a rather startling assertion: the trait we commonly call talent is highly overrated. Or, put another way, expert performers – whether in memory or surgery, ballet or computer programming – are nearly always made, not born.
  21. The birthday phenomenon found among soccer players is mentioned to
    [A] stress the importance of professional training.
    [B] spotlight the soccer superstars in the World Cup.
    [C] introduce the topic of what makes expert performance.
    [D] explain why some soccer teams play better than others.
  22. The word “mania” (Line 4, Paragraph 2) most probably means
    [A] fun. [B] craze. [C] hysteria. [D] excitement.
  23. According to Ericsson, good memory
    [A] depends on meaningful processing of information.
    [B] results from intuitive rather than cognitive exercises.
    [C] is determined by genetic rather than psychological factors.
    [D] requires immediate feedback and a high degree of concentration.
  24. Ericsson and his colleagues believe that
    [A] talent is a dominating factor for professional success.
    [B] biographical data provide the key to excellent performance.
    [C] the role of talent tends to be overlooked.
    [D] high achievers owe their success mostly to nurture.
  25. Which of the following proverbs is closest to the message the text tries to convey?
    [A] “Faith will move mountains.” [B] “One reaps what one sows.”
    [C] “Practice makes perfect.” [D] “Like father, like son.”
    Text 2
    For the past several years, the Sunday newspaper supplement Parade has featured a column called “Ask Marilyn.” People are invited to query Marilyn vos Savant, who at age 10 had tested at a mental level of someone about 23 years old; that gave her an IQ of 228 – the highest score ever recorded. IQ tests ask you to complete verbal and visual analogies, to envision paper after it has been folded and cut, and to deduce numerical sequences, among other similar tasks. So it is a bit confusing when vos Savant fields such queries from the average Joe (whose IQ is 100) as, What’s the difference between love and fondness? Or what is the nature of luck and coincidence? It’s not obvious how the capacity to visualize objects and to figure out numerical patterns suits one to answer questions that have eluded some of the best poets and philosophers.
    Clearly, intelligence encompasses more than a score on a test. Just what does it mean to be smart? How much of intelligence can be specified, and how much can we learn about it from neurology, genetics, computer science and other fields?
    The defining term of intelligence in humans still seems to be the IQ score, even though IQ tests are not given as often as they used to be. The test comes primarily in two forms: the Stanford-Binet Intelligence Scale and the Wechsler Intelligence Scales (both come in adult and children’s version). Generally costing several hundred dollars, they are usually given only by psychologists, although variations of them populate bookstores and the World Wide Web. Superhigh scores like vos Savant’s are no longer possible, because scoring is now based on a statistical population distribution among age peers, rather than simply dividing the mental age by the chronological age and multiplying by 100. Other standardized tests, such as the Scholastic Assessment Test (SAT) and the Graduate Record Exam (GRE), capture the main aspects of IQ tests.
    Such standardized tests may not assess all the important elements necessary to succeed in school and in life, argues Robert J. Sternberg. In his article “How Intelligent Is Intelligence Testing?”, Sternberg notes that traditional test best assess analytical and verbal skills but fail to measure creativity and practical knowledge, components also critical to problem solving and life success. Moreover, IQ tests do not necessarily predict so well once populations or situations change. Research has found that IQ predicted leadership skills when the tests were given under low-stress conditions, but under high-stress conditions, IQ was negatively correlated with leadership – that is, it predicted the opposite. Anyone who has toiled through SAT will testify that test-taking skill also matters, whether it’s knowing when to guess or what questions to skip.
  26. Which of the following may be required in an intelligence test?
    [A] Answering philosophical questions.
    [B] Folding or cutting paper into different shapes.
    [C] Telling the differences between certain concepts.
    [D] Choosing words or graphs similar to the given ones.
  27. What can be inferred about intelligence testing from Paragraph 3?
    [A] People no longer use IQ scores as an indicator of intelligence.
    [B] More versions of IQ tests are now available on the Internet.
    [C] The test contents and formats for adults and children may be different.
    [D] Scientists have defined the important elements of human intelligence.
  28. People nowadays can no longer achieve IQ scores as high as vos Savant’s because
    [A] the scores are obtained through different computational procedures.
    [B] creativity rather than analytical skills is emphasized now.
    [C] vos Savant’s case is an extreme one that will not repeat.
    [D] the defining characteristic of IQ tests has changed.
  29. We can conclude from the last paragraph that
    [A] test scores may not be reliable indicators of one’s ability.
    [B] IQ scores and SAT results are highly correlated.
    [C] testing involves a lot of guesswork.
    [D] traditional test are out of date.
  30. What is the author’s attitude towards IQ tests?
    [A] Supportive. [B] Skeptical. [C] Impartial. [D] Biased.
    Text 3
    During the past generation, the American middle-class family that once could count on hard work and fair play to keep itself financially secure had been transformed by economic risk and new realities. Now a pink slip, a bad diagnosis, or a disappearing spouse can reduce a family from solidly middle class to newly poor in a few months.
    In just one generation, millions of mothers have gone to work, transforming basic family economics. Scholars, policymakers, and critics of all stripes have debated the social implications of these changes, but few have looked at the side effect: family risk has risen as well. Today’s families have budgeted to the limits of their new two-paycheck status. As a result, they have lost the parachute they once had in times of financial setback – a back-up earner (usually Mom) who could go into the workforce if the primary earner got laid off or fell sick. This “added-worker effect” could support the safety net offered by unemployment insurance or disability insurance to help families weather bad times. But today, a disruption to family fortunes can no longer be made up with extra income from an otherwise-stay-at-home partner.
    During the same period, families have been asked to absorb much more risk in their retirement income. Steelworkers, airline employees, and now those in the auto industry are joining millions of families who must worry about interest rates, stock market fluctuation, and the harsh reality that they may outlive their retirement money. For much of the past year, President Bush campaigned to move Social Security to a saving-account model, with retirees trading much or all of their guaranteed payments for payments depending on investment returns. For younger families, the picture is not any better. Both the absolute cost of healthcare and the share of it borne by families have risen – and newly fashionable health-savings plans are spreading from legislative halls to Wal-Mart workers, with much higher deductibles and a large new dose of investment risk for families’ future healthcare. Even demographics are working against the middle class family, as the odds of having a weak elderly parent – and all the attendant need for physical and financial assistance – have jumped eightfold in just one generation.
    From the middle-class family perspective, much of this, understandably, looks far less like an opportunity to exercise more financial responsibility, and a good deal more like a frightening acceleration of the wholesale shift of financial risk onto their already overburdened shoulders. The financial fallout has begun, and the political fallout may not be far behind.
  31. Today’s double-income families are at greater financial risk in that
    [A] the safety net they used to enjoy has disappeared.
    [B] their chances of being laid off have greatly increased.
    [C] they are more vulnerable to changes in family economics.
    [D] they are deprived of unemployment or disability insurance.
  32. As a result of President Bush’s reform, retired people may have
    [A] a higher sense of security. [B] less secured payments.
    [C] less chance to invest. [D] a guaranteed future.
  33. According to the author, health-savings plans will
    [A] help reduce the cost of healthcare. [B] popularize among the middle class.
    [C] compensate for the reduced pensions. [D] increase the families’ investment risk.
  34. It can be inferred from the last paragraph that
    [A] financial risks tend to outweigh political risks.
    [B] the middle class may face greater political challenges.
    [C] financial problems may bring about political problems.
    [D] financial responsibility is an indicator of political status.
  35. Which of the following is the best title for this text?
    [A] The Middle Class on the Alert [B] The Middle Class on the Cliff
    [C] The Middle Class in Conflict [D] The Middle Class in Ruins
    Text 4
    It never rains but it pours. Just as bosses and boards have finally sorted out their worst accounting and compliance troubles, and improved their feeble corporation governance, a new problem threatens to earn them – especially in America – the sort of nasty headlines that inevitably lead to heads rolling in the executive suite: data insecurity. Left, until now, to odd, low-level IT staff to put right, and seen as a concern only of data-rich industries such as banking, telecoms and air travel, information protection is now high on the boss’s agenda in businesses of every variety.
    Several massive leakages of customer and employee data this year – from organizations as diverse as Time Warner, the American defense contractor Science Applications International Corp and even the University of California, Berkeley – have left managers hurriedly peering into their intricate IT systems and business processes in search of potential vulnerabilities.
    “Data is becoming an asset which needs to be guarded as much as any other asset,” says Haim Mendelson of Stanford University’s business school. “The ability to guard customer data is the key to market value, which the board is responsible for on behalf of shareholders.” Indeed, just as there is the concept of Generally Accepted Accounting Principles (GAAP), perhaps it is time for GASP, Generally Accepted Security Practices, suggested Eli Noam of New York’s Columbia Business School. “Setting the proper investment level for security, redundancy, and recovery is a management issue, not a technical one,” he says.
    The mystery is that this should come as a surprise to any boss. Surely it should be obvious to the dimmest executive that trust, that most valuable of economic assets, is easily destroyed and hugely expensive to restore – and that few things are more likely to destroy trust than a company letting sensitive personal data get into the wrong hands.
    The current state of affairs may have been encouraged – though not justified – by the lack of legal penalty (in America, but not Europe) for data leakage. Until California recently passed a law, American firms did not have to tell anyone, even the victim, when data went astray. That may change fast: lots of proposed data-security legislation is now doing the rounds in Washington, D.C. Meanwhile, the theft of information about some 40 million credit-card accounts in America, disclosed on June 17th, overshadowed a hugely important decision a day earlier by America’s Federal Trade Commission (FTC) that puts corporate America on notice that regulators will act if firms fail to provide adequate data security.
  36. The statement “It never rains but it pours” is used to introduce
    [A] the fierce business competition. [B] the feeble boss-board relations.
    [C] the threat from news reports. [D] the severity of data leakage.
  37. According to Paragraph 2, some organizations check their systems to find out
    [A] whether there is any weak point.[B] what sort of data has been stolen.
    [C] who is responsible for the leakage. [D] how the potential spies can be located.
  38. In bringing up the concept of GASP the author is making the point that
    [A] shareholders’ interests should be properly attended to.
    [B] information protection should be given due attention.
    [C] businesses should enhance their level of accounting security.
    [D] the market value of customer data should be emphasized.
  39. According to Paragraph 4, what puzzles the author is that some bosses fail to
    [A] see the link between trust and data protection.[B] perceive the sensitivity of personal data.
    [C] realize the high cost of data restoration.[D] appreciate the economic value of trust.
  40. It can be inferred from Paragraph 5 that
    [A] data leakage is more severe in Europe.[B] FTC’s decision is essential to data security.
    [C] California takes the lead in security legislation.
    [D] legal penalty is a major solution to data leakage.
    Part B
    Directions:
    You are going to read a list of headings and a text about what parents are supposed to do to guide their children into adulthood. Choose a heading from the list A—G that best fits the meaning of each numbered part of the text (41-45). The first and last paragraphs of the text are not numbered. There are two extra headings that you do not need to use. Mark your answers on ANSWER SHEET 1. (10 points)
    A. Set a Good Example for Your KidsB. Build Your Kids’ Work Skills
    C. Place Time Limits on Leisure ActivitiesD. Talk about the Future on a Regular Basis
    E. Help Kids Develop Coping StrategiesF. Help Your Kids Figure Out Who They Are
    G. Build Your Kids’ Sense of Responsibility
    How Can a Parent Help?
    Mothers and fathers can do a lot to ensure a safe landing in early adulthood for their kids. Even if a job’s starting salary seems too small to satisfy an emerging adult’s need for rapid content, the transition from school to work can be less of a setback if the start-up adult is ready for the move. Here are a few measures, drawn from my book Ready or Not, Here Life Comes, that parents can take to prevent what I call “work-life unreadiness.”
    You can start this process when they are 11 or 12. Periodically review their emerging strengths and weaknesses with them and work together on any shortcomings, like difficulty in communicating well or collaborating. Also, identify the kinds of interests they keep coming back to, as these offer clues to the careers that will fit them best.
    Kids need a range of authentic role models – as opposed to members of their clique, pop stars and vaunted athletes. Have regular dinner-table discussions about people the family knows and how they got where they are. Discuss the joys and downsides of your own career and encourage your kids to form some ideas about their own future. When asked what they want to do, they should be discouraged from saying “I have no idea.” They can change their minds 200 times, but having only a foggy view of the future is of little good.
    Teachers are responsible for teaching kids how to learn; parents should be responsible for teaching them how to work. Assign responsibilities around the house and make sure homework deadlines are met. Encourage teenagers to take a part-time job. Kids need plenty of practice delaying gratification and deploying effective organizational skills, such as managing time and setting priorities.
    Playing video games encourages immediate content. And hours of watching TV shows with canned laughter only teaches kids to process information in a passive way. At the same time, listening through earphones to the same monotonous beats for long stretches encourages kids to stay inside their bubble instead of pursuing other endeavors. All these activities can prevent the growth of important communication and thinking skills and make it difficult for kids to develop the kind of sustained concentration they will need for most jobs.
    They should know how to deal with setbacks, stresses and feelings of inadequacy. They should also learn how to solve problems and resolve conflicts, ways to brainstorm and think critically. Discussions at home can help kids practice doing these things and help them apply these skills to everyday life situations.
    What about the son or daughter who is grown but seems to be struggling and wandering aimlessly through early adulthood? Parents still have a major role to play, but now it is more delicate. They have to be careful not to come across as disappointed in their child. They should exhibit strong interest and respect for whatever currently interests their fledging adult (as naive or ill conceived as it may seem) while becoming a partner in exploring options for the future. Most of all, these new adults must feel that they are respected and supported by a family that appreciates them.
    Part C

Directions:
Read the following text carefully and then translate the underlined segments into Chinese. Your translation should be written clearly on ANSWER SHEET 2. (10 points)
The study of law has been recognized for centuries as a basic intellectual discipline in European universities. However, only in recent years has it become a feature of undergraduate programs in Canadian universities. (46) Traditionally, legal learning has been viewed in such institutions as the special preserve of lawyers, rather than a necessary part of the intellectual equipment of an educated person. Happily, the older and more continental view of legal education is establishing itself in a number of Canadian universities and some have even begun to offer undergraduate degrees in law.
If the study of law is beginning to establish itself as part and parcel of a general education, its aims and methods should appeal directly to journalism educators. Law is a discipline which encourages responsible judgment. On the one hand, it provides opportunities to analyze such ideas as justice, democracy and freedom. (47) On the other, it links these concepts to everyday realities in a manner which is parallel to the links journalists forge on a daily basis as they cover and comment on the news. For example, notions of evidence and fact, of basic rights and public interest are at work in the process of journalistic judgment and production just as in courts of law. Sharpening judgment by absorbing and reflecting on law is a desirable component of a journalist’s intellectual preparation for his or her career.
(48) But the idea that the journalist must understand the law more profoundly than an ordinary citizen rests on an understanding of the established conventions and special responsibilities of the news media. Politics or, more broadly, the functioning of the state, is a major subject for journalists. The better informed they are about the way the state works, the better their reporting will be. (49) In fact, it is difficult to see how journalists who do not have a clear grasp of the basic features of the Canadian Constitution can do a competent job on political stories.
Furthermore, the legal system and the events which occur within it are primary subjects for journalists. While the quality of legal journalism varies greatly, there is an undue reliance amongst many journalists on interpretations supplied to them by lawyers. (50) While comment and reaction from lawyers may enhance stories, it is preferable for journalists to rely on their own notions of significance and make their own judgments. These can only come from a well-grounded understanding of the legal system.
Section III Writing

Part A

  1. Directions:
    Write a letter to you university library, making suggestions for improving its service.
    You should write about 100 words on ANSWER SHEET 2.
    Do not sign your own name at the end of the letter. Use “Li Ming” instead.
    Do not write the address. (10 points)
    Part B

  2. Directions:

Write an essay of 160-200 words based on the following drawing. In your essay, you should

  1. describe the drawing briefly,
  2. explain its intended meaning, and then
  3. support your view with an example/examples.
    You should write neatly on ANSWER SHEET 2. (20 points)

2006年全国硕士研究生入学统一考试英语试题

Section I Use of English

Directions:
Read the following text. Choose the best word(s) for each numbered blank and mark [A], [B], [C] or [D] on ANSWER SHEET 1. (10 points)
The homeless make up a growing percentage of America’s population. homelessness has reached such proportions that local governments can’t possibly . To help homeless people independence, the federal government must support job training programs, the minimum wage, and fund more low-cost housing.
everyone agrees on the number of Americans who are homeless. Estimates anywhere from 600,000 to 3 million. the figure may vary, analysts do agree on another matter: that the number of the homeless is . One of the federal government’s studies that the number of the homeless will reach nearly 19 million by the end of this decade.
Finding ways to this growing homeless population has become increasingly difficult. when homeless individuals manage to find a that will give them three meals a day and a place to sleep at night, a good number still spend the bulk of each day the street. Part of the problem is that many homeless adults are addicted to alcohol or drugs. And a significant number of the homeless have serious mental disorders. Many others, not addicted or mentally ill, simply lack the everyday skills needed to turn their lives . Boston Globe reporter Chris Reidy notes that the situation will improve only when there are programs that address the many needs of the homeless. Edward Zlotkowski, director of community service at Bentley College in Massachusetts, it, “There has to be of programs. What’s needed is a package deal.”

  1. [A] Indeed [B] Likewise [C] Therefore [D] Furthermore
  2. [A] stand [B] cope [C] approve [D] retain
  3. [A] in [B] for [C] with [D] toward
  4. [A] raise [B] add [C] take [D] keep
  5. [A] generally [B] almost [C] hardly [D] not
  6. [A] cover [B] change [C] range [D] differ
  7. [A] Now that [B] Although [C] Provided [D] Except that
  8. [A] inflating [B] expanding [C] increasing [D] extending
  9. [A] predicts [B] displays [C] proves [D] discovers
  10. [A] assist [B] track [C] sustain [D] dismiss
  11. [A] Hence [B] But [C] Even [D] Only
  12. [A] lodging [B] shelter [C] dwelling [D] house
  13. [A] searching [B] strolling [C] crowding [D] wandering
  14. [A] when [B] once [C] while [D] whereas
  15. [A] life [B] existence [C] survival [D] maintenance
  16. [A] around [B] over [C] on [D] up
  17. [A] complex [B] comprehensive [C] complementary [D] compensating
  18. [A] So [B] Since [C] As [D] Thus
  19. [A] puts [B] interprets [C] assumes [D] makes
  20. [A] supervision [B] manipulation [C] regulation [D] coordination
    Section II Reading Comprehension

Part A

Directions:
Read the following four texts. Answer the questions below each text by choosing [A], [B], [C], or [D]. Mark your answers on ANSWER SHEET 1. (40 points)
Text 1
In spite of “endless talk of difference,” American society is an amazing machine for homogenizing people. There is “the democratizing uniformity of dress and discourse, and the casualness and absence of deference” characteristic of popular culture. People are absorbed into “a culture of consumption” launched by the 19th-century department stores that offered “vast arrays of goods in an elegant atmosphere. Instead of intimate shops catering to a knowledgeable elite,” these were stores “anyone could enter, regardless of class or background. This turned shopping into a public and democratic act.” The mass media, advertising and sports are other forces for homogenization.
Immigrants are quickly fitting into this common culture, which may not be altogether elevating but is hardly poisonous. Writing for the National Immigration Forum, Gregory Rodriguez reports that today’s immigration is neither at unprecedented levels nor resistant to assimilation. In 1998 immigrants were 9.8 percent of population; in 1900, 13.6 percent. In the 10 years prior to 1990, 3.1 immigrants arrived for every 1,000 residents; in the 10 years prior to 1890, 9.2 for every 1,000. Now, consider three indices of assimilation -- language, home ownership and intermarriage.
The 1990 Census revealed that “a majority of immigrants from each of the fifteen most common countries of origin spoke English ‘well’ or ‘very well’ after ten years of residence.” The children of immigrants tend to be bilingual and proficient in English. “By the third generation, the original language is lost in the majority of immigrant families.” Hence the description of America as a “graveyard” for languages. By 1996 foreign-born immigrants who had arrived before 1970 had a home ownership rate of 75.6 percent, higher than the 69.8 percent rate among native-born Americans.
Foreign-born Asians and Hispanics “have higher rates of intermarriage than do U.S.-born whites and blacks.” By the third generation, one third of Hispanic women are married to non-Hispanics, and 41 percent of Asian-American women are married to non-Asians.
Rodriguez notes that children in remote villages around the world are fans of superstars like Arnold Schwarzenegger and Garth Brooks, yet “some Americans fear that immigrants living within the United States remain somehow immune to the nation’s assimilative power.”
Are there divisive issues and pockets of seething anger in America? Indeed. It is big enough to have a bit of everything. But particularly when viewed against America’s turbulent past, today’s social indices hardly suggest a dark and deteriorating social environment.

  1. The word “homogenizing” (Line 2, Paragraph 1) most probably means ________.
    [A] identifying[B] associating[C] assimilating[D] monopolizing
  2. According to the author, the department stores of the 19th century ________.
    [A] played a role in the spread of popular culture[B] became intimate shops for common consumers
    [C] satisfied the needs of a knowledgeable elite[D] owed its emergence tothe culture of consumption
  3. The text suggests that immigrants now in the U.S. ________.
    [A] are resistant to homogenization[B] exert a great influence on American culture
    [C] are hardly a threat to the common culture[D] constitute the majority of the population
  4. Why are Arnold Schwarzenegger and Garth Brooks mentioned in Paragraph 5?
    [A] To prove their popularity around the world.[B] To reveal the public’s fear of immigrants.
    [C]To give examples of successful immigrants.[D]To show the powerful influence of American culture.
  5. In the author’s opinion, the absorption of immigrants into American society is ________.
    [A] rewarding[B] successful[C] fruitless[D] harmful
    Text 2
    Stratford-on-Avon, as we all know, has only one industry -- William Shakespeare -- but there are two distinctly separate and increasingly hostile branches. There is the Royal Shakespeare Company (RSC), which presents superb productions of the plays at the Shakespeare Memorial Theatre on the Avon. And there are the townsfolk who largely live off the tourists who come, not to see the plays, but to look at Anne Hathaway’s Cottage, Shakespeare’s birthplace and the other sights.
    The worthy residents of Stratford doubt that the theatre adds a penny to their revenue. They frankly dislike the RSC’s actors, them with their long hair and beards and sandals and noisiness. It’s all deliciously ironic when you consider that Shakespeare, who earns their living, was himself an actor (with a beard) and did his share of noise-making.
    The tourist streams are not entirely separate. The sightseers who come by bus -- and often take in Warwick Castle and Blenheim Palace on the side -- don’t usually see the plays, and some of them are even surprised to find a theatre in Stratford. However, the playgoers do manage a little sight-seeing along with their playgoing. It is the playgoers, the RSC contends, who bring in much of the town’s revenue because they spend the night (some of them four or five nights) pouring cash into the hotels and restaurants. The sightseers can take in everything and get out of town by nightfall.
    The townsfolk don’t see it this way and local council does not contribute directly to the subsidy of the Royal Shakespeare Company. Stratford cries poor traditionally. Nevertheless every hotel in town seems to be adding a new wing or cocktail lounge. Hilton is building its own hotel there, which you may be sure will be decorated with Hamlet Hamburger Bars, the Lear Lounge, the Banquo Banqueting Room, and so forth, and will be very expensive.
    Anyway, the townsfolk can’t understand why the Royal Shakespeare Company needs a subsidy. (The theatre has broken attendance records for three years in a row. Last year its 1,431 seats were 94 percent occupied all year long and this year they’ll do better.) The reason, of course, is that costs have rocketed and ticket prices have stayed low.
    It would be a shame to raise prices too much because it would drive away the young people who are Stratford’s most attractive clientele. They come entirely for the plays, not the sights. They all seem to look alike (though they come from all over) -- lean, pointed, dedicated faces, wearing jeans and sandals, eating their buns and bedding down for the night on the flagstones outside the theatre to buy the 20 seats and 80 standing-room tickets held for the sleepers and sold to them when the box office opens at 10:30 a.m.
  6. From the first two paragraphs, we learn that ________.
    [A] the townsfolk deny the RSC’s contribution to the town’s revenue
    [B] the actors of the RSC imitate Shakespeare on and off stage
    [C] the two branches of the RSC are not on good terms
    [D] the townsfolk earn little from tourism
  7. It can be inferred from Paragraph 3 that ________.
    [A] the sightseers cannot visit the Castle and the Palace separately
    [B] the playgoers spend more money than the sightseers
    [C] the sightseers do more shopping than the playgoers
    [D] the playgoers go to no other places in town than the theater
  8. By saying “Stratford cries poor traditionally” (Line 2-3, Paragraph 4), the author implies that ________.
    [A] Stratford cannot afford the expansion projects
    [B] Stratford has long been in financial difficulties
    [C] the town is not really short of money
    [D] the townsfolk used to be poorly paid
  9. According to the townsfolk, the RSC deserves no subsidy because ________.
    [A] ticket prices can be raised to cover the spending[B] the company is financially ill-managed
    [C] the behavior of the actors is not socially acceptable[D] the theatre attendance is on the rise
  10. From the text we can conclude that the author ________.
    [A] is supportive of both sides[B] favors the townsfolk’s view
    [C] takes a detached attitude[D] is sympathetic to the RSC
    Text 3
    When prehistoric man arrived in new parts of the world, something strange happened to the large animals. They suddenly became extinct. Smaller species survived. The large, slow-growing animals were easy game, and were quickly hunted to extinction. Now something similar could be happening in the oceans.
    That the seas are being overfished has been known for years. What researchers such as Ransom Myers and Boris Worm have shown is just how fast things are changing. They have looked at half a century of data from fisheries around the world. Their methods do not attempt to estimate the actual biomass (the amount of living biological matter) of fish species in particular parts of the ocean, but rather changes in that biomass over time. According to their latest paper published in Nature, the biomass of large predators (animals that kill and eat other animals) in a new fishery is reduced on average by 80% within 15 years of the start of exploitation. In some long-fished areas, it has halved again since then.
    Dr. Worm acknowledges that these figures are conservative. One reason for this is that fishing technology has improved. Today’s vessels can find their prey using satellites and sonar, which were not available 50 years ago. That means a higher proportion of what is in the sea is being caught, so the real difference between present and past is likely to be worse than the one recorded by changes in catch sizes. In the early days, too, longlines would have been more saturated with fish. Some individuals would therefore not have been caught, since no baited hooks would have been available to trap them, leading to an underestimate of fish stocks in the past. Furthermore, in the early days of longline fishing, a lot of fish were lost to sharks after they had been hooked. That is no longer a problem, because there are fewer sharks around now.
    Dr. Myers and Dr. Worm argue that their work gives a correct baseline, which future management efforts must take into account. They believe the data support an idea current among marine biologists, that of the “shifting baseline.” The notion is that people have failed to detect the massive changes which have happened in the ocean because they have been looking back only a relatively short time into the past. That matters because theory suggests that the maximum sustainable yield that can be cropped from a fishery comes when the biomass of a target species is about 50% of its original levels. Most fisheries are well below that, which is a bad way to do business.
  11. The extinction of large prehistoric animals is noted to suggest that ________.
    [A] large animal were vulnerable to the changing environment
    [B] small species survived as large animals disappeared
    [C] large sea animals may face the same threat today
    [D] slow-growing fish outlive fast-growing ones
  12. We can infer from Dr. Myers and Dr. Worm’s paper that ________.
    [A] the stock of large predators in some old fisheries has reduced by 90%
    [B] there are only half as many fisheries as there were 15 years ago
    [C] the catch sizes in new fisheries are only 20% of the original amount
    [D] the number of larger predators dropped faster in new fisheries than in the old
  13. By saying "these figures are conservative" (Line 1, paragraph 3), Dr. Worm means that ________.
    [A] fishing technology has improved rapidly
    [B] the catch-sizes are actually smaller than recorded
    [C] the marine biomass has suffered a greater loss
    [D] the data collected so far are out of date
  14. Dr. Myers and other researchers hold that ________.
    [A] people should look for a baseline that can work for a longer time
    [B] fisheries should keep their yields below 50% of the biomass
    [C] the ocean biomass should be restored to its original level
    [D] people should adjust the fishing baseline to the changing situation
  15. The author seems to be mainly concerned with most fisheries’ ________.
    [A] management efficiency[B] biomass level[C] catch-size limits[D] technological application
    Text 4
    Many things make people think artists are weird. But the weirdest may be this: artists’ only job is to explore emotions, and yet they choose to focus on the ones that feel bad.
    This wasn’t always so. The earliest forms of art, like painting and music, are those best suited for expressing joy. But somewhere from the 19th century onward, more artists began seeing happiness as meaningless, phony or, worst of all, boring, as we went from Wordsworth’s daffodils to Baudelaire’s flowers of evil.
    You could argue that art became more skeptical of happiness because modern times have seen so much misery. But it’s not as if earlier times didn’t know perpetual war, disaster and the massacre of innocents. The reason, in fact, may be just the opposite: there is too much damn happiness in the world today.
    After all, what is the one modern form of expression almost completely dedicated to depicting happiness? Advertising. The rise of anti-happy art almost exactly tracks the emergence of mass media, and with it, a commercial culture in which happiness is not just an ideal but an ideology.
    People in earlier eras were surrounded by reminders of misery. They worked until exhausted, lived with few protections and died young. In the West, before mass communication and literacy, the most powerful mass medium was the church, which reminded worshippers that their souls were in danger and that they would someday be meat for worms. Given all this, they did not exactly need their art to be a bummer too.
    Today the messages the average Westerner is surrounded with are not religious but commercial, and forever happy. Fast-food eaters, news anchors, text messengers, all smiling, smiling, smiling. Our magazines feature beaming celebrities and happy families in perfect homes. And since these messages have an agenda -- to lure us to open our wallets -- they make the very idea of happiness seem unreliable. “Celebrate!” commanded the ads for the arthritis drug Celebrex, before we found out it could increase the risk of heart attacks.
    But what we forget -- what our economy depends on us forgetting -- is that happiness is more than pleasure without pain. The things that bring the greatest joy carry the greatest potential for loss and disappointment. Today, surrounded by promises of easy happiness, we need art to tell us, as religion once did, Memento mori: remember that you will die, that everything ends, and that happiness comes not in denying this but in living with it. It’s a message even more bitter than a clove cigarette, yet, somehow, a breath of fresh air.
  16. By citing the examples of poets Wordsworth and Baudelaire, the author intends to show that ________.
    [A] poetry is not as expressive of joy as painting or music
    [B] art grows out of both positive and negative feelings
    [C] poets today are less skeptical of happiness
    [D] artists have changed their focus of interest
  17. The word “bummer” (Line 5, paragraph 5) most probably means something ________.
    [A] religious[B] unpleasant[C] entertaining[D] commercial
  18. In the author’s opinion, advertising ________.
    [A] emerges in the wake of the anti-happy art
    [B] is a cause of disappointment for the general public
    [C] replaces the church as a major source of information
    [D] creates an illusion of happiness rather than happiness itself
  19. We can learn from the last paragraph that the author believes ________.
    [A] happiness more often than not ends in sadness[B] the anti-happy art is distasteful but refreshing
    [C] misery should be enjoyed rather than denied
    [D] the anti-happy art flourishes when economy booms
  20. Which of the following is true of the text?
    [A] Religion once functioned as a reminder of misery.
    [B] Art provides a balance between expectation and reality.
    [C] People feel disappointed at the realities of modern society.
    [D] Mass media are inclined to cover disasters and deaths.
    Part B
    Directions:
    In the following article, some sentences have been removed. For Questions 41-45, choose the most suitable one from the list A-G to fit into each of the numbered gaps. There are two extra choices, which you do not need to use in any of the blanks. Mark your answers on ANSWER SHEET 1. (10 points)
    On the north bank of the Ohio river sits Evansville, Ind., home of David Williams, 52, and of a riverboat casino (a place where gambling games are played). During several years of gambling in that casino, Williams, a state auditor earning $35,000 a year, lost approximately $175,000. He had never gambled before the casino sent him a coupon for $20 worth of gambling.
    He visited the casino, lost the $20 and left. On his second visit he lost $800. The casino issued to him, as a good customer, a "Fun Card", which when used in the casino earns points for meals and drinks, and enables the casino to track the user’s gambling activities. For Williams, those activities become what he calls "electronic heroin".
    (41) ________. In 1997 he lost $21,000 to one slot machine in two days. In March 1997 he lost $72,186. He sometimes played two slot machines at a time, all night, until the boat docked at 5 a.m., then went back aboard when the casino opened at 9 a.m. Now he is suing the casino, charging that it should have refused his patronage because it knew he was addicted. It did know he had a problem.
    In March 1998 a friend of Williams’s got him involuntarily confined to a treatment center for addictions, and wrote to inform the casino of Williams’s gambling problem. The casino included a photo of Williams among those of banned gamblers, and wrote to him a “cease admissions” letter. Noting the medical/psychological nature of problem gambling behavior, the letter said that before being readmitted to the casino he would have to present medical/psychological information demonstrating that patronizing the casino would pose no threat to his safety or well-being.
    (42) ________.
    The Wall Street Journal reports that the casino has 24 signs warning: “Enjoy the fun... and always bet with your head, not over it.” Every entrance ticket lists a toll-free number for counseling from the Indiana Department of Mental Health. Nevertheless, Williams’s suit charges that the casino, knowing he was “helplessly addicted to gambling,” intentionally worked to “lure” him to “engage in conduct against his will.” Well.
    (43) ________.
    The fourth edition of the Diagnostic and Statistical Manual of Mental Disorders says “pathological gambling” involves persistent, recurring and uncontrollable pursuit less of money than of thrill of taking risks in quest of a windfall.
    (44) ________. Pushed by science, or what claims to be science, society is reclassifying what once were considered character flaws or moral failings as personality disorders akin to physical disabilities.
    (45) ________.
    Forty-four states have lotteries, 29 have casinos, and most of these states are to varying degrees dependent on -- you might say addicted to -- revenues from wagering. And since the first Internet gambling site was created in 1995, competition for gamblers’ dollars has become intense. The Oct. 28 issue of Newsweek reported that 2 million gamblers patronize 1,800 virtual casinos every week. With $3.5 billion being lost on Internet wagers this year, gambling has passed pornography as the Web’s most profitable business.
    [A] Although no such evidence was presented, the casino’s marketing department continued to pepper him with mailings. And he entered the casino and used his Fun Card without being detected.
    [B] It is unclear what luring was required, given his compulsive behavior. And in what sense was his will operative?
    [C] By the time he had lost $5,000 he said to himself that if he could get back to even, he would quit. One night he won $5,500, but he did not quit.
    [D] Gambling has been a common feature of American life forever, but for a long time it was broadly considered a sin, or a social disease. Now it is a social policy: the most important and aggressive promoter of gambling in America is the government.
    [E] David Williams’s suit should trouble this gambling nation. But don’t bet on it.
    [F] It is worrisome that society is medicalizing more and more behavioral problems, often defining as addictions what earlier, sterner generations explained as weakness of will.
    [G] The anonymous, lonely, undistracted nature of online gambling is especially conducive to compulsive behavior. But even if the government knew how to move against Internet gambling, what would be its grounds for doing so?
    Part C
    Directions:
    Read the following text carefully and then translate the underlined segments into Chinese. Your translation should be written clearly on ANSWER SHEET 2. (10 points)
    Is it true that the American intellectual is rejected and considered of no account in his society? I am going to suggest that it is not true. Father Bruckberger told part of the story when he observed that it is the intellectuals who have rejected America. But they have done more than that. They have grown dissatisfied with the role of intellectual. It is they, not America, who have become anti-intellectual.
    First, the object of our study pleads for definition. What is an intellectual? 46) I shall define him as an individual who has elected as his primary duty and pleasure in life the activity of thinking in a Socratic (苏格拉底) way about moral problems. He explores such problems consciously, articulately, and frankly, first by asking factual questions, then by asking moral questions, finally by suggesting action which seems appropriate in the light of the factual and moral information which he has obtained. 47) His function is analogous to that of a judge, who must accept the obligation of revealing in as obvious a manner as possible the course of reasoning which led him to his decision.
    This definition excludes many individuals usually referred to as intellectuals -- the average scientist, for one. 48) I have excluded him because, while his accomplishments may contribute to the solution of moral problems, he has not been charged with the task of approaching any but the factual aspects of those problems. Like other human beings, he encounters moral issues even in the everyday performance of his routine duties -- he is not supposed to cook his experiments, manufacture evidence, or doctor his reports. 49) But his primary task is not to think about the moral code which governs his activity, any more than a businessman is expected to dedicate his energies to an exploration of rules of conduct in business. During most of his waking life he will take his code for granted, as the businessman takes his ethics.
    The definition also excludes the majority of teachers, despite the fact that teaching has traditionally been the method whereby many intellectuals earn their living. 50) They may teach very well and more than earn their salaries, but most of them make little or no independent reflections on human problems which involve moral judgment. This description even fits the majority of eminent scholars. Being learned in some branch of human knowledge is one thing, living in "public and illustrious thoughts,” as Emerson would say, is something else.
    Section III Writing
    Part A
  21. Directions
    You want to contribute to Project Hope by offering financial aid to a child in a remote area. Write a letter to the department concerned, asking them to help find a candidate. You should specify what kind of child you want to help and how you will carry out your plan.
    Write your letter in no less than 100 words. Write it neatly on ANSWER SHEET 2.
    Do not sign your own name at the end of the letter; use “Li Ming” instead.
    Do not write the address. (10 points)
    Part B
  22. Directions:
    Study the following photos carefully and write an essay in which you should
  23. describe the photos briefly,
  24. interpret the social phenomenon reflected by them, and
  25. give your point of view.
    You should write 160-200 words neatly on ANSWER SHEET 2. (20 points)

2005年全国硕士研究生入学统一考试英语试题

Section I Use of English

Directions:
Read the following text. Choose the best word(s) for each numbered blank and mark [A], [B], [C] or [D] on ANSWER SHEET 1 (10 points)
The human nose is an underrated tool. Humans are often thought to be insensitive smellers compared with animals, this is largely because, animals, we stand upright. This means that our noses are to perceiving those smells which float through the air, the majority of smells which stick to surfaces. In fact, , we are extremely sensitive to smells, we do not generally realize it. Our noses are capable of human smells even when these are to far below one part in one million.
Strangely, some people find that they can smell one type of flower but not another, others are sensitive to the smells of both flowers. This may be because some people do not have the genes necessary to generate smell receptors in the nose. These receptors are the cells which sense smells and send to the brain. However, it has been found that even people insensitive to a certain smell can suddenly become sensitive to it when to it often enough.
The explanation for insensitivity to smell seems to be that the brain finds it to keep all smell receptors working all the time but can new receptors if necessary. This may explain why we are not usually sensitive to our own smells—we simply do not need to be. We are not of the usual smell of our own house, but we new smells when we visit someone else’s. The brain finds it best to keep smell receptors for unfamiliar and emergency signals the smell of smoke, which might indicate the danger of fire.

  1. [A] although[B] as[C] but[D] while
  2. [A] above[B] unlike[C] excluding[D] besides
  3. [A] limited[B] committed[C] dedicated[D] confined
  4. [A] catching[B] ignoring[C] missing[D] tracking
  5. [A] anyway[B] though[C] instead[D] therefore
  6. [A] even if[B] if only[C] only if[D] as if
  7. [A] distinguishing[B] discovering[C] determining[D] detecting
  8. [A] diluted[B] dissolved[C] dispersed[D] diffused
  9. [A] when[B] since[C] for[D] whereas
  10. [A] unusual[B] particular[C] unique[D] typical
  11. [A] signs[B] stimuli[C] messages[D] impulses
  12. [A] at first[B] at all[C] at large[D] at times
  13. [A] subjected[B] left[C] drawn[D] exposed
  14. [A] ineffective[B] incompetent[C] inefficient[D] insufficient
  15. [A] introduce[B] summon[C] trigger[D] create
  16. [A] still[B] also[C] otherwise[D] nevertheless
  17. [A] sure[B] sick[C] aware[D] tired
  18. [A] tolerate[B] repel[C] neglect[D] notice
  19. [A] available[B] reliable[C] identifiable[D] suitable
  20. [A] similar to[B] such as[C] along with[D] aside from
    Section II Reading Comprehension

Part A

Directions:
Read the following four texts. Answer the questions below each text by choosing [A], [B], [C] or D. Mark your answers on ANSWER SHEET 1 (40 points)
Text 1

Everybody loves a fat pay rise. Yet pleasure at your own can vanish if you learn that a colleague has been given a bigger one. Indeed, if he has a reputation for slacking, you might even be outraged. Such behaviour is regarded as “all too human,” with the underlying assumption that other animals would not be capable of this finely developed sense of grievance. But a study by Sarah Brosnan and Frans de Waal of Emory University in Atlanta, Georgia, which has just been published in Nature, suggests that it is all too monkey, as well.
The researchers studied the behaviour of female brown capuchin monkeys. They look cute. They are good-natured, co-operative creatures, and they share their food readily. Above all, like their female human counterparts, they tend to pay much closer attention to the value of “goods and services” than males.
Such characteristics make them perfect candidates for Dr. Brosnan’s and Dr. de Waal’s study. The researchers spent two years teaching their monkeys to exchange tokens for food. Normally, the monkeys were happy enough to exchange pieces of rock for slices of cucumber. However, when two monkeys were placed in separate but adjoining chambers, so that each could observe what the other was getting in return for its rock, their behaviour became markedly different.
In the world of capuchins, grapes are luxury goods (and much preferable to cucumbers). So when one monkey was handed a grape in exchange for her token, the second was reluctant to hand hers over for a mere piece of cucumber. And if one received a grape without having to provide her token in exchange at all, the other either tossed her own token at the researcher or out of the chamber, or refused to accept the slice of cucumber. Indeed, the mere presence of a grape in the other chamber (without an actual monkey to eat it) was enough to induce resentment in a female capuchin.
The researchers suggest that capuchin monkeys, like humans, are guided by social emotions. In the wild, they are a co-operative, group-living species. Such co-operation is likely to be stable only when each animal feels it is not being cheated. Feelings of righteous indignation, it seems, are not the preserve of people alone. Refusing a lesser reward completely makes these feelings abundantly clear to other members of the group. However, whether such a sense of fairness evolved independently in capuchins and humans, or whether it stems from the common ancestor that the species had 35 million years ago, is, as yet, an unanswered question.

  1. In the opening paragraph, the author introduces his topic by ________.
    [A] posing a contrast[B] justifying an assumption
    [C] making a comparison[D] explaining a phenomenon
  2. The statement “it is all too monkey” (Last line, Paragraph l) implies that ________.
    [A] monkeys are also outraged by slack rivals[B] resenting unfairness is also monkeys’ nature
    [C] monkeys, like humans, tend to be jealous of each other
    [D] no animals other than monkeys can develop such emotions
  3. Female capuchin monkeys were chosen for the research most probably because they are ________.
    [A] more inclined to weigh what they get[B] attentive to researchers’ instructions
    [C] nice in both appearance and temperament[D] more generous than their male companions
  4. Dr. Brosnan and Dr. de Waal have eventually found in their study that the monkeys ________.
    [A] prefer grapes to cucumbers[B] can be taught to exchange things
    [C] will not be co-operative if feeling cheated[D] are unhappy when separated from others
  5. What can we infer from the last paragraph?
    [A] Monkeys can be trained to develop social emotions.
    [B] Human indignation evolved from an uncertain source.
    [C] Animals usually show their feelings openly as humans do.
    [D] Cooperation among monkeys remains stable only in the wild.
    Text 2

Do you remember all those years when scientists argued that smoking would kill us but the doubters insisted that we didn’t know for sure? That the evidence was inconclusive, the science uncertain? That the antismoking lobby was out to destroy our way of life and the government should stay out of the way? Lots of Americans bought that nonsense, and over three decades, some 10 million smokers went to early graves.
There are upsetting parallels today, as scientists in one wave after another try to awaken us to the growing threat of global warming. The latest was a panel from the National Academy of Sciences, enlisted by the White House, to tell us that the Earth’s atmosphere is definitely warming and that the problem is largely man-made. The clear message is that we should get moving to protect ourselves. The president of the National Academy, Bruce Alberts, added this key point in the preface to the panel’s report: “Science never has all the answers. But science does provide us with the best available guide to the future, and it is critical that our nation and the world base important policies on the best judgments that science can provide concerning the future consequences of present actions.”
Just as on smoking, voices now come from many quarters insisting that the science about global warming is incomplete, that it’s OK to keep pouring fumes into the air until we know for sure. This is a dangerous game: by the time 100 percent of the evidence is in, it may be too late. With the risks obvious and growing, a prudent people would take out an insurance policy now.
Fortunately, the White House is starting to pay attention. But it’s obvious that a majority of the president’s advisers still don’t take global warming seriously. Instead of a plan of action, they continue to press for more research -- a classic case of “paralysis by analysis.”
To serve as responsible stewards of the planet, we must press forward on deeper atmospheric and oceanic research. But research alone is inadequate. If the Administration won’t take the legislative initiative, Congress should help to begin fashioning conservation measures. A bill by Democratic Senator Robert Byrd of West Virginia, which would offer financial incentives for private industry, is a promising start. Many see that the country is getting ready to build lots of new power plants to meet our energy needs. If we are ever going to protect the atmosphere, it is crucial that those new plants be environmentally sound.

  1. An argument made by supporters of smoking was that ________.
    [A] there was no scientific evidence of the correlation between smoking and death
    [B] the number of early deaths of smokers in the past decades was insignificant
    [C] people had the freedom to choose their own way of life
    [D] antismoking people were usually talking nonsense
  2. According to Bruce Alberts, science can serve as ________.
    [A] a protector[B] a judge[C] a critic[D] a guide
  3. What does the author mean by “paralysis by analysis” (Last line, Paragraph 4)?
    [A] Endless studies kill action.[B] Careful investigation reveals truth.
    [C] Prudent planning hinders progress.[D] Extensive research helps decision-making.
  4. According to the author, what should the Administration do about global warming?
    [A] Offer aid to build cleaner power plants.[B] Raise public awareness of conservation.
    [C] Press for further scientific research.[D] Take some legislative measures.
  5. The author associates the issue of global warming with that of smoking because ________.
    [A] they both suffered from the government’s negligence
    [B] a lesson from the latter is applicable to the former
    [C] the outcome of the latter aggravates the former[D] both of them have turned from bad to worse
    Text 3

Of all the components of a good night’s sleep, dreams seem to be least within our control. In dreams, a window opens into a world where logic is suspended and dead people speak. A century ago, Freud formulated his revolutionary theory that dreams were the disguised shadows of our unconscious desires and fears; by the late 1970s, neurologists had switched to thinking of them as just “mental noise” -- the random byproducts of the neural-repair work that goes on during sleep. Now researchers suspect that dreams are part of the mind’s emotional thermostat, regulating moods while the brain is “off-line.” And one leading authority says that these intensely powerful mental events can be not only harnessed but actually brought under conscious control, to help us sleep and feel better, “It’s your dream,” says Rosalind Cartwright, chair of psychology at Chicago’s Medical Center. “If you don’t like it, change it.”
Evidence from brain imaging supports this view. The brain is as active during REM (rapid eye movement) sleep -- when most vivid dreams occur -- as it is when fully awake, says Dr, Eric Nofzinger at the University of Pittsburgh. But not all parts of the brain are equally involved; the limbic system (the “emotional brain”) is especially active, while the prefrontal cortex (the center of intellect and reasoning) is relatively quiet. “We wake up from dreams happy or depressed, and those feelings can stay with us all day.” says Stanford sleep researcher Dr. William Dement.
The link between dreams and emotions shows up among the patients in Cartwright’s clinic. Most people seem to have more bad dreams early in the night, progressing toward happier ones before awakening, suggesting that they are working through negative feelings generated during the day. Because our conscious mind is occupied with daily life we don’t always think about the emotional significance of the day’s events -- until, it appears, we begin to dream.
And this process need not be left to the unconscious. Cartwright believes one can exercise conscious control over recurring bad dreams. As soon as you awaken, identify what is upsetting about the dream. Visualize how you would like it to end instead; the next time it occurs, try to wake up just enough to control its course. With much practice people can learn to, literally, do it in their sleep.
At the end of the day, there’s probably little reason to pay attention to our dreams at all unless they keep us from sleeping or “we wake up in a panic,” Cartwright says. Terrorism, economic uncertainties and general feelings of insecurity have increased people’s anxiety. Those suffering from persistent nightmares should seek help from a therapist. For the rest of us, the brain has its ways of working through bad feelings. Sleep -- or rather dream -- on it and you’ll feel better in the morning.

  1. Researchers have come to believe that dreams ________.
    [A] can be modified in their courses[B] are susceptible to emotional changes
    [C] reflect our innermost desires and fears[D] are a random outcome of neural repairs
  2. By referring to the limbic system, the author intends to show ________.
    [A] its function in our dreams[B] the mechanism of REM sleep
    [C] the relation of dreams to emotions[D] its difference from the prefrontal cortex
  3. The negative feelings generated during the day tend to ________.
    [A] aggravate in our unconscious mind[B] develop into happy dreams
    [C] persist till the time we fall asleep[D] show up in dreams early at night
  4. Cartwright seems to suggest that ________.
    [A] waking up in time is essential to the ridding of bad dreams[B] visualizing bad dreams helps bring them under control[C] dreams should be left to their natural progression
    [D] dreaming may not entirely belong to the unconscious
  5. What advice might Cartwright give to those who sometimes have bad dreams?
    [A] Lead your life as usual.[B] Seek professional help.
    [C] Exercise conscious control.[D] Avoid anxiety in the daytime.
    Text 4

Americans no longer expect public figures, whether in speech or in writing, to command the English language with skill and gift. Nor do they aspire to such command themselves. In his latest book, Doing Our Own Thing: The Degradation of Language and Music and Why We Should, Like, Care, John McWhorter, a linguist and controversialist of mixed liberal and conservative views, sees the triumph of 1960s counter-culture as responsible for the decline of formal English.
Blaming the permissive 1960s is nothing new, but this is not yet another criticism against the decline in education. Mr. McWhorter’s academic speciality is language history and change, and he sees the gradual disappearance of “whom,” for example, to be natural and no more regrettable than the loss of the case-endings of Old English.
But the cult of the authentic and the personal, “doing our own thing,” has spelt the death of formal speech, writing, poetry and music. While even the modestly educated sought an elevated tone when they put pen to paper before the 1960s, even the most well regarded writing since then has sought to capture spoken English on the page. Equally, in poetry, the highly personal, performative genre is the only form that could claim real liveliness. In both oral and written English, talking is triumphing over speaking, spontaneity over craft.
Illustrated with an entertaining array of examples from both high and low culture, the trend that Mr. McWhorter documents is unmistakable. But it is less clear, to take the question of his subtitle, why we should, like, care. As a linguist, he acknowledges that all varieties of human language, including non-standard ones like Black English, can be powerfully expressive -- there exists no language or dialect in the world that cannot convey complex ideas. He is not arguing, as many do, that we can no longer think straight because we do not talk proper.
Russians have a deep love for their own language and carry large chunks of memorized poetry in their heads, while Italian politicians tend to elaborate speech that would seem old-fashioned to most English-speakers. Mr. McWhorter acknowledges that formal language is not strictly necessary, and proposes no radical education reforms -- he is really grieving over the loss of something beautiful more than useful. We now take our English “on paper plates instead of china.” A shame, perhaps, but probably an inevitable one.

  1. According to McWhorter, the decline of formal English ________.
    [A] is inevitable in radical education reforms
    [B] is but all too natural in language development
    [C] has caused the controversy over the counter-culture
    [D] brought about changes in public attitudes in the 1960s
  2. The word “talking” (Line 6, Paragraph 3) denotes ________.
    [A] modesty[B] personality[C] liveliness[D] informality
  3. To which of the following statements would McWhorter most likely agree?
    [A] Logical thinking is not necessarily related to the way we talk.
    [B] Black English can be more expressive than standard English.
    [C] Non-standard varieties of human language are just as entertaining.
    [D] Of all the varieties, standard English can best convey complex ideas.
  4. The description of Russians’ love of memorizing poetry shows the author’s ________.
    [A] interest in their language[B] appreciation of their efforts
    [C] admiration for their memory
    [D] contempt for their old-fashionedness
  5. According to the last paragraph, “paper plates” is to “china” as ________.
    [A] “temporary” is to “permanent”[B] “radical” is to “conservative”
    [C] “functional” is to “artistic”[D] “humble” is to “noble”
    Part B

Directions:
In the following text, some sentences have been removed. For Questions 41-45, choose the most suitable one from the list A-G to fit into each of the numbered blanks. There are two extra choices, which do not fit in any of the gaps. Mark your answers on ANSWER SHEET 1. (10 points)
Canada’s premiers (the leaders of provincial governments), if they have any breath left after complaining about Ottawa at their late July annual meeting, might spare a moment to do something, together, to reduce health-care costs.
They’re all groaning about soaring health budgets, the fastest-growing component of which are pharmaceutical costs.


What to do? Both the Romanow commission and the Kirby committee on health care -- to say nothing of reports from other experts -- recommended the creation of a national drug agency. Instead of each province having its own list of approved drugs, bureaucracy, procedures and limited bargaining power, all would pool resources, work with Ottawa, and create a national institution.


But “national” doesn’t have to mean that. “National” could mean interprovincial -- provinces combining efforts to create one body.
Either way, one benefit of a “national” organization would be to negotiate better prices, if possible, with drug manufacturers. Instead of having one province -- or a series of hospitals within a province -- negotiate a price for a given drug on the provincial list, the national agency would negotiate on behalf of all provinces.
Rather than, say, Quebec, negotiating on behalf of seven million people, the national agency would negotiate on behalf of 31 million people. Basic economics suggests the greater the potential consumers, the higher the likelihood of a better price.


A small step has been taken in the direction of a national agency with the creation of the Canadian Co-ordinating Office for Health Technology Assessment, funded by Ottawa and the provinces. Under it, a Common Drug Review recommends to provincial lists which new drugs should be included. Predictably, and regrettably, Quebec refused to join.
A few premiers are suspicious of any federal-provincial deal-making. They (particularly Quebec and Alberta) just want Ottawa to fork over additional billions with few, if any, strings attached. That’s one reason why the idea of a national list hasn’t gone anywhere, while drug costs keep rising fast.


Premiers love to quote Mr. Romanow’s report selectively, especially the parts about more federal money. Perhaps they should read what he had to say about drugs: “A national drug agency would provide governments more influence on pharmaceutical companies in order to constrain the ever-increasing cost of drugs.”


So when the premiers gather in Niagara Falls to assemble their usual complaint list, they should also get cracking about something in their jurisdiction that would help their budgets and patients.
[A] Quebec’s resistance to a national agency is provincialist ideology. One of the first advocates for a national list was a researcher at Laval University. Quebec’s Drug Insurance Fund has seen its costs skyrocket with annual increases from 14.3 per cent to 26.8 per cent!
[B] Or they could read Mr. Kirby’s report: “the substantial buying power of such an agency would strengthen the public prescription-drug insurance plans to negotiate the lowest possible purchase prices from drug companies.”
[C] What does “national” mean? Roy Romanow and Senator Michael Kirby recommended a federal-provincial body much like the recently created National Health Council.
[D] The problem is simple and stark: health-care costs have been, are, and will continue to increase faster than government revenues.
[E] According to the Canadian Institute for Health Information, prescription drug costs have risen since 1997 at twice the rate of overall health-care spending. Part of the increase comes from drugs being used to replace other kinds of treatments. Part of it arises from new drugs costing more than older kinds. Part of it is higher prices.
[F] So, if the provinces want to run the health-care show, they should prove they can run it, starting with an interprovincial health list that would end duplication, save administrative costs, prevent one province from being played off against another, and bargain for better drug prices.
[G] Of course, the pharmaceutical companies will scream. They like divided buyers; they can lobby better that way. They can use the threat of removing jobs from one province to another. They can hope that, if one province includes a drug on its list, the pressure will cause others to include it on theirs. They wouldn’t like a national agency, but self-interest would lead them to deal with it.
Part C

Directions:
Read the following text carefully and then translate the underlined segments into Chinese. Your translation should be written clearly on ANSWER SHEET 2. (10 points)
It is not easy to talk about the role of the mass media in this overwhelmingly significant phase in European history. History and news become confused, and one’s impressions tend to be a mixture of skepticism and optimism. 46) Television is one of the means by which these feelings are created and conveyed -- and perhaps never before has it served so much to connect different peoples and nations as in the recent events in Europe. The Europe that is now forming cannot be anything other than its peoples, their cultures and national identities. With this in mind we can begin to analyze the European television scene. 47) In Europe, as elsewhere, multi-media groups have been increasingly successful: groups which bring together television, radio, newspapers, magazines and publishing houses that work in relation to one another. One Italian example would be the Berlusconi group, while abroad Maxwell and Murdoch come to mind.
Clearly, only the biggest and most flexible television companies are going to be able to compete in such a rich and hotly-contested market. 48) This alone demonstrates that the television business is not an easy world to survive in, a fact underlined by statistics that show that out of eighty European television networks, no less than 50% took a loss in 1989.
Moreover, the integration of the European community will oblige television companies to cooperate more closely in terms of both production and distribution.

  1. Creating a “European identity” that respects the different cultures and traditions which go to make up the connecting fabric of the Old Continent is no easy task and demands a strategic choice -- that of producing programs in Europe for Europe. This entails reducing our dependence on the North American market, whose programs relate to experiences and cultural traditions which are different from our own.
    In order to achieve these objectives, we must concentrate more on co-productions, the exchange of news, documentary services and training. This also involves the agreements between European countries for the creation of a European bank for Television Production which, on the model of the European Investments Bank, will handle the finances necessary for production costs. 50) In dealing with a challenge on such a scale, it is no exaggeration to say “United we stand, divided we fall” -- and if I had to choose a slogan it would be “Unity in our diversity.” A unity of objectives that nonetheless respect the varied peculiarities of each country.
    Section III Writing

Part A

  1. Directions:
    Two months ago you got a job as an editor for the magazine Designs & Fashions. But now you find that the work is not what you expected. You decide to quit. Write a letter to your boss, Mr. Wang, telling him your decision, stating your reason (s), and making an apology.
    Write your letter with no less than 100 words. Write it neatly on ANSWER SHEET 2.
    Do not sign your own name at the end of the letter; use “Li Ming” instead.
    You do not need to write the address. (10 points)
    Part B

  2. Directions:
    Write an essay of 160-200 words based on the following drawing. In your essay, you should first describe the drawing, then interpret its meaning, and give your comment on it.
    You should write neatly on ANSWER SHEET 2. (20 points)



    2004
    Section II Use of English

Directions:
Read the following text. Choose the best word (s) for each numbered blank and mark [A], [B], [C] or [D] on ANSWER SHEET 1. (10 points)
Many theories concerning the causes of juvenile delinquency (crimes committed by young people) focus either on the individual or on society as the major contributing influence. Theories on the individual suggest that children engage in criminal behavior they were not sufficiently penalized for previous misdeeds or that they have learned criminal behavior through with others. Theories focusing on the role of society suggest that children commit crimes in to their failure to rise above their socioeconomic status, as a rejection of middle-class values.
Most theories of juvenile delinquency have focused on children from disadvantaged families, the fact that children from wealthy homes also commit crimes. The latter may commit crimes lack of adequate parental control. All theories, however, are tentative and are to criticism.
Changes in the social structure may indirectly juvenile crime rates. For example, changes in the economy that to fewer job opportunities for youth and rising unemployment make gainful employment increasingly difficult to obtain. The resulting discontent may in lead more youths into criminal behavior.
Families have also changes these years. More families consist of one-parent households or two working parents; , children are likely to have less supervision at home was common in the traditional family . This lack of parental supervision is thought to be an influence on juvenile crime rates. Other causes of offensive acts include frustration or failure in school, the increased of drugs and alcohol, and the growing of child abuse and child neglect. All these conditions tend to increase the probability of a child committing a criminal act, a direct causal relationship has not yet been established.

  1. [A] acting[B] relying[C] centering[D] commenting
  2. [A] before[B] unless[C] until[D] because
  3. [A] interactions[B] assimilation[C] cooperation[D] consultation
  4. [A] return[B] reply[C] reference[D] response
  5. [A] or[B] but rather[C] but[D] or else
    26 [A] considering[B] ignoring[C] highlighting[D] discarding
  6. [A] on[B] in[C] for[D] with
  7. [A] immune[B] resistant[C] sensitive[D] subject
  8. [A] affect[B] reduce[C] check[D] reflect
  9. [A] point[B] lead[C] come[D] amount
  10. [A] in general[B] on average[C] by contrast[D] at length
  11. [A] case[B] short[C] turn[D] essence
  12. [A] survived[B] noticed[C] undertaken[D] experienced
  13. [A] contrarily[B] consequently[C] similarly[D] simultaneously
  14. [A] than[B] that[C] which[D] as
  15. [A] system[B] structure[C] concept[D] heritage
  16. [A] assessable[B] identifiable[C] negligible[D] incredible
  17. [A] expense[B] restriction[C] allocation[D] availability
  18. [A] incidence[B] awareness[C] exposure[D] popularity
  19. [A] provided[B] since[C] although[D] supposing
    Section III Reading Comprehension

Part A

Directions:
Read the following four texts. Answer the questions below each text by choosing [A], [B], [C] or [D] Mark your answers on ANSWER SHEET 1. (40 points)
Text 1

Hunting for a job late last year, lawyer Gant Redmon stumbled across CareerBuilder, a job database on the Internet. He searched it with no success but was attracted by the site’s “personal search agent.” It’s an interactive feature that lets visitors key in job criteria such as location, title, and salary, then E-mails them when a matching position is posted in the database. Redmon chose the keywords legal, intellectual property, and Washington, D.C. Three weeks later, he got his first notification of an opening. “I struck gold,” says Redmon, who E-mailed his resume to the employer and won a position as in-house counsel for a company.
With thousands of career-related sites on the Internet, finding promising openings can be time-consuming and inefficient. Search agents reduce the need for repeated visits to the databases. But although a search agent worked for Redmon, career experts see drawbacks. Narrowing your criteria, for example, may work against you: “Every time you answer a question you eliminate a possibility.” says one expert.
For any job search, you should start with a narrow concept -- what you think you want to do -- then broaden it. “None of these programs do that,” says another expert. “There’s no career counseling implicit in all of this.” Instead, the best strategy is to use the agent as a kind of tip service to keep abreast of jobs in a particular database; when you get E-mail, consider it a reminder to check the database again. “I would not rely on agents for finding everything that is added to a database that might interest me,” says the author of a job-searching guide.
Some sites design their agents to tempt job hunters to return. When CareerSite’s agent sends out messages to those who have signed up for its service, for example, it includes only three potential jobs -- those it considers the best matches. There may be more matches in the database; job hunters will have to visit the site again to find them -- and they do. “On the day after we send our messages, we see a sharp increase in our traffic,” says Seth Peets, vice president of marketing for CareerSite.
Even those who aren’t hunting for jobs may find search agents worthwhile. Some use them to keep a close watch on the demand for their line of work or gather information on compensation to arm themselves when negotiating for a raise. Although happily employed, Redmon maintains his agent at CareerBuilder. “You always keep your eyes open,” he says. Working with a personal search agent means having another set of eyes looking out for you.

  1. How did Redmon find his job?
    [A] By searching openings in a job database.[B] By posting a matching position in a database.
    [C] By using a special service of a database.[D] By E-mailing his resume to a database.
  2. Which of the following can be a disadvantage of search agents?
    [A] Lack of counseling. [B] Limited number of visits.
    [C] Lower efficiency. [D] Fewer successful matches.
  3. The expression “tip service” (Line 4, Paragraph 3) most probably means ________.
    [A] advisory [B] compensation [C] interaction [D] reminder
  4. Why does CareerSite’s agent offer each job hunter only three job options?
    [A] To focus on better job matches.[B] To attract more returning visits.
    [C] To reserve space for more messages.[D] To increase the rate of success.
  5. Which of the following is true according to the text?
    [A] Personal search agents are indispensable to job-hunters.
    [B] Some sites keep E-mailing job seekers to trace their demands.
    [C] Personal search agents are also helpful to those already employed.
    [D] Some agents stop sending information to people once they are employed.
    Text 2

Over the past century, all kinds of unfairness and discrimination have been condemned or made illegal. But one insidious form continues to thrive: alphabetism. This, for those as yet unaware of such a disadvantage, refers to discrimination against those whose surnames begin with a letter in the lower half of the alphabet.
It has long been known that a taxi firm called AAAA cars has a big advantage over Zodiac cars when customers thumb through their phone directories. Less well known is the advantage that Adam Abbott has in life over Zoë Zysman. English names are fairly evenly spread between the halves of the alphabet. Yet a suspiciously large number of top people have surnames beginning with letters between A and K.
Thus the American president and vice-president have surnames starting with B and C respectively; and 26 of George Bush’s predecessors (including his father) had surnames in the first half of the alphabet against just 16 in the second half. Even more striking, six of the seven heads of government of the G7 rich countries are alphabetically advantaged (Berlusconi, Blair, Bush, Chirac, Chrétien and Koizumi). The world’s three top central bankers (Greenspan, Duisenberg and Hayami) are all close to the top of the alphabet, even if one of them really uses Japanese characters. As are the world’s five richest men (Gates, Buffett, Allen, Ellison and Albrecht).
Can this merely be coincidence? One theory, dreamt up in all the spare time enjoyed by the alphabetically disadvantaged, is that the rot sets in early. At the start of the first year in infant school, teachers seat pupils alphabetically from the front, to make it easier to remember their names. So short-sighted Zysman junior gets stuck in the back row, and is rarely asked the improving questions posed by those insensitive teachers. At the time the alphabetically disadvantaged may think they have had a lucky escape. Yet the result may be worse qualifications, because they get less individual attention, as well as less confidence in speaking publicly.
The humiliation continues. At university graduation ceremonies, the ABCs proudly get their awards first; by the time they reach the Zysmans most people are literally having a ZZZ. Shortlists for job interviews, election ballot papers, lists of conference speakers and attendees: all tend to be drawn up alphabetically, and their recipients lose interest as they plough through them.

  1. What does the author intend to illustrate with AAA A cars and Zodiac cars?
    [A] A kind of overlooked inequality.[B] A type of conspicuous bias.
    [C] A type of personal prejudice.[D] A kind of brand discrimination.
  2. What can we infer from the first three paragraphs?
    [A] In both East and West, names are essential to success.
    [B] The alphabet is to blame for the failure of Zoë Zysman.
    [C] Customers often pay a lot of attention to companies’ names.
    [D] Some form of discrimination is too subtle to recognize.
  3. The 4th paragraph suggests that ________.
    [A] questions are often put to the more intelligent students
    [B] alphabetically disadvantaged students often escape from class
    [C] teachers should pay attention to all of their students
    [D] students should be seated according to their eyesight
  4. What does the author mean by “most people are literally having a ZZZ” (Lines 2-3, Paragraph 5)?
    [A] They are getting impatient. [B] They are noisily dozing off.
    [C] They are feeling humiliated. [D] They are busy with word puzzles.
  5. Which of the following is true according to the text?
    [A] People with surnames beginning with N to Z are often ill-treated.
    [B] VIPs in the Western world gain a great deal from alphabetism.
    [C] The campaign to eliminate alphabetism still has a long way to go.
    [D] Putting things alphabetically may lead to unintentional bias.
    Text 3

When it comes to the slowing economy, Ellen Spero isn’t biting her nails just yet. But the 47-year-old manicurist isn’t cutting, filing or polishing as many nails as she’d like to, either. Most of her clients spend $12 to $50 weekly, but last month two longtime customers suddenly stopped showing up. Spero blames the softening economy. “I’m a good economic indicator,” she says. “I provide a service that people can do without when they’re concerned about saving some dollars.” So Spero is downscaling, shopping at middle-brow Dillard’s department store near her suburban Cleveland home, instead of Neiman Marcus. “I don’t know if other clients are going to abandon me, too.” she says.
Even before Alan Greenspan’s admission that America’s red-hot economy is cooling, lots of working folks had already seen signs of the slowdown themselves. From car dealerships to Gap outlets, sales have been lagging for months as shoppers temper their spending. For retailers, who last year took in 24 percent of their revenue between Thanksgiving and Christmas, the cautious approach is coming at a crucial time. Already, experts say, holiday sales are off 7 percent from last year’s pace. But don’t sound any alarms just yet. Consumers seem only mildly concerned, not panicked, and many say they remain optimistic about the economy’s long-term prospects, even as they do some modest belt-tightening.
Consumers say they’re not in despair because, despite the dreadful headlines, their own fortunes still feel pretty good. Home prices are holding steady in most regions. In Manhattan, “there’s a new gold rush happening in the $4 million to $10 million range, predominantly fed by Wall Street bonuses,” says broker Barbara Corcoran. In San Francisco, prices are still rising even as frenzied overbidding quiets. “Instead of 20 to 30 offers, now maybe you only get two or three,” says John Tealdi, a Bay Area real-estate broker. And most folks still feel pretty comfortable about their ability to find and keep a job.
Many folks see silver linings to this slowdown. Potential home buyers would cheer for lower interest rates. Employers wouldn’t mind a little fewer bubbles in the job market. Many consumers seem to have been influenced by stock-market swings, which investors now view as a necessary ingredient to a sustained boom. Diners might see an upside, too. Getting a table at Manhattan’s hot new Alain Ducasse restaurant used to be impossible. Not anymore. For that, Greenspan & Co. may still be worth toasting.

  1. By “Ellen Spero isn’t biting her nails just yet” (Lines 1-2, Paragraph 1), the author means ________.
    [A] Spero can hardly maintain her business [B] Spero is too much engaged in her work
    [C] Spero has grown out of her bad habit [D] Spero is not in a desperate situation
  2. How do the public feel about the current economic situation?
    [A] Optimistic. [B] Confused. [C] Carefree. [D] Panicked.
  3. When mentioning “the $4 million to $10 million range” (Lines 3-4, Paragraph 3) the author is talking about ________.
    [A] gold market [B] real estate [C] stock exchange [D] venture investment
  4. Why can many people see “silver linings” to the economic slowdown?
    [A] They would benefit in certain ways.[B] The stock market shows signs of recovery.
    [C] Such a slowdown usually precedes a boom.[D] The purchasing power would be enhanced.
  5. To which of the following is the author likely to agree?
    [A] A new boom, on the horizon.[B] Tighten the belt, the single remedy.
    [C] Caution all right, panic not.[D] The more ventures, the more chances.
    Text 4

Americans today don’t place a very high value on intellect. Our heroes are athletes, entertainers, and entrepreneurs, not scholars. Even our schools are where we send our children to get a practical education -- not to pursue knowledge for the sake of knowledge. Symptoms of pervasive anti-intellectualism in our schools aren’t difficult to find.
“Schools have always been in a society where practical is more important than intellectual,” says education writer Diane Ravitch. “Schools could be a counterbalance.” Ravitch’s latest book, Left Back: A Century of Failed School Reforms, traces the roots of anti-intellectualism in our schools, concluding they are anything but a counterbalance to the American distaste for intellectual pursuits.
But they could and should be. Encouraging kids to reject the life of the mind leaves them vulnerable to exploitation and control. Without the ability to think critically, to defend their ideas and understand the ideas of others, they cannot fully participate in our democracy. Continuing along this path, says writer Earl Shorris, “We will become a second-rate country. We will have a less civil society.”
“Intellect is resented as a form of power or privilege,” writes historian and professor Richard Hofstadter in Anti-Intellectualism in American Life, a Pulitzer-Prize winning book on the roots of anti-intellectualism in US politics, religion, and education. From the beginning of our history, says Hofstadter, our democratic and populist urges have driven us to reject anything that smells of elitism. Practicality, common sense, and native intelligence have been considered more noble qualities than anything you could learn from a book.
Ralph Waldo Emerson and other Transcendentalist philosophers thought schooling and rigorous book learning put unnatural restraints on children: “We are shut up in schools and college recitation rooms for 10 or 15 years and come out at last with a bellyful of words and do not know a thing.” Mark Twain’s Huckleberry Finn exemplified American anti-intellectualism. Its hero avoids being civilized -- going to school and learning to read -- so he can preserve his innate goodness.
Intellect, according to Hofstadter, is different from native intelligence, a quality we reluctantly admire. Intellect is the critical, creative, and contemplative side of the mind. Intelligence seeks to grasp, manipulate, re-order, and adjust, while intellect examines, ponders, wonders, theorizes, criticizes and imagines.
School remains a place where intellect is mistrusted. Hofstadter says our country’s educational system is in the grips of people who “joyfully and militantly proclaim their hostility to intellect and their eagerness to identify with children who show the least intellectual promise.”

  1. What do American parents expect their children to acquire in school?
    [A] The habit of thinking independently.[B] Profound knowledge of the world.
    [C] Practical abilities for future career.[D] The confidence in intellectual pursuits.
  2. We can learn from the text that Americans have a history of ________.
    [A] undervaluing intellect [B] favoring intellectualism
    [C] supporting school reform [D] suppressing native intelligence
  3. The views of Ravitch and Emerson on schooling are ________.
    [A] identical [B] similar [C] complementary [D] opposite
  4. Emerson, according to the text, is probably ________.
    [A] a pioneer of education reform [B] an opponent of intellectualism
    [C] a scholar in favor of intellect [D] an advocate of regular schooling
  5. What does the author think of intellect?
    [A] It is second to intelligence. [B] It evolves from common sense.
    [C] It is to be pursued. [D] It underlies power.
    Part B

Directions:
Read the following text carefully and then translate the underlined segments into Chinese. Your translation should be written clearly on ANSWER SHEET 2. (10 points)
The relation of language and mind has interested philosophers for many centuries. 61) The Greeks assumed that the structure of language had some connection with the process of thought, which took root in Europe long before people realized how diverse languages could be.
Only recently did linguists begin the serious study of languages that were very different from their own. Two anthropologist-linguists, Franz Boas and Edward Sapir, were pioneers in describing many native languages of North and South America during the first half of the twentieth century. 62) We are obliged to them because some of these languages have since vanished, as the peoples who spoke them died out or became assimilated and lost their native languages. Other linguists in the earlier part of this century, however, who were less eager to deal with bizarre data from “exotic” language, were not always so grateful. 63) The newly described languages were often so strikingly different from the well studied languages of Europe and Southeast Asia that some scholars even accused Boas and Sapir of fabricating their data. Native American languages are indeed different, so much so in fact that Navajo could be used by the US military as a code during World War II to send secret messages.
Sapir’s pupil, Benjamin Lee Whorf, continued the study of American Indian languages. 64) Being interested in the relationship of language and thought, Whorf developed the idea that the structure of language determines the structure of habitual thought in a society. He reasoned that because it is easier to formulate certain concepts and not others in a given language, the speakers of that language think along one track and not along another. 65) Whorf came to believe in a sort of linguistic determinism which, in its strongest form, states that language imprisons the mind, and that the grammatical patterns in a language can produce far-reaching consequences for the culture of a society. Later, this idea became to be known as the Sapir-Whorf hypothesis, but this term is somewhat inappropriate. Although both Sapir and Whorf emphasized the diversity of languages, Sapir himself never explicitly supported the notion of linguistic determinism.

  1. ________ 62. ________ 63. ________64. ________65. ________
    Section IV Writing

  2. Directions:
    Study the following drawing carefully and write an essay in which you should

  1. describe the drawing, 2) interpret its meaning, and 3) support your view with examples.
    You should write about 200 words neatly on ANSWER SHEET 2. (20 points)



    Section IV: Writing (20 points)
    2003
    Section II Use of English

Directions:
Read the following text. Choose the best word (s) for each numbered blank and mark [A], [B], [C] or [D] on ANSWER SHEET 1. (10 points)
Teachers need to be aware of the emotional, intellectual, and physical changes that young adults experience. And they also need to give serious to how they can best such changes. Growing bodies need movement and , but not just in ways that emphasize competition. they are adjusting to their new bodies and a whole host of new intellectual and emotional challenges, teenagers are especially self-conscious and need the that comes from achieving success and knowing that their accomplishments are by others. However, the typical teenage lifestyle is already filled with so much competition that it would be to plan activities in which there are more winners than losers, , publishing newsletters with many student-written book reviews, student artwork, and sponsoring book discussion clubs. A variety of small clubs can provide opportunities for leadership, as well as for practice in successful dynamics. Making friends is extremely important to teenagers, and many shy students need the of some kind of organization with a supportive adult visible in the background.
In these activities, it is important to remember that the young teens have attention spans. A variety of activities should be organized participants can remain active as long as they want and then go on to else without feeling guilty and without letting the other participants . This does not mean that adults must accept irresponsibility. , they can help students acquire a sense of commitment by for roles that are within their and their attention spans and by having clearly stated rules.

  1. [A] thought[B] idea[C] opinion[D] advice
  2. [A] strengthen[B] accommodate[C] stimulate[D] enhance
  3. [A] care[B] nutrition[C] exercise[D] leisure
  4. [A] If[B] Although[C] Whereas[D] Because
  5. [A] assistance[B] guidance[C] confidence[D] tolerance
  6. [A] claimed[B] admired[C] ignored[D] surpassed
  7. [A] improper[B] risky[C] fair[D] wise
  8. [A] in effect[B] as a result[C] for example[D] in a sense
  9. [A] displaying[B] describing[C] creating[D] exchanging
  10. [A] durable[B] excessive[C] surplus[D] multiple
  11. [A] groups[B] individual[C] personnel[D] corporation
  12. [A] consent[B] insurance[C] admission[D] security
  13. [A] particularly[B] barely[C] definitely[D] rarely
  14. [A] similar[B] long[C] different[D] short
  15. [A] if only[B] now that[C] so that[D] even if
  16. [A] everything[B] anything[C] nothing[D] something
  17. [A] off[B] down[C] out[D] alone
  18. [A] On the contrary[B] On the average[C] On the whole[D] On the other hand
  19. [A] making[B] standing[C] planning[D] taking
  20. [A] capability[B] responsibility[C] proficiency[D] efficiency
    Section III Reading Comprehension

Part A

Directions:
Read the following four texts. Answer the questions below each text by choosing [A], [B], [C] or [D]. Mark your answers on ANSWER SHEET 1 (40 points)
Text 1

Wild Bill Donovan would have loved the Internet. The American spymaster who built the Office of Strategic Services in the World War II and later laid the roots for the CIA was fascinated with information. Donovan believed in using whatever tools came to hand in the “great game” of espionage -- spying as a “profession.” These days the Net, which has already re-made such everyday pastimes as buying books and sending mail, is reshaping Donovan’s vocation as well.
The latest revolution isn’t simply a matter of gentlemen reading other gentlemen’s e-mail. That kind of electronic spying has been going on for decades. In the past three or four years, the World Wide Web has given birth to a whole industry of point-and-click spying. The spooks call it “open-source intelligence,” and as the Net grows, it is becoming increasingly influential. In 1995 the CIA held a contest to see who could compile the most data about Burundi. The winner, by a large margin, was a tiny Virginia company called Open Source Solutions, whose clear advantage was its mastery of the electronic world.
Among the firms making the biggest splash in the new world is Straitford, Inc., a private intelligence-analysis firm based in Austin, Texas. Straitford makes money by selling the results of spying (covering nations from Chile to Russia) to corporations like energy-services firm McDermott International. Many of its predictions are available online at www.straitford.com.
Straitford president George Friedman says he sees the online world as a kind of mutually reinforcing tool for both information collection and distribution, a spymaster’s dream. Last week his firm was busy vacuuming up data bits from the far corners of the world and predicting a crisis in Ukraine. “As soon as that report runs, we’ll suddenly get 500 new Internet sign-ups from Ukraine,” says Friedman, a former political science professor. “And we’ll hear back from some of them.” Open-source spying does have its risks, of course, since it can be difficult to tell good information from bad. That’s where Straitford earns its keep.
Friedman relies on a lean staff in Austin. Several of his staff members have military-intelligence backgrounds. He sees the firm’s outsider status as the key to its success. Straitford’s briefs don’t sound like the usual Washington back-and-forthing, whereby agencies avoid dramatic declarations on the chance they might be wrong. Straitford, says Friedman, takes pride in its independent voice.

  1. The emergence of the Net has ________.
    [A] received support from fans like Donovan [B] remolded the intelligence services
    [C] restored many common pastimes [D] revived spying as a profession
  2. Donovan’s story is mentioned in the text to ________.
    [A] introduce the topic of online spying [B] show how he fought for the U.S.
    [C] give an episode of the information war [D] honor his unique services to the CIA
  3. The phrase “making the biggest splash” (Line 1, Paragraph 3) most probably means ________.
    [A] causing the biggest trouble [B] exerting the greatest effort
    [C] achieving the greatest success [D] enjoying the widest popularity
  4. It can be learned from Paragraph 4 that ________.
    [A] Straitford’s prediction about Ukraine has proved true
    [B] Straitford guarantees the truthfulness of its information
    [C] Straitford’s business is characterized by unpredictability
    [D] Straitford is able to provide fairly reliable information
  5. Straitford is most proud of its ________.
    [A] official status [B] nonconformist image [C] efficient staff [D] military background
    Text 2

To paraphrase 18th-century statesman Edmund Burke, “all that is needed for the triumph of a misguided cause is that good people do nothing.” One such cause now seeks to end biomedical research because of the theory that animals have rights ruling out their use in research. Scientists need to respond forcefully to animal rights advocates, whose arguments are confusing the public and thereby threatening advances in health knowledge and care. Leaders of the animal rights movement target biomedical research because it depends on public funding, and few people understand the process of health care research. Hearing allegations of cruelty to animals in research settings, many are perplexed that anyone would deliberately harm an animal.
For example, a grandmotherly woman staffing an animal rights booth at a recent street fair was distributing a brochure that encouraged readers not to use anything that comes from or is tested in animals—no meat, no fur, no medicines. Asked if she opposed immunizations, she wanted to know if vaccines come from animal research. When assured that they do, she replied, “Then I would have to say yes.” Asked what will happen when epidemics return, she said, “Don’t worry, scientists will find some way of using computers.” Such well-meaning people just don’t understand.
Scientists must communicate their message to the public in a compassionate, understandable way -- in human terms, not in the language of molecular biology. We need to make clear the connection between animal research and a grandmother’s hip replacement, a father’s bypass operation, a baby’s vaccinations, and even a pet’s shots. To those who are unaware that animal research was needed to produce these treatments, as well as new treatments and vaccines, animal research seems wasteful at best and cruel at worst.
Much can be done. Scientists could “adopt” middle school classes and present their own research. They should be quick to respond to letters to the editor, lest animal rights misinformation go unchallenged and acquire a deceptive appearance of truth. Research institutions could be opened to tours, to show that laboratory animals receive humane care. Finally, because the ultimate stakeholders are patients, the health research community should actively recruit to its cause not only well-known personalities such as Stephen Cooper, who has made courageous statements about the value of animal research, but all who receive medical treatment. If good people do nothing there is a real possibility that an uninformed citizenry will extinguish the precious embers of medical progress.

  1. The author begins his article with Edmund Burke’s words to ________.
    [A] call on scientists to take some actions[B] criticize the misguided cause of animal rights
    [C] warn of the doom of biomedical research[D] show the triumph of the animal rights movement
  2. Misled people tend to think that using an animal in research is ________.
    [A] cruel but natural [B] inhuman and unacceptable
    [C] inevitable but vicious [D] pointless and wasteful
  3. The example of the grandmotherly woman is used to show the public’s ________.
    [A] discontent with animal research [B] ignorance about medical science
    [C] indifference to epidemics [D] anxiety about animal rights
  4. The author believes that, in face of the challenge from animal rights advocates, scientists should ________.
    [A] communicate more with the public [B] employ hi-tech means in research
    [C] feel no shame for their cause [D] strive to develop new cures
  5. From the text we learn that Stephen Cooper is ________.
    [A] a well-known humanist [B] a medical practitioner
    [C] an enthusiast in animal rights [D] a supporter of animal research
    Text 3

In recent years, railroads have been combining with each other, merging into super systems, causing heightened concerns about monopoly. As recently as 1995, the top four railroads accounted for under 70 percent of the total ton-miles moved by rails. Next year, after a series of mergers is completed, just four railroads will control well over 90 percent of all the freight moved by major rail carriers.
Supporters of the new super systems argue that these mergers will allow for substantial cost reductions and better coordinated service. Any threat of monopoly, they argue, is removed by fierce competition from trucks. But many shippers complain that for heavy bulk commodities traveling long distances, such as coal, chemicals, and grain, trucking is too costly and the railroads therefore have them by the throat.
The vast consolidation within the rail industry means that most shippers are served by only one rail company. Railroads typically charge such “captive” shippers 20 to 30 percent more than they do when another railroad is competing for the business. Shippers who feel they are being overcharged have the right to appeal to the federal government’s Surface Transportation Board for rate relief, but the process is expensive, time-consuming, and will work only in truly extreme cases.
Railroads justify rate discrimination against captive shippers on the grounds that in the long run it reduces everyone’s cost. If railroads charged all customers the same average rate, they argue, shippers who have the option of switching to trucks or other forms of transportation would do so, leaving remaining customers to shoulder the cost of keeping up the line. It’s a theory to which many economists subscribe, but in practice it often leaves railroads in the position of determining which companies will flourish and which will fail. “Do we really want railroads to be the arbiters of who wins and who loses in the marketplace?” asks Martin Bercovici, a Washington lawyer who frequently represents shipper.
Many captive shippers also worry they will soon be hit with a round of huge rate increases. The railroad industry as a whole, despite its brightening fortunes, still does not earn enough to cover the cost of the capital it must invest to keep up with its surging traffic. Yet railroads continue to borrow billions to acquire one another, with Wall Street cheering them on. Consider the $10.2 billion bid by Norfolk Southern and CSX to acquire Conrail this year. Conrail’s net railway operating income in 1996 was just $427 million, less than half of the carrying costs of the transaction. Who’s going to pay for the rest of the bill? Many captive shippers fear that they will, as Norfolk Southern and CSX increase their grip on the market.

  1. According to those who support mergers, railway monopoly is unlikely because ________.
    [A] cost reduction is based on competition[B] services call for cross-trade coordination
    [C] outside competitors will continue to exist[D] shippers will have the railway by the throat
  2. What is many captive shippers’ attitude towards the consolidation in the rail industry?
    [A] Indifferent. [B] Supportive. [C] Indignant.[D] Apprehensive.
  3. It can be inferred from Paragraph 3 that ________.
    [A] shippers will be charged less without a rival railroad
    [B] there will soon be only one railroad company nationwide
    [C] overcharged shippers are unlikely to appeal for rate relief
    [D] a government board ensures fair play in railway business
  4. The word “arbiters” (Line 7, Paragraph 4) most probably refers to those ________.
    [A] who work as coordinators [B] who function as judges
    [C] who supervise transactions[D] who determine the price
  5. According to the text, the cost increase in the rail industry is mainly caused by ________.
    [A] the continuing acquisition [B] the growing traffic
    [C] the cheering Wall Street[D] the shrinking market
    Text 4

It is said that in England death is pressing, in Canada inevitable and in California optional. Small wonder. Americans’ life expectancy has nearly doubled over the past century. Failing hips can be replaced, clinical depression controlled, cataracts removed in a 30-minutes surgical procedure. Such advances offer the aging population a quality of life that was unimaginable when I entered medicine 50 years ago. But not even a great health-care system can cure death -- and our failure to confront that reality now threatens this greatness of ours.
Death is normal; we are genetically programmed to disintegrate and perish, even under ideal conditions. We all understand that at some level, yet as medical consumers we treat death as a problem to be solved. Shielded by third-party payers from the cost of our care, we demand everything that can possibly be done for us, even if it’s useless. The most obvious example is late-stage cancer care. Physicians -- frustrated by their inability to cure the disease and fearing loss of hope in the patient -- too often offer aggressive treatment far beyond what is scientifically justified.
In 1950, the U.S. spent $12.7 billion on health care. In 2002, the cost will be $1,540 billion. Anyone can see this trend is unsustainable. Yet few seem willing to try to reverse it. Some scholars conclude that a government with finite resources should simply stop paying for medical care that sustains life beyond a certain age -- say 83 or so. Former Colorado governor Richard Lamm has been quoted as saying that the old and infirm “have a duty to die and get out of the way,” so that younger, healthier people can realize their potential.
I would not go that far. Energetic people now routinely work through their 60s and beyond, and remain dazzlingly productive. At 78, Viacom chairman Sumner Redstone jokingly claims to be 53. Supreme Court Justice Sandra Day O’Connor is in her 70s, and former surgeon general C. Everett Koop chairs an Internet start-up in his 80s. These leaders are living proof that prevention works and that we can manage the health problems that come naturally with age. As a mere 68-year-old, I wish to age as productively as they have.
Yet there are limits to what a society can spend in this pursuit. Ask a physician, I know the most costly and dramatic measures may be ineffective and painful. I also know that people in Japan and Sweden, countries that spend far less on medical care, have achieved longer, healthier lives than we have. As a nation, we may be overfunding the quest for unlikely cures while underfunding research on humbler therapies that could improve people’s lives.

  1. What is implied in the first sentence?
    [A] Americans are better prepared for death than other people.
    [B] Americans enjoy a higher life quality than ever before.
    [C] Americans are over-confident of their medical technology.
    [D] Americans take a vain pride in their long life expectancy.
  2. The author uses the example of cancer patients to show that ________.
    [A] medical resources are often wasted [B] doctors are helpless against fatal diseases
    [C] some treatments are too aggressive[D] medical costs are becoming unaffordable
  3. The author’s attitude toward Richard Lamm’s remark is one of ________.
    [A] strong disapproval[B] reserved consent[C] slight contempt[D] enthusiastic support
  4. In contrast to the U.S., Japan and Sweden are funding their medical care ________.
    [A] more flexibly[B] more extravagantly[C] more cautiously[D] more reasonably
  5. The text intends to express the idea that ________.
    [A] medicine will further prolong people’s lives[B] life beyond a certain limit is not worth living
    [C] death should be accepted as a fact of life[D] excessive demands increase the cost of health care
    Part B

Directions:
Read the following text carefully and then translate the underlined segments into Chinese. Your translation should be written clearly on ANSWER SHEET 2. (10 points)
Human beings in all times and places think about their world and wonder at their place in it. Humans are thoughtful and creative, possessed of insatiable curiosity. 61) Furthermore, humans have the ability to modify the environment in which they live, thus subjecting all other life forms to their own peculiar ideas and fancies. Therefore, it is important to study humans in all their richness and diversity in a calm and systematic manner, with the hope that the knowledge resulting from such studies can lead humans to a more harmonious way of living with themselves and with all other life forms on this planet Earth.
“Anthropology” derives from the Greek words anthropos “human” and logos “the study of.” By its very name, anthropology encompasses the study of all humankind.
Anthropology is one of the social sciences. 62) Social science is that branch of intellectual enquiry which seeks to study humans and their endeavors in the same reasoned, orderly, systematic, and dispassioned manner that natural scientists use for the study of natural phenomena.
Social science disciplines include geography, economics, political science, psychology, and sociology. Each of these social sciences has a subfield or specialization which lies particularly close to anthropology.
All the social sciences focus upon the study of humanity. Anthropology is a field-study oriented discipline which makes extensive use of the comparative method in analysis. 63) The emphasis on data gathered first-hand, combined with a cross-cultural perspective brought to the analysis of cultures past and present, makes this study a unique and distinctly important social science.
Anthropological analyses rest heavily upon the concept of culture. Sir Edward Tylor’s formulation of the concept of culture was one of the great intellectual achievements of 19th century science. 64) Tylor defined culture as “… that complex whole which includes belief, art, morals, law, custom, and any other capabilities and habits acquired by man as a member of society.” This insight, so profound in its simplicity, opened up an entirely new way of perceiving and understanding human life. Implicit within Tylor’s definition is the concept that culture is learned, shared, and patterned behavior.

  1. Thus, the anthropological concept of “culture,” like the concept of “set” in mathematics, is an abstract concept which makes possible immense amounts of concrete research and understanding.
    Section IV Writing
  1. Directions:
    Study the following set of drawings carefully and write an essay in which you should
  1. describe the set of drawings, interpret its meaning, and
  2. point out its implications in our life.
    You should write about 200 words neatly on ANSWER SHEET 2. (20 points)



    **

最后编辑于
©著作权归作者所有,转载或内容合作请联系作者
  • 序言:七十年代末,一起剥皮案震惊了整个滨河市,随后出现的几起案子,更是在滨河造成了极大的恐慌,老刑警刘岩,带你破解...
    沈念sama阅读 148,827评论 1 317
  • 序言:滨河连续发生了三起死亡事件,死亡现场离奇诡异,居然都是意外死亡,警方通过查阅死者的电脑和手机,发现死者居然都...
    沈念sama阅读 63,511评论 1 266
  • 文/潘晓璐 我一进店门,熙熙楼的掌柜王于贵愁眉苦脸地迎上来,“玉大人,你说我怎么就摊上这事。” “怎么了?”我有些...
    开封第一讲书人阅读 99,318评论 0 218
  • 文/不坏的土叔 我叫张陵,是天一观的道长。 经常有香客问我,道长,这世上最难降的妖魔是什么? 我笑而不...
    开封第一讲书人阅读 42,108评论 0 189
  • 正文 为了忘掉前任,我火速办了婚礼,结果婚礼上,老公的妹妹穿的比我还像新娘。我一直安慰自己,他们只是感情好,可当我...
    茶点故事阅读 50,112评论 1 266
  • 文/花漫 我一把揭开白布。 她就那样静静地躺着,像睡着了一般。 火红的嫁衣衬着肌肤如雪。 梳的纹丝不乱的头发上,一...
    开封第一讲书人阅读 39,387评论 1 185
  • 那天,我揣着相机与录音,去河边找鬼。 笑死,一个胖子当着我的面吹牛,可吹牛的内容都是我干的。 我是一名探鬼主播,决...
    沈念sama阅读 30,905评论 2 283
  • 文/苍兰香墨 我猛地睁开眼,长吁一口气:“原来是场噩梦啊……” “哼!你这毒妇竟也来了?” 一声冷哼从身侧响起,我...
    开封第一讲书人阅读 29,657评论 0 177
  • 序言:老挝万荣一对情侣失踪,失踪者是张志新(化名)和其女友刘颖,没想到半个月后,有当地人在树林里发现了一具尸体,经...
    沈念sama阅读 33,144评论 0 223
  • 正文 独居荒郊野岭守林人离奇死亡,尸身上长有42处带血的脓包…… 初始之章·张勋 以下内容为张勋视角 年9月15日...
    茶点故事阅读 29,744评论 2 225
  • 正文 我和宋清朗相恋三年,在试婚纱的时候发现自己被绿了。 大学时的朋友给我发了我未婚夫和他白月光在一起吃饭的照片。...
    茶点故事阅读 31,100评论 1 236
  • 序言:一个原本活蹦乱跳的男人离奇死亡,死状恐怖,灵堂内的尸体忽然破棺而出,到底是诈尸还是另有隐情,我是刑警宁泽,带...
    沈念sama阅读 27,565评论 2 222
  • 正文 年R本政府宣布,位于F岛的核电站,受9级特大地震影响,放射性物质发生泄漏。R本人自食恶果不足惜,却给世界环境...
    茶点故事阅读 32,041评论 3 216
  • 文/蒙蒙 一、第九天 我趴在偏房一处隐蔽的房顶上张望。 院中可真热闹,春花似锦、人声如沸。这庄子的主人今日做“春日...
    开封第一讲书人阅读 25,769评论 0 9
  • 文/苍兰香墨 我抬头看了看天上的太阳。三九已至,却和暖如春,着一层夹袄步出监牢的瞬间,已是汗流浃背。 一阵脚步声响...
    开封第一讲书人阅读 26,301评论 0 178
  • 我被黑心中介骗来泰国打工, 没想到刚下飞机就差点儿被人妖公主榨干…… 1. 我叫王不留,地道东北人。 一个月前我还...
    沈念sama阅读 34,173评论 2 239
  • 正文 我出身青楼,却偏偏与公主长得像,于是被迫代替她去往敌国和亲。 传闻我的和亲对象是个残疾皇子,可洞房花烛夜当晚...
    茶点故事阅读 34,313评论 2 242

推荐阅读更多精彩内容

  • 烈日当头 艳阳高照 地表散发的热气如蒸笼般 惹得人燥动不安 门前的花花草草 耷拉下了脑袋 迫切渴望着 雨水的滋润 ...
    lz枝子阅读 388评论 1 5
  • 人生,即是一班列车。生为始,死为终。 活着,目的是领略沿途风光,感悟车厢内的悲欢离合、世态炎凉。 于是,出现了一个...
    虚凡UU阅读 589评论 0 3
  • 你来 我欣喜若狂 踩着细碎的阳光去接你 愉悦洒了一地 你走 我难忍不舍 和着蒙蒙的飘雨去送你 悲伤润了一路
    半似凉秋阅读 333评论 2 5
  • 刚开始接触vue.js,感觉还没入门就被一脚踹了出来。 以下是安装环境时踩过的坑的总结: 首先需要最新的node环...
    走错说爱你阅读 424评论 0 4
  • 蓝蓝是我工作以后结识的朋友,91年出生的她,从小就一直是大家口中“别人家的孩子”。 成绩优异、知书达理、五官精致、...
    非常非的非阅读 734评论 2 7